Sie sind auf Seite 1von 1235

Set Theory

A set is an unordered collection of objects without repetitions.

Set Theory

A set is an unordered collection of objects without repetitions. Example {a , b , c } = {b , c , a }

Set Theory

A set is an unordered collection of objects without repetitions. Example {a , b , c } = {b , c , a } We usually use capital letters to denote sets. A = {a, b , c } B = {b , c , d , e }

Set Theory

A set is an unordered collection of objects without repetitions. Example {a , b , c } = {b , c , a } We usually use capital letters to denote sets. A = {a, b , c } B = {b , c , d , e }

Sets are the nouns of Set Theory, theyre what its about.

Relations

In mathematics a relation is like a verb: it relates two things in a way that can be either true or false.

Relations

In mathematics a relation is like a verb: it relates two things in a way that can be either true or false. In high school algebra the common relations are =, <, >.

Relations

In mathematics a relation is like a verb: it relates two things in a way that can be either true or false. In high school algebra the common relations are =, <, >. In Set Theory the most common relations are =, , , .


The expression AB means that every object (or element) of A is also in B .


The expression AB means that every object (or element) of A is also in B . Example: A B A B B Also, A A B B, so is analagous to . = = {a, b , c } {a, b , c , d , e } B A A

The expression aA means that the object a is contained in A.

The expression aA means that the object a is contained in A. Example: A a b d = {a , b , c } A A A

Operations

An operation is something you do to objects to get new ones. In algebra the common operations are +, , , , etc.

Operations

An operation is something you do to objects to get new ones. In algebra the common operations are +, , , , etc. In Set Theory the common operations are , , , and .

The expression AB means combine all elements of A with those of B .

The expression AB means combine all elements of A with those of B . Example: A = {a, b , c } B = {b , c , d , e } A B = {a, b , c , d , e }

The expression AB means combine all elements of A with those of B . Example: A = {a, b , c } B = {b , c , d , e } A B = {a, b , c , d , e } Loosely, means or.

The expression AB means take only those elements of A which are also in B .

The expression AB means take only those elements of A which are also in B . Example: A = {a, b , c } B = {b , c , d , e } A B = {b , c }

The expression AB means take only those elements of A which are also in B . Example: A = {a, b , c } B = {b , c , d , e } A B = {b , c } Loosely, means and. But the English word and sometimes means or!

U
The letter U is used to denote the universe, the set of every object that matters in a problem.

U
The letter U is used to denote the universe, the set of every object that matters in a problem. Example: If a problem is about English letters, the universe might be U = {a, b , c , d , e , f , g , h, i , j , k , l , m, n, o , p , q , r , s , t , u , v , w , x , y , z }. But if its about Greek letters the universe would be U = {, , , , , , , , , , , , , , o , , , , , , , , , }. If its about rolling dice the universe might be U = {1, 2, 3, 4, 5, 6}.

The expression A is read A complement and denotes the objects of U not in A.

The expression A is read A complement and denotes the objects of U not in A. Example: U = {1, 2, 3, 4, 5, 6} E = {2, 4, 6} E = {1, 3, 5}

The expression A is read A complement and denotes the objects of U not in A. Example: U = {1, 2, 3, 4, 5, 6} E = {2, 4, 6} E = {1, 3, 5} Loosely, means not.

Example: Suppose that U = F = A = all animals sh aquatic animals W M S = = = whales mammals seals.

Translate the following English phrases into Set Theory. 1. Whales are aquatic mammals. 2. Whales are not sh.

Example: Suppose that U = F = A = all animals sh aquatic animals W M S = = = whales mammals seals.

Translate the following English phrases into Set Theory. 1. Whales are aquatic mammals. 2. Whales are not sh. 1. W W W W = = AM AM AM AM

Example: Suppose that U = F = A = all animals sh aquatic animals W M S = = = whales mammals seals.

Translate the following English phrases into Set Theory. 1. Whales are aquatic mammals. 2. Whales are not sh. 1. Solution: W AM

Example: Suppose that U = F = A = all animals sh aquatic animals W M S = = = whales mammals seals.

Translate the following English phrases into Set Theory. 1. Whales are aquatic mammals. 2. Whales are not sh.

Example: Suppose that U = F = A = all animals sh aquatic animals W M S = = = whales mammals seals.

Translate the following English phrases into Set Theory. 1. Whales are aquatic mammals. 2. Whales are not sh. 2. W W W W F = = F F F = {}

Example: Suppose that U = F = A = all animals sh aquatic animals W M S = = = whales mammals seals.

Translate the following English phrases into Set Theory. 1. Whales are aquatic mammals. 2. Whales are not sh. 2. Solution:

W W F

F =

Venn Diagrams
A Venn diagram is a visual representation of sets.

How can we describe the shaded region?

(A B ) (A B ) (A B )

How can we describe the shaded region?

(A B ) (A B ) (A B ) (A B )

How can we describe the shaded region?

(A B C ) (A B C ) (A B C )

How can we describe the shaded region?

(A B C ) (A B C ) (A B C ) (A B ) C

Using Venn Diagrams to Count

Venn diagrams can be helpful in some counting problems. Well use the notation n[A] to denote the number of elements in A. Example: A = {a, r , y , p , j } n[A] = 5 There are two important principles: AP The addition principle, also called the partition principle; CP the complement principle.

Disjoint

Sets are disjoint if they dont overlap: A B = {} = .

Example: disjoint: not disjoint: {a, d , t , p }, {b , f , o , v , z } {a, d , t , p }, {b , f , p , v , z }

AP
The addition principle, or partition principle, says that the size of the union of two disjoint sets is the sum of the sizes of each one. If A B = then n[A B ] = n[A] + n[B ].

AP
The addition principle, or partition principle, says that the size of the union of two disjoint sets is the sum of the sizes of each one. If A B = then n[A B ] = n[A] + n[B ].

AP
The addition principle, or partition principle, says that the size of the union of two disjoint sets is the sum of the sizes of each one. If A B = then n[A B ] = n[A] + n[B ].

Example: What is n[A]?

Example: What is n[A]?

n[A] = 4 + 3 = 7

Example: What is n[A B ]?

Example: What is n[A B ]?

n[A B ] = 3 + 5 + 12 = 20

CP

The complement principle is based on the observation that U =AA. n[A] + n[A ] = n[U ]

CP

The complement principle is based U =AA. n[A] + n[A ] = n[A] = n[A ] =

on the observation that n[U ] n[U ] n[A ] n[U ] n[A].

CP

The complement principle is based U =AA. n[A] + n[A ] = n[A] = n[A ] =

on the observation that n[U ] n[U ] n[A ] n[U ] n[A].

So if you know n[U ] and either n[A] or n[A ] you can gure out the other.

Example: What is n[A]?

Example: What is n[A]?

n[A] = 15 (5 + 7) = 3

Example: What is n[A B C ]?

Example: What is n[A B C ]?

n[A] = 40 (3 + 5 + 6 + 12) = 14

Play VennDoku at http://mypage.iu.edu/dabrowsa/FiniteJS.html!

Example: There are 100 members of Beta Epsilon Rho fraternity, of whom 60 are business majors and 50 are psych majors; 20 are majoring in both business and psych. How many members of the fraternity are majoring in neither business nor psych?

Example: There are 100 members of Beta Epsilon Rho fraternity, of whom 60 are business majors and 50 are psych majors; 20 are majoring in both business and psych. How many members of the fraternity are majoring in neither business nor psych? Venn Diagram

Example: The 100 members of Beta Epsilon Rho fraternity are ordering pizza. Pepperoni is requested by 80 members, and Brussels sprouts by 15; 12 members want neither pepperoni nor Brussels sprouts. How many want both pepperoni and Brussels sprouts?

Example: The 100 members of Beta Epsilon Rho fraternity are ordering pizza. Pepperoni is requested by 80 members, and Brussels sprouts by 15; 12 members want neither pepperoni nor Brussels sprouts. How many want both pepperoni and Brussels sprouts? Venn Diagram

Note that 12+? = 100, so ? = 88.

Now 80+? = 88, so ? = 8.

Now work across.

Answer: 7 people like pizza with pepperoni and Brussels sprouts.

Alternate Method

Alternate Method

n[B ] + n[P ] = n[B P ] + n[B P ]

Alternate Method

n[B ] + n[P ] = n[B P ] + n[B P ] 15 + 80 = 88 +

Alternate Method

n[B ] + n[P ] = n[B P ] + n[B P ] 15 + 80 = 88 + = 7

Inclusion-Exclusion Formula

In general, n[A] + n[B ] = n[A B ] + n[A B ] n[A B ] = n[A] + n[B ] n[A B ] n[A B ] = n[A] + n[B ] n[A B ].

Example: According to a survey of the members of an investment club, 150 own stock in Alcoa, 180 own stock in Boeing, 100 own stock in Coke, 70 own stock in Alcoa and Boeing, 30 own stock in Alcoa and Coke, 50 own stock in Boeing and Coke, 20 own stock in all three companies, and 60 own stock in none of these of companies. 1. How many members were surveyed? 2. How many of those surveyed do not own stock in Coke? 3. How many own stock in Boeing but not Alcoa? 4. How many own stock in Alcoa or Boeing and in Coke? 5. How many own stock in Alcoa or in Boeing and Coke? 6. How many own stock in Alcoa or Boeing but not in Coke? 7. How many own stock in exactly 1 company? 8. How many own stock in exactly 2 companies?

Example: According to a survey of the members of an investment club, 150 own stock in Alcoa, 180 own stock in Boeing, 100 own stock in Coke, 70 own stock in Alcoa and Boeing, 30 own stock in Alcoa and Coke, 50 own stock in Boeing and Coke, 20 own stock in all three companies, and 60 own stock in none of these of companies. Venn Diagram

Work from the middle out.

Keep going.

Finish up.

Now we can start answering the questions.

How many members were surveyed? 360

How many of those surveyed do not own stock in Coke? CP: 360 100 = 260

How many own stock in Boeing but not Alcoa? n[B A ] = 80 + 30 = 110

How many own stock in Alcoa or Boeing and in Coke? n[(A B ) C ] = 10 + 20 + 30 = 60

How many own stock in Alcoa or in Boeing and Coke? n[A (B C )] = 150 + 30 = 180

How many own stock in Alcoa or Boeing but not in Coke? n[(A B ) C ] = 70 + 50 + 80 = 200

How many own stock in exactly 1 company? 70 + 80 + 40 = 190

How many own stock in exactly 2 companies?

How many own stock in exactly 2 companies? 10 + 30 + 50 = 90

Example: Of the 360 members of the Investment Club, 180 own stock in Alcoa (A), 180 own stock in Boeing (B), 120 own stock in Coke (C), 100 own stock in Alcoa and Boeing, 90 own stock in Alcoa and Coke, 70 own stock in Boeing and Coke, and 80 own stock in none of these companies. 1. How many members own stock in all three companies? 2. How many members own stock in exactly one of Boeing and Coke? 3. How many own stock in Coke or in Boeing and Alcoa? 4. How many do not own stock in Boeing and Coke? 5. How many do not own stock in Alcoa or Coke?

Example: Of the 360 members of the Investment Club, 180 own stock in Alcoa (A), 180 own stock in Boeing (B), 120 own stock in Coke (C), 100 own stock in Alcoa and Boeing, 90 own stock in Alcoa and Coke, 70 own stock in Boeing and Coke, and 80 own stock in none of these companies.

Use the Inclusion-Exclusion formula on A B . n[A B ] = n[A] + n[B ] n[A B ] = 180 + 180 100

= 260

Now 360 = 260 + 80 + ? ? = 20.

Now focus on C : 120 = 20 + 70 + ? ? = 30.

Now focus on A C : 90 = 30 + ? ? = 60.

Now that we have the center its easy to work from the middle out.

How many members own stock in all three companies?

How many members own stock in all three companies? 60

How many members own stock in exactly one of Boeing and Coke?

How many members own stock in exactly one of Boeing and Coke? 40 + 70 + 30 + 20 = 160

How many own stock in Coke or in Boeing and Alcoa?

How many own stock in Coke or in Boeing and Alcoa? 120 + 40 = 160

How many do not own stock in Boeing and Coke?

How many do not own stock in Boeing and Coke? CP: 360 70 = 290

How many do not own stock in Alcoa or Coke?

How many do not own stock in Alcoa or Coke? CP: 360 210 = 150

Partitions
A partition of a set A is a division of A up into nonoverlapping pieces. Example: If A = {1, 2, 3, 4, 5, 6} then A1 = {1, 6} A2 = {2, 3, 5} A3 = {4} is a partition of A. Of course the AP (PP) automatically applies: n[A] = n[A1 ] + n[A2 ] + n[A3 ].

Example: Suppose that n[A] = 120 and A is partitioned into 4 subsets W , X , Y , and Z . Suppose further that W has 12 elements, X and Y are the same size, and Z is twice as large as X . Find the sizes of X , Y , and Z .

Example: Suppose that n[A] = 120 and A is partitioned into 4 subsets W , X , Y , and Z . Suppose further that W has 12 elements, X and Y are the same size, and Z is twice as large as X . Find the sizes of X , Y , and Z . A partition can always be diagrammed as a pizza.

Example: Suppose that n[A] = 120 and A is partitioned into 4 subsets W , X , Y , and Z . Suppose further that W has 12 elements, X and Y are the same size, and Z is twice as large as X . Find the sizes of X , Y , and Z . A partition can always be diagrammed as a pizza.

Fill in the sizes using a variable if necessary.

Example: Suppose that n[A] = 120 and A is partitioned into 4 subsets W , X , Y , and Z . Suppose further that W has 12 elements, X and Y are the same size, and Z is twice as large as X . Find the sizes of X , Y , and Z . X is the smallest piece, start with that and call its size x .

Y has the same size, and Z has size 2x .

Example: Suppose that n[A] = 120 and A is partitioned into 4 subsets W , X , Y , and Z . Suppose further that W has 12 elements, X and Y are the same size, and Z is twice as large as X . Find the sizes of X , Y , and Z . X is the smallest piece, start with that and call its size x .

Y has the same size, and Z has size 2x . Now use the AP: 120 = 12 + x + x + 2x

Example: Suppose that n[A] = 120 and A is partitioned into 4 subsets W , X , Y , and Z . Suppose further that W has 12 elements, X and Y are the same size, and Z is twice as large as X . Find the sizes of X , Y , and Z . X is the smallest piece, start with that and call its size x .

Y has the same size, and Z has size 2x . Now use the AP: 120 = 12 + x + x + 2x 108 = 4x 27 = x .

Example: Suppose that n[A] = 120 and A is partitioned into 4 subsets W , X , Y , and Z . Suppose further that W has 12 elements, X and Y are the same size, and Z is twice as large as X . Find the sizes of X , Y , and Z . Now we can determine the other sizes.

= 27

n[X ] = 27 n[Y ] = 27 n[Z ] = 54

Outcomes: What Matters?


Example 1: A sale table at a jewelry store has 5 bracelets, 4 necklaces, and 8 pairs of earrings, all dierent and all at the same price. Shelly selects 3 items to purchase. How many outcomes are possible? Example 2: A sale table has 6 red scarves, 4 blue scarves, 6 green scarves, and 6 purple scarves. Emily buys one of the red scarves, Kristen buys one of the blue scarves, Sara buys one of the green scarves, and Laura buys one of the purple scarves. How many outcomes are possible? Example 3: 6 IU students and 5 Purdue students go to the Summer Intern Job Fair in Indianapolis. Bank One, Neimann Marcus, Conseco, and the Lilly company each get one summer intern from this group. How many outcomes are possible? Example 4: A sale catalogue lists 6 rock CDs and 4 country CDs. Each of Kim, Stacey, and Lynn picks out her favorite CD from this list. How many outcomes are possible?

All the examples examples involve selecting some objects. The details involve:

All the examples examples involve selecting some objects. The details involve:

All the examples examples involve selecting some objects. The details involve: 1. What objects are you choosing? (Domain)

All the examples examples involve selecting some objects. The details involve: 1. What objects are you choosing? (Domain)

All the examples examples involve selecting some objects. The details involve: 1. What objects are you choosing? (Domain) 2. Do dierent objects have dierent qualities that matter? (Qualities)

All the examples examples involve selecting some objects. The details involve: 1. What objects are you choosing? (Domain) 2. Do dierent objects have dierent qualities that matter? (Qualities)

All the examples examples involve selecting some objects. The details involve: 1. What objects are you choosing? (Domain) 2. Do dierent objects have dierent qualities that matter? (Qualities) 3. Do you do dierent things with dierent selected objects? (Roles)

All the examples examples involve selecting some objects. The details involve: 1. What objects are you choosing? (Domain) 2. Do dierent objects have dierent qualities that matter? (Qualities) 3. Do you do dierent things with dierent selected objects? (Roles)

All the examples examples involve selecting some objects. The details involve: 1. What objects are you choosing? (Domain) 2. Do dierent objects have dierent qualities that matter? (Qualities) 3. Do you do dierent things with dierent selected objects? (Roles) 4. Can the same object be selected multiple times? (Replacement)

Example: A sale table at a jewelry store has 5 bracelets, 4 necklaces, and 8 pairs of earrings, all dierent and all at the same price. Gladys selects 3 items to purchase. How many outcomes are possible? Analysis:

Example: A sale table at a jewelry store has 5 bracelets, 4 necklaces, and 8 pairs of earrings, all dierent and all at the same price. Gladys selects 3 items to purchase. How many outcomes are possible? Analysis:

Example: A sale table at a jewelry store has 5 bracelets, 4 necklaces, and 8 pairs of earrings, all dierent and all at the same price. Gladys selects 3 items to purchase. How many outcomes are possible? Analysis: 1. Domain: 17 items of jewelry.

Example: A sale table at a jewelry store has 5 bracelets, 4 necklaces, and 8 pairs of earrings, all dierent and all at the same price. Gladys selects 3 items to purchase. How many outcomes are possible? Analysis: 1. Domain: 17 items of jewelry.

Example: A sale table at a jewelry store has 5 bracelets, 4 necklaces, and 8 pairs of earrings, all dierent and all at the same price. Gladys selects 3 items to purchase. How many outcomes are possible? Analysis: 1. Domain: 17 items of jewelry. 2. Qualities: none that matter.

Example: A sale table at a jewelry store has 5 bracelets, 4 necklaces, and 8 pairs of earrings, all dierent and all at the same price. Gladys selects 3 items to purchase. How many outcomes are possible? Analysis: 1. Domain: 17 items of jewelry. 2. Qualities: none that matter.

Example: A sale table at a jewelry store has 5 bracelets, 4 necklaces, and 8 pairs of earrings, all dierent and all at the same price. Gladys selects 3 items to purchase. How many outcomes are possible? Analysis: 1. Domain: 17 items of jewelry. 2. Qualities: none that matter. 3. Roles: none.

Example: A sale table at a jewelry store has 5 bracelets, 4 necklaces, and 8 pairs of earrings, all dierent and all at the same price. Gladys selects 3 items to purchase. How many outcomes are possible? Analysis: 1. Domain: 17 items of jewelry. 2. Qualities: none that matter. 3. Roles: none.

Example: A sale table at a jewelry store has 5 bracelets, 4 necklaces, and 8 pairs of earrings, all dierent and all at the same price. Gladys selects 3 items to purchase. How many outcomes are possible? Analysis: 1. Domain: 17 items of jewelry. 2. Qualities: none that matter. 3. Roles: none. 4. Replacement: no.

Example: A sale table has 6 red scarves, 4 blue scarves, 6 green scarves, and 6 purple scarves. Emily buys one of the red scarves, Kristen buys one of the blue scarves, Sara buys one of the green scarves, and Laura buys one of the purple scarves. How many outcomes are possible? Analysis:

Example: A sale table has 6 red scarves, 4 blue scarves, 6 green scarves, and 6 purple scarves. Emily buys one of the red scarves, Kristen buys one of the blue scarves, Sara buys one of the green scarves, and Laura buys one of the purple scarves. How many outcomes are possible? Analysis:

Example: A sale table has 6 red scarves, 4 blue scarves, 6 green scarves, and 6 purple scarves. Emily buys one of the red scarves, Kristen buys one of the blue scarves, Sara buys one of the green scarves, and Laura buys one of the purple scarves. How many outcomes are possible? Analysis: 1. Domain: 22 scarves.

Example: A sale table has 6 red scarves, 4 blue scarves, 6 green scarves, and 6 purple scarves. Emily buys one of the red scarves, Kristen buys one of the blue scarves, Sara buys one of the green scarves, and Laura buys one of the purple scarves. How many outcomes are possible? Analysis: 1. Domain: 22 scarves.

Example: A sale table has 6 red scarves, 4 blue scarves, 6 green scarves, and 6 purple scarves. Emily buys one of the red scarves, Kristen buys one of the blue scarves, Sara buys one of the green scarves, and Laura buys one of the purple scarves. How many outcomes are possible? Analysis: 1. Domain: 22 scarves. 2. Qualities: red, blue, green, and purple.

Example: A sale table has 6 red scarves, 4 blue scarves, 6 green scarves, and 6 purple scarves. Emily buys one of the red scarves, Kristen buys one of the blue scarves, Sara buys one of the green scarves, and Laura buys one of the purple scarves. How many outcomes are possible? Analysis: 1. Domain: 22 scarves. 2. Qualities: red, blue, green, and purple.

Example: A sale table has 6 red scarves, 4 blue scarves, 6 green scarves, and 6 purple scarves. Emily buys one of the red scarves, Kristen buys one of the blue scarves, Sara buys one of the green scarves, and Laura buys one of the purple scarves. How many outcomes are possible? Analysis: 1. Domain: 22 scarves. 2. Qualities: red, blue, green, and purple. 3. Roles: Emily, Kristen, Sara, and Laura.

Example: A sale table has 6 red scarves, 4 blue scarves, 6 green scarves, and 6 purple scarves. Emily buys one of the red scarves, Kristen buys one of the blue scarves, Sara buys one of the green scarves, and Laura buys one of the purple scarves. How many outcomes are possible? Analysis: 1. Domain: 22 scarves. 2. Qualities: red, blue, green, and purple. 3. Roles: Emily, Kristen, Sara, and Laura.

Example: A sale table has 6 red scarves, 4 blue scarves, 6 green scarves, and 6 purple scarves. Emily buys one of the red scarves, Kristen buys one of the blue scarves, Sara buys one of the green scarves, and Laura buys one of the purple scarves. How many outcomes are possible? Analysis: 1. Domain: 22 scarves. 2. Qualities: red, blue, green, and purple. 3. Roles: Emily, Kristen, Sara, and Laura. 4. Replacement: no.

Example: 6 IU students and 5 Purdue students go to the Summer Intern Job Fair in Indianapolis. Bank One, Neimann Marcus, Conseco, and the Lilly company each get one summer intern from this group. How many outcomes are possible? Analysis:

Example: 6 IU students and 5 Purdue students go to the Summer Intern Job Fair in Indianapolis. Bank One, Neimann Marcus, Conseco, and the Lilly company each get one summer intern from this group. How many outcomes are possible? Analysis:

Example: 6 IU students and 5 Purdue students go to the Summer Intern Job Fair in Indianapolis. Bank One, Neimann Marcus, Conseco, and the Lilly company each get one summer intern from this group. How many outcomes are possible? Analysis: 1. Domain: 11 students.

Example: 6 IU students and 5 Purdue students go to the Summer Intern Job Fair in Indianapolis. Bank One, Neimann Marcus, Conseco, and the Lilly company each get one summer intern from this group. How many outcomes are possible? Analysis: 1. Domain: 11 students.

Example: 6 IU students and 5 Purdue students go to the Summer Intern Job Fair in Indianapolis. Bank One, Neimann Marcus, Conseco, and the Lilly company each get one summer intern from this group. How many outcomes are possible? Analysis: 1. Domain: 11 students. 2. Qualities: none that matter.

Example: 6 IU students and 5 Purdue students go to the Summer Intern Job Fair in Indianapolis. Bank One, Neimann Marcus, Conseco, and the Lilly company each get one summer intern from this group. How many outcomes are possible? Analysis: 1. Domain: 11 students. 2. Qualities: none that matter.

Example: 6 IU students and 5 Purdue students go to the Summer Intern Job Fair in Indianapolis. Bank One, Neimann Marcus, Conseco, and the Lilly company each get one summer intern from this group. How many outcomes are possible? Analysis: 1. Domain: 11 students. 2. Qualities: none that matter. 3. Roles: Bank One, Neimann Marcus, Conseco, and Lilly.

Example: 6 IU students and 5 Purdue students go to the Summer Intern Job Fair in Indianapolis. Bank One, Neimann Marcus, Conseco, and the Lilly company each get one summer intern from this group. How many outcomes are possible? Analysis: 1. Domain: 11 students. 2. Qualities: none that matter. 3. Roles: Bank One, Neimann Marcus, Conseco, and Lilly.

Example: 6 IU students and 5 Purdue students go to the Summer Intern Job Fair in Indianapolis. Bank One, Neimann Marcus, Conseco, and the Lilly company each get one summer intern from this group. How many outcomes are possible? Analysis: 1. Domain: 11 students. 2. Qualities: none that matter. 3. Roles: Bank One, Neimann Marcus, Conseco, and Lilly. 4. Replacement: no.

Example: A sale catalogue lists 6 rock CDs and 4 country CDs. Each of Kim, Stacey, and Lynn picks out her favorite CD from this list. How many outcomes are possible? Analysis:

Example: A sale catalogue lists 6 rock CDs and 4 country CDs. Each of Kim, Stacey, and Lynn picks out her favorite CD from this list. How many outcomes are possible? Analysis:

Example: A sale catalogue lists 6 rock CDs and 4 country CDs. Each of Kim, Stacey, and Lynn picks out her favorite CD from this list. How many outcomes are possible? Analysis: 1. Domain: 10 CDs.

Example: A sale catalogue lists 6 rock CDs and 4 country CDs. Each of Kim, Stacey, and Lynn picks out her favorite CD from this list. How many outcomes are possible? Analysis: 1. Domain: 10 CDs.

Example: A sale catalogue lists 6 rock CDs and 4 country CDs. Each of Kim, Stacey, and Lynn picks out her favorite CD from this list. How many outcomes are possible? Analysis: 1. Domain: 10 CDs. 2. Qualities: none that matter.

Example: A sale catalogue lists 6 rock CDs and 4 country CDs. Each of Kim, Stacey, and Lynn picks out her favorite CD from this list. How many outcomes are possible? Analysis: 1. Domain: 10 CDs. 2. Qualities: none that matter.

Example: A sale catalogue lists 6 rock CDs and 4 country CDs. Each of Kim, Stacey, and Lynn picks out her favorite CD from this list. How many outcomes are possible? Analysis: 1. Domain: 10 CDs. 2. Qualities: none that matter. 3. Roles: Kim, Stacey, and Lynn.

Example: A sale catalogue lists 6 rock CDs and 4 country CDs. Each of Kim, Stacey, and Lynn picks out her favorite CD from this list. How many outcomes are possible? Analysis: 1. Domain: 10 CDs. 2. Qualities: none that matter. 3. Roles: Kim, Stacey, and Lynn.

Example: A sale catalogue lists 6 rock CDs and 4 country CDs. Each of Kim, Stacey, and Lynn picks out her favorite CD from this list. How many outcomes are possible? Analysis: 1. Domain: 10 CDs. 2. Qualities: none that matter. 3. Roles: Kim, Stacey, and Lynn. 4. Replacement: yes.

Shorthand
Well need a shorthand to use for analyzing problems.

Shorthand
Well need a shorthand to use for analyzing problems. Well use D to stand for all possible choices of object.

Shorthand
Well need a shorthand to use for analyzing problems. Well use D to stand for all possible choices of object. Well use S to stand for all possible outcomes.

Shorthand
Well need a shorthand to use for analyzing problems. Well use D to stand for all possible choices of object. Well use S to stand for all possible outcomes. So if you have to pick one tomato out of 40, let D = 40 tomatoes and then the set S of all possible outcomes can be represented by role: quality: how many: Tomato D 1.

That was the simplest possible example.

Now suppose you must choose two tomatoes.

Now suppose you must choose two tomatoes. Roles dont matter (both tomatoes suer the same fate).

Now suppose you must choose two tomatoes. Roles dont matter (both tomatoes suer the same fate). No Replacements (you cant eat the same tomato twice).

Now suppose you must choose two tomatoes. Roles dont matter (both tomatoes suer the same fate). No Replacements (you cant eat the same tomato twice). Well use to indicate that there are no replacements.

Now suppose you must choose two tomatoes. Roles dont matter (both tomatoes suer the same fate). No Replacements (you cant eat the same tomato twice). Well use Then T and S is described by T 2 Note the role was omitted, because there is only one. . = 40 tomatoes to indicate that there are no replacements.

Now suppose you also need an onion, and that there are 50 onions to choose from. Thats a dierent quality .

Now suppose you also need an onion, and that there are 50 onions to choose from. Thats a dierent quality . Ill denote the tomatoes by T and the onions by O .

Now suppose you also need an onion, and that there are 50 onions to choose from. Thats a dierent quality . Ill denote the tomatoes by T and the onions by O . Then S is described by T 2 O 1 .

Now suppose that you need one tomato for a marinara sauce, and the other tomato and onion for a salad. Now there are two roles , sauce and salad.

Now suppose that you need one tomato for a marinara sauce, and the other tomato and onion for a salad. Now there are two roles , sauce and salad. Then S is described by Sauce T 1 Salad T O . 1 1

Example: A sale table at a jewelry store has 5 bracelets, 4 necklaces, and 8 pairs of earrings, all dierent and all at the same price. Gladys selects 3 items to purchase. How many outcomes are possible? Analysis: 1. Domain: 17 items of jewelry. 2. Qualities: none that matter. 3. Roles: none. 4. Replacement: no.

Example: A sale table at a jewelry store has 5 bracelets, 4 necklaces, and 8 pairs of earrings, all dierent and all at the same price. Gladys selects 3 items to purchase. How many outcomes are possible? Analysis: 1. Domain: 17 items of jewelry. 2. Qualities: none that matter. 3. Roles: none. 4. Replacement: no. Therefore, D = 17 articles of jewelry and S = D 3 .

Example: A sale table has 6 red scarves, 4 blue scarves, 6 green scarves, and 6 purple scarves. Emily buys one of the red scarves, Kristen buys one of the blue scarves, Sara buys one of the green scarves, and Laura buys one of the purple scarves. How many outcomes are possible? Analysis: 1. Domain: 22 scarves. 2. Qualities: red, blue, green, and purple. 3. Roles: Emily, Kristen, Sara, and Laura. 4. Replacement: no.

Example: A sale table has 6 red scarves, 4 blue scarves, 6 green scarves, and 6 purple scarves. Emily buys one of the red scarves, Kristen buys one of the blue scarves, Sara buys one of the green scarves, and Laura buys one of the purple scarves. How many outcomes are possible? Analysis: 1. Domain: 22 scarves. 2. Qualities: red, blue, green, and purple. 3. Roles: Emily, Kristen, Sara, and Laura. 4. Replacement: no. Therefore, R = 6 red scarves G = 6 green scarves and S = Emily R 1 Kristen B 1 Sara G 1 Laura P . 1 B = 4 blue scarves P = 6 purple scarves

Example: 6 IU students and 5 Purdue students go to the Summer Intern Job Fair in Indianapolis. Bank One, Neimann Marcus, Conseco, and the Lilly company each get one summer intern from this group. How many outcomes are possible? Analysis: 1. Domain: 11 students. 2. Qualities: none that matter. 3. Roles: Bank One, Neimann Marcus, Conseco, and Lilly. 4. Replacement: no.

Example: 6 IU students and 5 Purdue students go to the Summer Intern Job Fair in Indianapolis. Bank One, Neimann Marcus, Conseco, and the Lilly company each get one summer intern from this group. How many outcomes are possible? Analysis: 1. Domain: 11 students. 2. Qualities: none that matter. 3. Roles: Bank One, Neimann Marcus, Conseco, and Lilly. 4. Replacement: no. Therefore, D = 11 students and S = Bank1 D 1 Neimann D 1 Conseco D 1 Lilly D . 1

Example: A sale catalogue lists 6 rock CDs and 4 country CDs. Each of Kim, Stacey, and Lynn picks out her favorite CD from this list. How many outcomes are possible? Analysis: 1. Domain: 10 CDs. 2. Qualities: none that matter. 3. Roles: Kim, Stacey, and Lynn. 4. Replacement: yes.

Example: A sale catalogue lists 6 rock CDs and 4 country CDs. Each of Kim, Stacey, and Lynn picks out her favorite CD from this list. How many outcomes are possible? Analysis: 1. Domain: 10 CDs. 2. Qualities: none that matter. 3. Roles: Kim, Stacey, and Lynn. 4. Replacement: yes. Therefore, D = 10 CDs and S = Kim D 1 Stacey D 1 Lynn D . 1

Paradigms
Paradigm: City Council Committee: A city council has 5 Democratic members (Anne, Bill, Cathy, Dan, and Ellen) and 3 Republican members (Frank, Gina, and Hank). The mayor is to appoint a committee consisting of 3 council members. Analysis:

Paradigms
Paradigm: City Council Committee: A city council has 5 Democratic members (Anne, Bill, Cathy, Dan, and Ellen) and 3 Republican members (Frank, Gina, and Hank). The mayor is to appoint a committee consisting of 3 council members. Analysis: 1. Domain: 8 council members 2. Qualities: none that matter 3. Roles: none that matter 4. Replacement: no

Paradigms
Paradigm: City Council Committee: A city council has 5 Democratic members (Anne, Bill, Cathy, Dan, and Ellen) and 3 Republican members (Frank, Gina, and Hank). The mayor is to appoint a committee consisting of 3 council members. Analysis: 1. Domain: 8 council members 2. Qualities: none that matter 3. Roles: none that matter 4. Replacement: no Therefore, D = 8 council members and S = D 3 .

Paradigm: Soup, Salad, Entree, Veg: For dinner, a customer at a restaurant must select one of 3 soups, one of 5 salads, one of 4 entrees, and one of 6 vegetables (see the menu below). Analysis:

Paradigm: Soup, Salad, Entree, Veg: For dinner, a customer at a restaurant must select one of 3 soups, one of 5 salads, one of 4 entrees, and one of 6 vegetables (see the menu below). Analysis: 1. Domain: 18 menu items 2. Qualities: 3 soups, 5 salads, 4 entrees, 6 vegetables 3. Roles: soup, salad, entree, vegetable 4. Replacement: no (n/a)

Paradigm: Soup, Salad, Entree, Veg: For dinner, a customer at a restaurant must select one of 3 soups, one of 5 salads, one of 4 entrees, and one of 6 vegetables (see the menu below). Analysis: 1. Domain: 18 menu items 2. Qualities: 3 soups, 5 salads, 4 entrees, 6 vegetables 3. Roles: soup, salad, entree, vegetable 4. Replacement: no (n/a) Therefore, P = 3 soups E = 4 entrees and S = P 1 L 1 E 1 V 1 . L = 5 salads V = 6 vegetables

Paradigm: Soup, Salad, Entree, Veg: For dinner, a customer at a restaurant must select one of 3 soups, one of 5 salads, one of 4 entrees, and one of 6 vegetables (see the menu below). Analysis: 1. Domain: 18 menu items 2. Qualities: 3 soups, 5 salads, 4 entrees, 6 vegetables 3. Roles: soup, salad, entree, vegetable 4. Replacement: no (n/a) Therefore, P = 3 soups E = 4 entrees and S = P 1 L 1 E 1 V 1 . L = 5 salads V = 6 vegetables

Another notation for this is the Cartesian Product: S =P LE V

Paradigm: Coee, Tea, Coke, Sprite: John, Mary, and Bill must each select one of four beverages for dinner (for instance, coee, tea, coke, or sprite). Analysis:

Paradigm: Coee, Tea, Coke, Sprite: John, Mary, and Bill must each select one of four beverages for dinner (for instance, coee, tea, coke, or sprite). Analysis: 1. Domain: 4 beverages 2. Qualities: none that matter 3. Roles: John, Mary, and Bill 4. Replacement: yes

Paradigm: Coee, Tea, Coke, Sprite: John, Mary, and Bill must each select one of four beverages for dinner (for instance, coee, tea, coke, or sprite). Analysis: 1. Domain: 4 beverages 2. Qualities: none that matter 3. Roles: John, Mary, and Bill 4. Replacement: yes Therefore, D = 4 beverages and S = John D 1 Mary D 1 Bill D . 1

Paradigm: Casting a Play: A play has three female roles: the grandmother, the mother, and the daughter. Six (6) women Anne, Barb, Carol, Debra, Ellen, and Fran audition for these roles. (One should presume that no one can ll two roles, because all three characters may have to be on stage simultaneously.) Analysis:

Paradigm: Casting a Play: A play has three female roles: the grandmother, the mother, and the daughter. Six (6) women Anne, Barb, Carol, Debra, Ellen, and Fran audition for these roles. (One should presume that no one can ll two roles, because all three characters may have to be on stage simultaneously.) Analysis: 1. Domain: 6 Actresses 2. Qualities: none that matter 3. Roles: Grandmother, Mother, Daughter 4. Replacement: no

Paradigm: Casting a Play: A play has three female roles: the grandmother, the mother, and the daughter. Six (6) women Anne, Barb, Carol, Debra, Ellen, and Fran audition for these roles. (One should presume that no one can ll two roles, because all three characters may have to be on stage simultaneously.) Analysis: 1. Domain: 6 Actresses 2. Qualities: none that matter 3. Roles: Grandmother, Mother, Daughter 4. Replacement: no Therefore, D = 6 Actresses and S = Grandmother D 1 Mother D 1 Daughter D . 1

Paradigm City Council Committee Qualities no Soup, Salad, Entree, Veg yes Coee, Tea, Coke, Sprite no Casting a Play no

Roles

no

yes

yes

yes

Replacement

no

no (n/a)

yes

no

Example: At dinner one night at summer camp, 8 campers and a counselor are sitting at a long table. The counselor must select 3 of the campers to carry the dishes to the cleanup window. How many overall outcomes are possible?

Example: At dinner one night at summer camp, 8 campers and a counselor are sitting at a long table. The counselor must select 3 of the campers to carry the dishes to the cleanup window. How many overall outcomes are possible? Analysis: 1. Domain: 8 campers (Counseler? Apply Creative Tunnel Vision.) 2. Qualities: none that matter 3. Roles: none 4. Replacement: no

Example: At dinner one night at summer camp, 8 campers and a counselor are sitting at a long table. The counselor must select 3 of the campers to carry the dishes to the cleanup window. How many overall outcomes are possible? Analysis: 1. Domain: 8 campers (Counseler? Apply Creative Tunnel Vision.) 2. Qualities: none that matter 3. Roles: none 4. Replacement: no City Council Committee Paradigm

Example: At dinner one night at summer camp, 8 campers and a counselor are sitting at a long table. The counselor must select 3 of the campers to carry the dishes to the cleanup window. How many overall outcomes are possible? Analysis: 1. Domain: 8 campers (Counseler? Apply Creative Tunnel Vision.) 2. Qualities: none that matter 3. Roles: none 4. Replacement: no City Council Committee Paradigm Therefore, D = 8 campers and S = D 3 .

Example: At dinner one night at summer camp, 8 campers and a counselor are sitting at a long table. The counselor must select 3 of the campers to carry the dishes to the cleanup window. How many overall outcomes are possible? Alternate Analysis: 1. Domain: 9 people at dinner 2. Qualities: 8 campers, 1 counseler 3. Roles: none 4. Replacement: no Therefore, D1 = 8 campers D2 = 1 counseler and S = D1 3 .

Example: A voter must select one city council candidate, one school board candidate, and one county commissioner candidate. There are 4 candidates for the city council, 9 candidates for the school board, and 5 candidates for county commissioner.

Example: A voter must select one city council candidate, one school board candidate, and one county commissioner candidate. There are 4 candidates for the city council, 9 candidates for the school board, and 5 candidates for county commissioner. Analysis: 1. Domain: 18 candidates 2. Qualities: city council, school board, county commission 3. Roles: city council, school board, county commission 4. Replacement: no

Example: A voter must select one city council candidate, one school board candidate, and one county commissioner candidate. There are 4 candidates for the city council, 9 candidates for the school board, and 5 candidates for county commissioner. Analysis: 1. Domain: 18 candidates 2. Qualities: city council, school board, county commission 3. Roles: city council, school board, county commission 4. Replacement: no Soup, Salad, Entree, Veg Paradigm

Example: A voter must select one city council candidate, one school board candidate, and one county commissioner candidate. There are 4 candidates for the city council, 9 candidates for the school board, and 5 candidates for county commissioner. Analysis: 1. Domain: 18 candidates 2. Qualities: city council, school board, county commission 3. Roles: city council, school board, county commission 4. Replacement: no Soup, Salad, Entree, Veg Paradigm Therefore, D1 = 4 city council candidates D2 = 9 school board candidates D3 = 5 county commission candidates D1 1 D2 1 D3 1

Example: Mary, Paul and Ryan are late for breakfast in the dorm. When they arrive, the pastry tray has only 8 dierent donuts (e.g., one glazed donut, one bear claw, one apple fritter, etc.), 4 dierent Danishes (e.g., one cheese, one cheery, etc.), and 4 dierent muns (e.g., one blueberry, one apple, etc.). Each of Mary, Paul, and Ryan takes one of these pastries.

Example: Mary, Paul and Ryan are late for breakfast in the dorm. When they arrive, the pastry tray has only 8 dierent donuts (e.g., one glazed donut, one bear claw, one apple fritter, etc.), 4 dierent Danishes (e.g., one cheese, one cheery, etc.), and 4 dierent muns (e.g., one blueberry, one apple, etc.). Each of Mary, Paul, and Ryan takes one of these pastries. Analysis: 1. Domain: 16 pastries 2. Qualities: none that matter 3. Roles: Mary, Paul, Ryan 4. Replacement: no

Example: Mary, Paul and Ryan are late for breakfast in the dorm. When they arrive, the pastry tray has only 8 dierent donuts (e.g., one glazed donut, one bear claw, one apple fritter, etc.), 4 dierent Danishes (e.g., one cheese, one cheery, etc.), and 4 dierent muns (e.g., one blueberry, one apple, etc.). Each of Mary, Paul, and Ryan takes one of these pastries. Analysis: 1. Domain: 16 pastries 2. Qualities: none that matter 3. Roles: Mary, Paul, Ryan 4. Replacement: no Casting a Play Paradigm

Example: Mary, Paul and Ryan are late for breakfast in the dorm. When they arrive, the pastry tray has only 8 dierent donuts (e.g., one glazed donut, one bear claw, one apple fritter, etc.), 4 dierent Danishes (e.g., one cheese, one cheery, etc.), and 4 dierent muns (e.g., one blueberry, one apple, etc.). Each of Mary, Paul, and Ryan takes one of these pastries. Analysis: 1. Domain: 16 pastries 2. Qualities: none that matter 3. Roles: Mary, Paul, Ryan 4. Replacement: no Casting a Play Paradigm Therefore, D = 16 pastries, and S = Mary D 1 Paul D 1 Ryan D . 1

Example: Claire, Emily, and Laura are at a store that sells works by local artists. The store has 6 oil paintings, 5 water color sketches, and 9 textile pieces for sale. Clair purchases an oil painting, Emily purchases a water color sketch, and Laura purchases a textile piece.

Example: Claire, Emily, and Laura are at a store that sells works by local artists. The store has 6 oil paintings, 5 water color sketches, and 9 textile pieces for sale. Clair purchases an oil painting, Emily purchases a water color sketch, and Laura purchases a textile piece. Analysis: 1. Domain: 20 artworks 2. Qualities: oil, water color, textile 3. Roles: Clair, Emily, Laura 4. Replacement: no

Example: Claire, Emily, and Laura are at a store that sells works by local artists. The store has 6 oil paintings, 5 water color sketches, and 9 textile pieces for sale. Clair purchases an oil painting, Emily purchases a water color sketch, and Laura purchases a textile piece. Analysis: 1. Domain: 20 artworks 2. Qualities: oil, water color, textile 3. Roles: Clair, Emily, Laura 4. Replacement: no Soup, Salad, Entree, Veg Paradigm

Example: Claire, Emily, and Laura are at a store that sells works by local artists. The store has 6 oil paintings, 5 water color sketches, and 9 textile pieces for sale. Clair purchases an oil painting, Emily purchases a water color sketch, and Laura purchases a textile piece. Analysis: 1. Domain: 20 artworks 2. Qualities: oil, water color, textile 3. Roles: Clair, Emily, Laura 4. Replacement: no Soup, Salad, Entree, Veg Paradigm Therefore, O = 6 oil paintings and S = Clair O 1 Emily W 1 Laura T . 1 W = 5 water colors T = 9 textiles

Example: The Dean of Students has 4 Associate Deans, 5 Assistant Deans, and 9 program specialists on his sta. To check for alcohol violations this weekend, the Dean is going to select one of these sta members to visit the Sigma Lambda house at 11:00 pm on Saturday, one of these sta members to visit the Delta Phi house at 11:00 pm on Saturday, and one of these sta members to visit the Phi Sigma house at 11:00 pm on Saturday.

Example: The Dean of Students has 4 Associate Deans, 5 Assistant Deans, and 9 program specialists on his sta. To check for alcohol violations this weekend, the Dean is going to select one of these sta members to visit the Sigma Lambda house at 11:00 pm on Saturday, one of these sta members to visit the Delta Phi house at 11:00 pm on Saturday, and one of these sta members to visit the Phi Sigma house at 11:00 pm on Saturday. Analysis: 1. Domain: 19 sta members 2. Qualities: none that matter 3. Roles: , , 4. Replacement: no

Example: The Dean of Students has 4 Associate Deans, 5 Assistant Deans, and 9 program specialists on his sta. To check for alcohol violations this weekend, the Dean is going to select one of these sta members to visit the Sigma Lambda house at 11:00 pm on Saturday, one of these sta members to visit the Delta Phi house at 11:00 pm on Saturday, and one of these sta members to visit the Phi Sigma house at 11:00 pm on Saturday. Analysis: 1. Domain: 19 sta members 2. Qualities: none that matter 3. Roles: , , 4. Replacement: no Casting a Play Paradigm

Example: The Dean of Students has 4 Associate Deans, 5 Assistant Deans, and 9 program specialists on his sta. To check for alcohol violations this weekend, the Dean is going to select one of these sta members to visit the Sigma Lambda house at 11:00 pm on Saturday, one of these sta members to visit the Delta Phi house at 11:00 pm on Saturday, and one of these sta members to visit the Phi Sigma house at 11:00 pm on Saturday. Analysis: 1. Domain: 19 sta members 2. Qualities: none that matter 3. Roles: , , 4. Replacement: no Casting a Play Paradigm Therefore, D = 18 sta members, and S = D 1 D 1 D . 1

Example: Laura, Susan, and Chris have just enrolled in a university in Chicago. Each one must open a checking/savings account at a local institution. They may choose from 8 (regular) banks, 4 savings banks, and 4 credit unions.

Example: Laura, Susan, and Chris have just enrolled in a university in Chicago. Each one must open a checking/savings account at a local institution. They may choose from 8 (regular) banks, 4 savings banks, and 4 credit unions. Analysis: 1. Domain: 16 banks 2. Qualities: none that matter 3. Roles: Laura, Susan, Chris 4. Replacement: yes

Example: Laura, Susan, and Chris have just enrolled in a university in Chicago. Each one must open a checking/savings account at a local institution. They may choose from 8 (regular) banks, 4 savings banks, and 4 credit unions. Analysis: 1. Domain: 16 banks 2. Qualities: none that matter 3. Roles: Laura, Susan, Chris 4. Replacement: yes Coee, Tea, Coke, Sprite Paradigm

Example: Laura, Susan, and Chris have just enrolled in a university in Chicago. Each one must open a checking/savings account at a local institution. They may choose from 8 (regular) banks, 4 savings banks, and 4 credit unions. Analysis: 1. Domain: 16 banks 2. Qualities: none that matter 3. Roles: Laura, Susan, Chris 4. Replacement: yes Coee, Tea, Coke, Sprite Paradigm Therefore, B = 16 banks and S = Laura B 1 Susan B 1 Chris B . 1

Example: Alyssa, Brandy,and Carl are having dinner at a Chinese restaurant. The menu lists 9 beef dishes, 6 pork dishes, and 6 chicken dishes. Alyssa, Brandy, and Carl have decided that they will share 3 dishes. They jointly select one beef dish, one pork dish, and one chicken dish.

Example: Alyssa, Brandy,and Carl are having dinner at a Chinese restaurant. The menu lists 9 beef dishes, 6 pork dishes, and 6 chicken dishes. Alyssa, Brandy, and Carl have decided that they will share 3 dishes. They jointly select one beef dish, one pork dish, and one chicken dish. Analysis: 1. Domain: 21 dishes 2. Qualities: beef, pork, chicken 3. Roles: beef, pork, chicken 4. Replacement: yes(?), n/a

Example: Alyssa, Brandy,and Carl are having dinner at a Chinese restaurant. The menu lists 9 beef dishes, 6 pork dishes, and 6 chicken dishes. Alyssa, Brandy, and Carl have decided that they will share 3 dishes. They jointly select one beef dish, one pork dish, and one chicken dish. Analysis: 1. Domain: 21 dishes 2. Qualities: beef, pork, chicken 3. Roles: beef, pork, chicken 4. Replacement: yes(?), n/a Soup, Salad, Entree, Veg Paradigm

Example: Alyssa, Brandy,and Carl are having dinner at a Chinese restaurant. The menu lists 9 beef dishes, 6 pork dishes, and 6 chicken dishes. Alyssa, Brandy, and Carl have decided that they will share 3 dishes. They jointly select one beef dish, one pork dish, and one chicken dish. Analysis: 1. Domain: 21 dishes 2. Qualities: beef, pork, chicken 3. Roles: beef, pork, chicken 4. Replacement: yes(?), n/a Soup, Salad, Entree, Veg Paradigm Therefore, B = 9 beef dishes and S = B 1 P 1 C 1 . P = 6 pork dishes C = 6 chicken dishes

Example: Allyson, Beth, and Chelsea, who are sharing an apartment, have gone to a bookstore to purchase 3 cookbooks. The store had 5 dierent general cookbooks, 3 dierent Italian cookbooks, and 6 dierent French cookbooks. Together they select 3 of these cookbooks.

Example: Allyson, Beth, and Chelsea, who are sharing an apartment, have gone to a bookstore to purchase 3 cookbooks. The store had 5 dierent general cookbooks, 3 dierent Italian cookbooks, and 6 dierent French cookbooks. Together they select 3 of these cookbooks. Analysis: 1. Domain: 14 cookbooks 2. Qualities: none that matter 3. Roles: none 4. Replacement: no

Example: Allyson, Beth, and Chelsea, who are sharing an apartment, have gone to a bookstore to purchase 3 cookbooks. The store had 5 dierent general cookbooks, 3 dierent Italian cookbooks, and 6 dierent French cookbooks. Together they select 3 of these cookbooks. Analysis: 1. Domain: 14 cookbooks 2. Qualities: none that matter 3. Roles: none 4. Replacement: no City Council Committee Paradigm

Example: Allyson, Beth, and Chelsea, who are sharing an apartment, have gone to a bookstore to purchase 3 cookbooks. The store had 5 dierent general cookbooks, 3 dierent Italian cookbooks, and 6 dierent French cookbooks. Together they select 3 of these cookbooks. Analysis: 1. Domain: 14 cookbooks 2. Qualities: none that matter 3. Roles: none 4. Replacement: no City Council Committee Paradigm Therefore, C and S = C 3 . = 14 cookbooks

Calculating Sizes

Now that you know how to analyze problems, the next thing is to learn: How do you use your analysis to count the number of outcomes?

Soup, Salad, Entree, Veg Paradigm S E and S = P # choices: 1 3 L 1 5 E 1 4 V 1 . 6 = 3 soups = 4 entrees L = 5 salads V = 6 vegetables

Soup, Salad, Entree, Veg Paradigm S E and S = P # choices: Answer: n[S ] = size of S is n[S ] = 3 5 4 6 = 360. 1 3 L 1 5 E 1 4 V 1 . 6 = 3 soups = 4 entrees L = 5 salads V = 6 vegetables

Coee, Tea, Coke, Sprite Paradigm D = 4 beverages and S = Laura B # choices: 1 4 Susan B 1 4 Chris B 1 4

Coee, Tea, Coke, Sprite Paradigm D = 4 beverages and S = Laura B # choices: Answer: n[S ] = 4 4 4 = 43 = 64. 1 4 Susan B 1 4 Chris B 1 4

Coee, Tea, Coke, Sprite Paradigm D = 4 beverages and S = Laura B # choices: Answer: n[S ] = 4 4 4 = 43 = 64. 1 4 Susan B 1 4 Chris B 1 4

In general, if d = size of domain and r = # roles, n[S ] = d r .

Casting a Play Paradigm D = 6 Actresses and S = Grandmother D # choices: 1 6 Mother D 1 5 Daughter D 1 4

Casting a Play Paradigm D = 6 Actresses and S = Grandmother D # choices: Answer: n[S ] = 6 5 4 = P(6, 3) = 120. 1 6 Mother D 1 5 Daughter D 1 4

Casting a Play Paradigm D = 6 Actresses and S = Grandmother D # choices: Answer: n[S ] = 6 5 4 = P(6, 3) = 120. 1 6 Mother D 1 5 Daughter D 1 4

In general, if d = size of domain and r = # roles, n[S ] = P(d , r ).

City Council Committee Paradigm D = 8 council members and S = D # choices: . 3 C(8, 3) = 56

City Council Committee Paradigm D = 8 council members and S = D # choices: Answer: n[S ] = C(8, 3) = 56. . 3 C(8, 3) = 56

City Council Committee Paradigm D = 8 council members and S = D # choices: Answer: n[S ] = C(8, 3) = 56. . 3 C(8, 3) = 56

In general, if d = size of domain and n = # chosen, n[S ] = C(d , n).

Combinations
The C function calculates the number of combinations. A combination is an unordered list without repetitions. (A permutation is an ordered list.) An outcome in the City Council Committee Paradigm is a combo.

Combinations
The C function calculates the number of combinations. A combination is an unordered list without repetitions. (A permutation is an ordered list.) An outcome in the City Council Committee Paradigm is a combo. Mathematically, C(d , r ) = For example, C(8, 3) = C(8, 5) =
876 321 P(d ,r ) P(r ,r ) .

876 321

= 56 = 56.

87654 54321

87654 54321

Combinations
The C function calculates the number of combinations. A combination is an unordered list without repetitions. (A permutation is an ordered list.) An outcome in the City Council Committee Paradigm is a combo. Mathematically, C(d , r ) = For example, C(8, 3) = C(8, 5) = In general, C(d , r ) = C(d , d r ).
876 321 P(d ,r ) P(r ,r ) .

876 321

= 56 = 56.

87654 54321

87654 54321

Example: At dinner one night at summer camp, 8 campers and a counselor are sitting at a long table. The counselor must select 3 of the campers to carry the dishes to the cleanup window. How many overall outcomes are possible?

Example: At dinner one night at summer camp, 8 campers and a counselor are sitting at a long table. The counselor must select 3 of the campers to carry the dishes to the cleanup window. How many overall outcomes are possible? City Council Committee Paradigm D = 8 campers and S = D . 3 C(8, 3) = 56

Example: At dinner one night at summer camp, 8 campers and a counselor are sitting at a long table. The counselor must select 3 of the campers to carry the dishes to the cleanup window. How many overall outcomes are possible? City Council Committee Paradigm D = 8 campers and S = D . 3 C(8, 3) = 56 Answer: n[S ] = C(8, 3) = 56

Example: A voter must select one city council candidate, one school board candidate, and one county commissioner candidate. There are 4 candidates for the city council, 9 candidates for the school board, and 5 candidates for county commissioner. D1 = 4 city council candidates D2 = 9 school board candidates D3 = 5 county commission candidates and S = City Council D1 1 4 School Board D2 1 9 County Commission D3 1 5

Example: A voter must select one city council candidate, one school board candidate, and one county commissioner candidate. There are 4 candidates for the city council, 9 candidates for the school board, and 5 candidates for county commissioner. D1 = 4 city council candidates D2 = 9 school board candidates D3 = 5 county commission candidates and S = City Council D1 1 4 School Board D2 1 9 County Commission D3 1 5

Soup, Salad, Entree, Veg Paradigm

Example: A voter must select one city council candidate, one school board candidate, and one county commissioner candidate. There are 4 candidates for the city council, 9 candidates for the school board, and 5 candidates for county commissioner. D1 = 4 city council candidates D2 = 9 school board candidates D3 = 5 county commission candidates and S = City Council D1 1 4 School Board D2 1 9 County Commission D3 1 5

Soup, Salad, Entree, Veg Paradigm Answer: n[S ] = 4 9 5 = 180

Example: Mary, Paul and Ryan are late for breakfast in the dorm. When they arrive, the pastry tray has only 8 dierent donuts (e.g., one glazed donut, one bear claw, one apple fritter, etc.), 4 dierent Danishes (e.g., one cheese, one cheery, etc.), and 4 dierent muns (e.g., one blueberry, one apple, etc.). Each of Mary, Paul, and Ryan takes one of these pastries. D = 16 pastries and S = Mary D 1 16 Paul D 1 15 Ryan D 1 14.

Example: Mary, Paul and Ryan are late for breakfast in the dorm. When they arrive, the pastry tray has only 8 dierent donuts (e.g., one glazed donut, one bear claw, one apple fritter, etc.), 4 dierent Danishes (e.g., one cheese, one cheery, etc.), and 4 dierent muns (e.g., one blueberry, one apple, etc.). Each of Mary, Paul, and Ryan takes one of these pastries. D = 16 pastries and S = Mary D 1 16 Casting a Play Paradigm Paul D 1 15 Ryan D 1 14.

Example: Mary, Paul and Ryan are late for breakfast in the dorm. When they arrive, the pastry tray has only 8 dierent donuts (e.g., one glazed donut, one bear claw, one apple fritter, etc.), 4 dierent Danishes (e.g., one cheese, one cheery, etc.), and 4 dierent muns (e.g., one blueberry, one apple, etc.). Each of Mary, Paul, and Ryan takes one of these pastries. D = 16 pastries and S = Mary D 1 16 Casting a Play Paradigm Answer: n[S ] = P(16, 3) = 16 15 14 = 3360 Paul D 1 15 Ryan D 1 14.

Example: Claire, Emily, and Laura are at a store that sells works by local artists. The store has 6 oil paintings, 5 water color sketches, and 9 textile pieces for sale. All of the items are dierent from one another. Clair purchases an oil painting, Emily purchases a water color sketch, and Laura purchases a textile piece. O = 6 oil paintings W = 5 water colors T = 9 textiles and S = Claire O 1 6 Emily W 1 5 Laura T 1 9

Example: Claire, Emily, and Laura are at a store that sells works by local artists. The store has 6 oil paintings, 5 water color sketches, and 9 textile pieces for sale. All of the items are dierent from one another. Clair purchases an oil painting, Emily purchases a water color sketch, and Laura purchases a textile piece. O = 6 oil paintings W = 5 water colors T = 9 textiles and S = Claire O 1 6 Emily W 1 5 Laura T 1 9

Soup, Salad, Entree, Veg Paradigm

Example: Claire, Emily, and Laura are at a store that sells works by local artists. The store has 6 oil paintings, 5 water color sketches, and 9 textile pieces for sale. All of the items are dierent from one another. Clair purchases an oil painting, Emily purchases a water color sketch, and Laura purchases a textile piece. O = 6 oil paintings W = 5 water colors T = 9 textiles and S = Claire O 1 6 Emily W 1 5 Laura T 1 9

Soup, Salad, Entree, Veg Paradigm Answer: n[S ] = 6 5 9 = 270

Example: The Dean of Students has 4 Associate Deans, 5 Assistant Deans, and 9 program specialists on his sta. To check for alcohol violations this weekend, the Dean is going to select one of these sta members to visit the Sigma Lambda house at 11:00 pm on Saturday, one of these sta members to visit the Delta Phi house at 11:00 pm on Saturday, and one of these sta members to visit the Phi Sigma house at 11:00 pm on Saturday. D = 18 sta members and S = D 1 18 D 1 17 D 1 16

Example: The Dean of Students has 4 Associate Deans, 5 Assistant Deans, and 9 program specialists on his sta. To check for alcohol violations this weekend, the Dean is going to select one of these sta members to visit the Sigma Lambda house at 11:00 pm on Saturday, one of these sta members to visit the Delta Phi house at 11:00 pm on Saturday, and one of these sta members to visit the Phi Sigma house at 11:00 pm on Saturday. D = 18 sta members and S = D 1 18 Casting a Play Paradigm D 1 17 D 1 16

Example: The Dean of Students has 4 Associate Deans, 5 Assistant Deans, and 9 program specialists on his sta. To check for alcohol violations this weekend, the Dean is going to select one of these sta members to visit the Sigma Lambda house at 11:00 pm on Saturday, one of these sta members to visit the Delta Phi house at 11:00 pm on Saturday, and one of these sta members to visit the Phi Sigma house at 11:00 pm on Saturday. D = 18 sta members and S = D 1 18 Casting a Play Paradigm Answer: n[S ] = P(18, 3) = 18 17 16 = 4896 D 1 17 D 1 16

Example: Laura, Susan, and Chris have just enrolled in a university in Chicago. Each one must open a checking/savings account at a local institution. They may choose from 8 (regular) banks, 4 savings banks, and 4 credit unions. D = 16 banks and S = Laura B 1 16 Susan B 1 16 Chris B 1 16

Example: Laura, Susan, and Chris have just enrolled in a university in Chicago. Each one must open a checking/savings account at a local institution. They may choose from 8 (regular) banks, 4 savings banks, and 4 credit unions. D = 16 banks and S = Laura B 1 16 Susan B 1 16 Chris B 1 16

Coee, Tea, Coke, Sprite Paradigm

Example: Laura, Susan, and Chris have just enrolled in a university in Chicago. Each one must open a checking/savings account at a local institution. They may choose from 8 (regular) banks, 4 savings banks, and 4 credit unions. D = 16 banks and S = Laura B 1 16 Susan B 1 16 Chris B 1 16

Coee, Tea, Coke, Sprite Paradigm Answer: n[S ] = 163 = 4096

Example: Alyssa, Brandy,and Carl are having dinner at a Chinese restaurant. The menu lists 9 beef dishes, 6 pork dishes, and 6 chicken dishes. Alyssa, Brandy, and Carl have decided that they will share 3 dishes. They jointly select one beef dish, one pork dish, and one chicken dish. B = 9 beef dishes P = 6 pork dishes C = 6 chicken dishes and S = B 1 9 P 1 6 C 1 6

Example: Alyssa, Brandy,and Carl are having dinner at a Chinese restaurant. The menu lists 9 beef dishes, 6 pork dishes, and 6 chicken dishes. Alyssa, Brandy, and Carl have decided that they will share 3 dishes. They jointly select one beef dish, one pork dish, and one chicken dish. B = 9 beef dishes P = 6 pork dishes C = 6 chicken dishes and S = B 1 9 P 1 6 C 1 6

Soup, Salad, Entree, Veg Paradigm

Example: Alyssa, Brandy,and Carl are having dinner at a Chinese restaurant. The menu lists 9 beef dishes, 6 pork dishes, and 6 chicken dishes. Alyssa, Brandy, and Carl have decided that they will share 3 dishes. They jointly select one beef dish, one pork dish, and one chicken dish. B = 9 beef dishes P = 6 pork dishes C = 6 chicken dishes and S = B 1 9 P 1 6 C 1 6

Soup, Salad, Entree, Veg Paradigm Answer: n[S ] = 9 6 6 = 324

Example: Allyson, Beth, and Chelsea, who are sharing an apartment, have gone to a bookstore to purchase 3 cookbooks. The store had 5 dierent general cookbooks, 3 dierent Italian cookbooks, and 6 dierent French cookbooks. Together they select 3 of these cookbooks. D = 14 cookbooks and S = D 3 C(14, 3).

Example: Allyson, Beth, and Chelsea, who are sharing an apartment, have gone to a bookstore to purchase 3 cookbooks. The store had 5 dierent general cookbooks, 3 dierent Italian cookbooks, and 6 dierent French cookbooks. Together they select 3 of these cookbooks. D = 14 cookbooks and S = D 3 C(14, 3). City Council Committee Paradigm

Example: Allyson, Beth, and Chelsea, who are sharing an apartment, have gone to a bookstore to purchase 3 cookbooks. The store had 5 dierent general cookbooks, 3 dierent Italian cookbooks, and 6 dierent French cookbooks. Together they select 3 of these cookbooks. D = 14 cookbooks and S = D 3 C(14, 3). City Council Committee Paradigm Answer: n[S ] = C(14, 3) =
141312 321

= 364

Example: A Republican member to the Zoning

city council has 5 Democratic members and 3 members. The mayor is to assign 1 Democratic council the Police Com. and 1 Republican council member to Com. How many outcomes are possible?

Example: A Republican member to the Zoning

city council has 5 Democratic members and 3 members. The mayor is to assign 1 Democratic council the Police Com. and 1 Republican council member to Com. How many outcomes are possible?

Analysis: 1. Domain: 8 council members 2. Qualities: Dem, Rep 3. Roles: Police, Zoning 4. Replacement: no Soup, Salad, Entree, Veg Paradigm

Example: A Republican member to the Zoning

city council has 5 Democratic members and 3 members. The mayor is to assign 1 Democratic council the Police Com. and 1 Republican council member to Com. How many outcomes are possible?

Analysis: 1. Domain: 8 council members 2. Qualities: Dem, Rep 3. Roles: Police, Zoning 4. Replacement: no Soup, Salad, Entree, Veg Paradigm D = 5 Dems and S = Police D 1 5 Answer: n[S ] = 5 3 = 15 Zoning R 1 3. R = 3 Reps

Example: A city council has 5 Democratic and 3 Republican members. The mayor is to assign 1 council member to the Police Commission and 1 council member to the Zoning Commission. (The same person could be assigned to both commissions.) How many outcomes are possible?

Example: A city council has 5 Democratic and 3 Republican members. The mayor is to assign 1 council member to the Police Commission and 1 council member to the Zoning Commission. (The same person could be assigned to both commissions.) How many outcomes are possible? Analysis: 1. Domain: 8 council members 2. Qualities: none that matter 3. Roles: Police, Zoning 4. Replacement: yes Coee, Tea, Coke, Sprite Paradigm

Example: A city council has 5 Democratic and 3 Republican members. The mayor is to assign 1 council member to the Police Commission and 1 council member to the Zoning Commission. (The same person could be assigned to both commissions.) How many outcomes are possible? Analysis: 1. Domain: 8 council members 2. Qualities: none that matter 3. Roles: Police, Zoning 4. Replacement: yes Coee, Tea, Coke, Sprite Paradigm Let C = 8 council members, then S is Police C 1 8 Zoning C 1 8 = 64.

Example: A city council has 5 Democratic members and 3 Republican members. The mayor is to assign 1 council member to the Police Commission and 1 council member to the Zoning Commission. No one can be assigned to two commissions. How many outcomes are possible?

Example: A city council has 5 Democratic members and 3 Republican members. The mayor is to assign 1 council member to the Police Commission and 1 council member to the Zoning Commission. No one can be assigned to two commissions. How many outcomes are possible? Analysis: 1. Domain: 8 council members 2. Qualities: none that matter 3. Roles: Police, Zoning 4. Replacement: no Casting a Play Paradigm

Example: A city council has 5 Democratic members and 3 Republican members. The mayor is to assign 1 council member to the Police Commission and 1 council member to the Zoning Commission. No one can be assigned to two commissions. How many outcomes are possible? Analysis: 1. Domain: 8 council members 2. Qualities: none that matter 3. Roles: Police, Zoning 4. Replacement: no Casting a Play Paradigm Let C = 8 council members, then S is Police C 1 8 Zoning C 1 7 = 56.

Example: A city council has 5 Democratic members and 3 Republican members. The mayor is to assign 2 council members to the Police Commission. How many outcomes are possible?

Example: A city council has 5 Democratic members and 3 Republican members. The mayor is to assign 2 council members to the Police Commission. How many outcomes are possible? Analysis: 1. Domain: 8 council members 2. Qualities: none that matter 3. Roles: none that matter 4. Replacement: no City Council Committee Paradigm

Example: A city council has 5 Democratic members and 3 Republican members. The mayor is to assign 2 council members to the Police Commission. How many outcomes are possible? Analysis: 1. Domain: 8 council members 2. Qualities: none that matter 3. Roles: none that matter 4. Replacement: no City Council Committee Paradigm Let C = 8 council members, then S is Police C 2 C(8, 2) = 28.

Example: A city council has 5 Democratic members and 3 Republican members. In how many dierent ways can the mayor appoint a committee consisting of 3 Democratic council members and 2 Republican council members?

Example: A city council has 5 Democratic members and 3 Republican members. In how many dierent ways can the mayor appoint a committee consisting of 3 Democratic council members and 2 Republican council members? Analysis: 1. Domain: 8 council members 2. Qualities: Dem, Rep 3. Roles: Dem, Rep 4. Replacement: no

Example: A city council has 5 Democratic members and 3 Republican members. In how many dierent ways can the mayor appoint a committee consisting of 3 Democratic council members and 2 Republican council members? Analysis: 1. Domain: 8 council members 2. Qualities: Dem, Rep 3. Roles: Dem, Rep 4. Replacement: no Doesnt exactly t any paradigm

Example: A city council has 5 Democratic members and 3 Republican members. In how many dierent ways can the mayor appoint a committee consisting of 3 Democratic council members and 2 Republican council members? Analysis: 1. Domain: 8 council members 2. Qualities: Dem, Rep 3. Roles: Dem, Rep 4. Replacement: no Doesnt exactly t any paradigm D = 5 Dems and S is Committee D R 3 C(5, 3) 2 C(3, 2) = 30. R = 3 Reps

Example: A small fraternity consists of 7 seniors, 11 juniors, and 9 sophomores. In how many dierent ways could a steering committee consisting of 3 seniors, 2 juniors, and 1 sophomore be selected.

Example: A small fraternity consists of 7 seniors, 11 juniors, and 9 sophomores. In how many dierent ways could a steering committee consisting of 3 seniors, 2 juniors, and 1 sophomore be selected. Analysis: 1. Domain: 27 frat bros 2. Qualities: Senior, Junior, Sophomore 3. Roles: Senior, Junior, Sophomore 4. Replacement: no

Example: A small fraternity consists of 7 seniors, 11 juniors, and 9 sophomores. In how many dierent ways could a steering committee consisting of 3 seniors, 2 juniors, and 1 sophomore be selected. Analysis: 1. Domain: 27 frat bros 2. Qualities: Senior, Junior, Sophomore 3. Roles: Senior, Junior, Sophomore 4. Replacement: no N = 7 Seniors J = 11 Juniors M = 9 Sophomores and S is N 3 C(7, 3) Committee J 2 C(11, 2) M 1 C(9, 1) = 17325.

Example: The Board of Directors of a company has 12 members. If 3 are to be assigned to the budget committee, 2 are to be assigned to the personnel committee, and 4 are to be assigned to the government relations committee, and no one can serve on more than one committee, how many dierent outcomes are possible?

Example: The Board of Directors of a company has 12 members. If 3 are to be assigned to the budget committee, 2 are to be assigned to the personnel committee, and 4 are to be assigned to the government relations committee, and no one can serve on more than one committee, how many dierent outcomes are possible? Analysis: 1. Domain: 12 Directors 2. Qualities: none that matter 3. Roles: Budget, Personnel, GovRel 4. Replacement: no

Example: The Board of Directors of a company has 12 members. If 3 are to be assigned to the budget committee, 2 are to be assigned to the personnel committee, and 4 are to be assigned to the government relations committee, and no one can serve on more than one committee, how many dierent outcomes are possible? Analysis: 1. Domain: 12 Directors 2. Qualities: none that matter 3. Roles: Budget, Personnel, GovRel 4. Replacement: no Let D = 12 Directors, then S is Budget D 3 C(12, 3) Personnel D 2 C(9, 2) GovRel D 4 C(7, 4) = 277200.

Example: The Board of Directors of a company has 12 members. If 3 are to be assigned to the budget committee, 2 are to be assigned to the personnel committee, and 4 are to be assigned to the government relations committee, and one can serve on more than one committee, how many dierent outcomes are possible?

Example: The Board of Directors of a company has 12 members. If 3 are to be assigned to the budget committee, 2 are to be assigned to the personnel committee, and 4 are to be assigned to the government relations committee, and one can serve on more than one committee, how many dierent outcomes are possible? Analysis: 1. Domain: 12 Directors 2. Qualities: none that matter 3. Roles: Budget, Personnel, GovRel 4. Replacement: within committee: no; between committees: yes

Example: The Board of Directors of a company has 12 members. If 3 are to be assigned to the budget committee, 2 are to be assigned to the personnel committee, and 4 are to be assigned to the government relations committee, and one can serve on more than one committee, how many dierent outcomes are possible? Analysis: 1. Domain: 12 Directors 2. Qualities: none that matter 3. Roles: Budget, Personnel, GovRel 4. Replacement: within committee: no; between committees: yes Let D = 12 Directors, then S is Budget D 3 C(12, 3) Personnel D 2 C(12, 2) GovRel D 4 C(12, 4) = 7187400.

Example: (Anagrams) How many words can be formed using the letters in the word card?

Example: (Anagrams) How many words can be formed using the letters in the word card? Analysis: 1. Domain: 4 letters (c, a, r, d) 2. Qualities: none that matter 3. Roles: Position (14) 4. Replacement: no

Example: (Anagrams) How many words can be formed using the letters in the word card? Analysis: 1. Domain: 4 letters (c, a, r, d) 2. Qualities: none that matter 3. Roles: Position (14) 4. Replacement: no D = 4 Letters and S is 1st D 1 4 2nd D 1 3 3rd D 1 2 4th D 1 1 = 24

Alternate Solution
Example: (Anagrams) How many words can be formed using the letters in the word card?

Alternate Solution
Example: (Anagrams) How many words can be formed using the letters in the word card? Analysis: 1. Domain: 4 Positions 2. Qualities: none that matter 3. Roles: c, a, r, d 4. Replacement: no

Alternate Solution
Example: (Anagrams) How many words can be formed using the letters in the word card? Analysis: 1. Domain: 4 Positions 2. Qualities: none that matter 3. Roles: c, a, r, d 4. Replacement: no D = 4 Positions and S is C D 1 4 A D 1 3 R D 1 2 D D 1 1 = 24

Example: How many words can be formed using the letters in the word cool?

Example: How many words can be formed using the letters in the word cool? Analysis: 1. Domain: 4 Positions 2. Qualities: none that matter 3. Roles: c, o, l 4. Replacement: no

Example: How many words can be formed using the letters in the word cool? Analysis: 1. Domain: 4 Positions 2. Qualities: none that matter 3. Roles: c, o, l 4. Replacement: no D = 4 Positions and S is C D 1 C(4, 1) O D 2 C(3, 2) L D 1 C(1, 1) = 12

Example: How many words can be formed using the letters in the word bananas?

Example: How many words can be formed using the letters in the word bananas? Analysis: 1. Domain: 7 Positions 2. Qualities: none that matter 3. Roles: b, a, n, s 4. Replacement: no

Example: How many words can be formed using the letters in the word bananas? Analysis: 1. Domain: 7 Positions 2. Qualities: none that matter 3. Roles: b, a, n, s 4. Replacement: no D = 7 Positions and S is B D 1 C(7, 1) A D 3 C(6, 3) N D 2 C(3, 2) S D 1 C(1, 1) = 420

Compound Problems

Some problems are like Russian dolls: problems inside problems. Youll need to use multiple strategies on a single problem.

Example: The city council has 5 Democratic and 3 Republican members. The mayor is to appoint 3 or 4 of the council members to a committee. How many dierent outcomes are possible?

Example: The city council has 5 Democratic and 3 Republican members. The mayor is to appoint 3 or 4 of the council members to a committee. How many dierent outcomes are possible? Analysis: 1. Domain: 8 council members 2. Qualities: none that matter 3. Roles: none 4. Replacement: no

Example: The city council has 5 Democratic and 3 Republican members. The mayor is to appoint 3 or 4 of the council members to a committee. How many dierent outcomes are possible? Analysis: 1. Domain: 8 council members 2. Qualities: none that matter 3. Roles: none 4. Replacement: no Let D = 8 council members, then S is D 3 C(8, 3) 4 C(8, 4) 56 70 126 .

Example: The city council has 5 Democratic and 3 Republican members. The mayor is to appoint 3 of the council members to a bipartisan committee. How many dierent outcomes are possible?

Example: The city council has 5 Democratic and 3 Republican members. The mayor is to appoint 3 of the council members to a bipartisan committee. How many dierent outcomes are possible? Analysis: 1. Domain: 8 council members 2. Qualities: Dem, Rep 3. Roles: none that matter 4. Replacement: no

Example: The city council has 5 Democratic and 3 Republican members. The mayor is to appoint 3 of the council members to a bipartisan committee. How many dierent outcomes are possible? Analysis: 1. Domain: 8 council members 2. Qualities: Dem, Rep 3. Roles: none that matter 4. Replacement: no D = 5 Democrats and S is D 1 C(5, 1) 2 C(5, 2) R 2 C(3, 2) 1 C(3, 1) 15 30 45 R = 3 Republicans

Example: The city council has 5 Democratic and 3 Republican members. The mayor is to appoint 3 of the council members to a committee with at most 1 Democrat. How many dierent outcomes are possible?

Example: The city council has 5 Democratic and 3 Republican members. The mayor is to appoint 3 of the council members to a committee with at most 1 Democrat. How many dierent outcomes are possible? Analysis: 1. Domain: 8 council members 2. Qualities: Dem, Rep 3. Roles: none that matter 4. Replacement: no

Example: The city council has 5 Democratic and 3 Republican members. The mayor is to appoint 3 of the council members to a committee with at most 1 Democrat. How many dierent outcomes are possible? Analysis: 1. Domain: 8 council members 2. Qualities: Dem, Rep 3. Roles: none that matter 4. Replacement: no D = 5 Democrats R = 3 Republicans and then S is D 0 C(5, 0) 1 C(5, 1) R 3 C(3, 3) 2 C(3, 2) 1 15 16

Example: The city council has 5 Democratic and 3 Republican members. The mayor is to appoint 3 of the council members to a committee with at least 1 Democrat. How many dierent outcomes are possible?

Example: The city council has 5 Democratic and 3 Republican members. The mayor is to appoint 3 of the council members to a committee with at least 1 Democrat. How many dierent outcomes are possible? Analysis: House Subcommittee Paradigm

Example: The city council has 5 Democratic and 3 Republican members. The mayor is to appoint 3 of the council members to a committee with at least 1 Democrat. How many dierent outcomes are possible? Analysis: House Subcommittee Paradigm 5 Dems 1 C(5, 1) 2 C(5, 2) 3 C(5, 3) 3 Reps 2 C(3, 2) 1 C(3, 1) 0 C(3, 0) 15 30 10 55

Alternate Method: by Complement


Example: The city council has 5 Democratic and 3 Republican members. The mayor is to appoint 3 of the council members to a committee with at least 1 Democrat. How many dierent outcomes are possible? Analysis:

Alternate Method: by Complement


Example: The city council has 5 Democratic and 3 Republican members. The mayor is to appoint 3 of the council members to a committee with at least 1 Democrat. How many dierent outcomes are possible? Analysis:

ST SR SR

= all outcomes with no restrictions = outcomes with at least 1 Dem = outcomes with no Dems

Alternate Method: by Complement


Example: The city council has 5 Democratic and 3 Republican members. The mayor is to appoint 3 of the council members to a committee with at least 1 Democrat. How many dierent outcomes are possible? Analysis:

ST SR SR n[ST ]

= = = =

all outcomes with no restrictions outcomes with at least 1 Dem outcomes with no Dems n[SR ] + n[SR ]

Alternate Method: by Complement


Example: The city council has 5 Democratic and 3 Republican members. The mayor is to appoint 3 of the council members to a committee with at least 1 Democrat. How many dierent outcomes are possible? Analysis:

ST SR SR n[ST ] n[ST ] n[SR ]

= = = = =

all outcomes with no restrictions outcomes with at least 1 Dem outcomes with no Dems n[SR ] + n[SR ] n[SR ]

Alternate Method: by Complement


Example: The city council has 5 Democratic and 3 Republican members. The mayor is to appoint 3 of the council members to a committee with at least 1 Democrat. How many dierent outcomes are possible? Analysis:

ST SR SR n[ST ] n[ST ] n[SR ] C(8, 3)

= = = = =

all outcomes with no restrictions outcomes with at least 1 Dem outcomes with no Dems n[SR ] + n[SR ] n[SR ]

Alternate Method: by Complement


Example: The city council has 5 Democratic and 3 Republican members. The mayor is to appoint 3 of the council members to a committee with at least 1 Democrat. How many dierent outcomes are possible? Analysis:

ST SR SR n[ST ] n[ST ] n[SR ] C(8, 3) C(3, 3) 56 1

= = = = = = =

all outcomes with no restrictions outcomes with at least 1 Dem outcomes with no Dems n[SR ] + n[SR ] n[SR ] 55

The Complement Principle


The symbol denotes the complement of a set: A =S A = { x | x S & x A }.

Then: In general, n[A] = n[S ] n[A ].

The Complement Principle


The symbol denotes the complement of a set: A =S A = { x | x S & x A }.

Then: In general, n[A] = n[S ] n[A ].

The Complement Principle (CP) is worth considering if the problem contains phrases like at least 1 or at most 5.

Example: The House committee on Education has 8 Democratic and 9 Republican members. They want to form a subcommittee of 7 to investigate corruption in Finite Math, and the subcommittee must have at least 1 Democrat. How many dierent outcomes are possible? Analysis: House Subcommittee Paradigm

Example: The House committee on Education has 8 Democratic and 9 Republican members. They want to form a subcommittee of 7 to investigate corruption in Finite Math, and the subcommittee must have at least 1 Democrat. How many dierent outcomes are possible? Analysis: House Subcommittee Paradigm S E E = = = = all outcomes with no restrictions outcomes with at least 1 Dem outcomes with no Dems outcomes with all Reps

Example: The House committee on Education has 8 Democratic and 9 Republican members. They want to form a subcommittee of 7 to investigate corruption in Finite Math, and the subcommittee must have at least 1 Democrat. How many dierent outcomes are possible? Analysis: House Subcommittee Paradigm S E E = = = = all outcomes with no restrictions outcomes with at least 1 Dem outcomes with no Dems outcomes with all Reps

Then by the CP, n[E ] = n[S ] n[E ]

Example: The House committee on Education has 8 Democratic and 9 Republican members. They want to form a subcommittee of 7 to investigate corruption in Finite Math, and the subcommittee must have at least 1 Democrat. How many dierent outcomes are possible? Analysis: House Subcommittee Paradigm S E E = = = = all outcomes with no restrictions outcomes with at least 1 Dem outcomes with no Dems outcomes with all Reps

Then by the CP, n[E ] = n[S ] n[E ] = C(17, 7) C(9, 7) = 19412.

Example: A class has 5 sophomores, 7 juniors, and 6 seniors. If 3 students are selected to give a report, how many dierent outcomes are possible in which the students are not all in the same year? Analysis: S E E n[E ] = = = = all outcomes with no restrictions outcomes with students not all in same year outcomes with students all in same year n[S ] n[E ] (CP)

Example: A class has 5 sophomores, 7 juniors, and 6 seniors. If 3 students are selected to give a report, how many dierent outcomes are possible in which the students are not all in the same year? Analysis: S E E n[E ] = = = = all outcomes with no restrictions outcomes with students not all in same year outcomes with students all in same year n[S ] n[E ] (CP)

And we easily get n[S ] = C(18, 3) = 816. But what is n[E ]?

Subproblem: A class has 5 sophomores, 7 juniors, and 6 seniors. If 3 students are selected to give a report, how many dierent outcomes are possible in which all students are in the same year?

Subproblem: A class has 5 sophomores, 7 juniors, and 6 seniors. If 3 students are selected to give a report, how many dierent outcomes are possible in which all students are in the same year? Analysis:

Subproblem: A class has 5 sophomores, 7 juniors, and 6 seniors. If 3 students are selected to give a report, how many dierent outcomes are possible in which all students are in the same year? Analysis: Year 7 Jun 0 1 3 C(7, 3) = 35 0 1

5 Soph 3 C(5, 3) = 10 0 1 0 1

6 Sen 0 1 0 1 3 C(6, 3) = 20

size 10 35 20 65

Alternatives

Example: (cont.) A class has 5 sophomores, 7 juniors, and 6 seniors. If 3 students are selected to give a report, how many dierent outcomes are possible in which the students are not all in the same year? Analysis: S E E n[E ] = = = = all outcomes with no restrictions outcomes with students not all in same year outcomes with students all in same year n[S ] n[E ] (CP)

Example: (cont.) A class has 5 sophomores, 7 juniors, and 6 seniors. If 3 students are selected to give a report, how many dierent outcomes are possible in which the students are not all in the same year? Analysis: S E E n[E ] = = = = all outcomes with no restrictions outcomes with students not all in same year outcomes with students all in same year n[S ] n[E ] (CP)

n[S ] = 816 n[E ] = 65 n[E ] = 816 65 = 751

Example: The math chairperson has a meeting at 11 to decide on the recipient of this years Outstanding Finite Student award. He also has time for meetings at 10, 12, and 2. He has three other tasks to do today: meet with the winner of the award, draft a budget proposal, and nalize the summer teaching schedule. How many ways are there to schedule his day?

Example: The math chairperson has a meeting at 11 to decide on the recipient of this years Outstanding Finite Student award. He also has time for meetings at 10, 12, and 2. He has three other tasks to do today: meet with the winner of the award, draft a budget proposal, and nalize the summer teaching schedule. How many ways are there to schedule his day? Analysis: 1. Objects: 3 tasks 2. Qualities: award and nonaward 3. Roles: 10, 12, 2 4. Replacement: no

Example: The math chairperson has a meeting at 11 to decide on the recipient of this years Outstanding Finite Student award. He also has time for meetings at 10, 12, and 2. He has three other tasks to do today: meet with the winner of the award, draft a budget proposal, and nalize the summer teaching schedule. How many ways are there to schedule his day? Analysis: 1. Objects: 3 tasks 2. Qualities: award and nonaward 3. Roles: 10, 12, 2 4. Replacement: no

Analysis: 1. Objects: 3 tasks 2. Qualities: award and nonaward 3. Roles: 10, 12, 2 4. Replacement: no

Analysis: 1. Objects: 3 tasks 2. Qualities: award and nonaward 3. Roles: 10, 12, 2 4. Replacement: no D = 3 tasks A = meeting with award winner N = 2 other meetings S: 10 D 1 3 12 D 1 2 2 D 1 1 6

Analysis: 1. Objects: 3 tasks 2. Qualities: award and nonaward 3. Roles: 10, 12, 2 4. Replacement: no D = 3 tasks A = meeting with award winner N = 2 other meetings E = feasible schedules: 10 N 1 2 12 D 1 2 2 D 1 1 4

Example: A department chairperson has 4 tasks to do today: select an award recipient; have a meeting with this recipient; write a letter to the dean; and prepare the departments budget request for next year. If he schedules these tasks one after another, how many dierent schedules are possible. (Note that he cannot meet with the committee until after he has selected its members.)

Example: A department chairperson has 4 tasks to do today: select an award recipient; have a meeting with this recipient; write a letter to the dean; and prepare the departments budget request for next year. If he schedules these tasks one after another, how many dierent schedules are possible. (Note that he cannot meet with the committee until after he has selected its members.) Analysis: 1. Objects: 4 tasks 2. Qualities: award and nonaward 3. Roles: 4 time slots 4. Replacement: no

Example: A department chairperson has 4 tasks to do today: select an award recipient; have a meeting with this recipient; write a letter to the dean; and prepare the departments budget request for next year. If he schedules these tasks one after another, how many dierent schedules are possible. (Note that he cannot meet with the committee until after he has selected its members.) Analysis: 1. Objects: 4 tasks 2. Qualities: award and nonaward 3. Roles: 4 time slots 4. Replacement: no We can break this problem down into alternatives.

C = meeting to choose award winner N = 3 other meetings, including one with awardee and E = feasible schedules is 1st C 1 2nd N 3 3rd N 2 4th N 1 6

C = meeting to choose award winner N = 3 other meetings, including one with awardee and E = feasible schedules is 1st C 1 N 2 N 2 N 2 2nd N 3 C 1 N 1 N 1 3rd N 2 N 2 C 1 N 0 4th N 1 N 1 N 1 C 1 6 4 2 0 12 Since we are always picking just one task for each slot I have omitted all the 1s.

Example: The Personnel Manager at a company must schedule 4 job interviews and 3 dierent committee meetings for tomorrow. If she decides to schedule the job interviews one after another and to schedule the committee meetings one after another, how many dierent schedules are possible?

Example: The Personnel Manager at a company must schedule 4 job interviews and 3 dierent committee meetings for tomorrow. If she decides to schedule the job interviews one after another and to schedule the committee meetings one after another, how many dierent schedules are possible? Analysis: 1. Objects: 7 tasks 2. Qualities: interviews, meetings 3. Roles: 2 time blocks 4. Replacement: no

Let I = 4 job interviews M = 3 committee meetings then E = desired schedules is Block 1 I 4 P(4, 4) M 3 P(3, 3) Block 2 M 3 P(3, 3) I 4 P(4, 4)

144

144 288

Example: A student has 4 novels by Thomas Hardy, 2 novels by George Eliot, and 5 novels by Charles Dickens. In how many dierent ways could she arrange these books on a book shelf (from left to right) if each authors books are kept together?

Example: A student has 4 novels by Thomas Hardy, 2 novels by George Eliot, and 5 novels by Charles Dickens. In how many dierent ways could she arrange these books on a book shelf (from left to right) if each authors books are kept together? Analysis: 1. Objects: 11 books 2. Qualities: Hardy, Eliot, Dickens 3. Roles: 3 Blocks 4. Replacement: no

Example: A student has 4 novels by Thomas Hardy, 2 novels by George Eliot, and 5 novels by Charles Dickens. In how many dierent ways could she arrange these books on a book shelf (from left to right) if each authors books are kept together? Analysis: 1. Objects: 11 books 2. Qualities: Hardy, Eliot, Dickens 3. Roles: 3 Blocks 4. Replacement: no H = 4 Hardy novels E = 2 Eliot novels D = 5 Dickens novels

Block 1 H P(4, 4) H P(4, 4) E P(2, 2) E P(2, 2) D P(5, 5) D P(5, 5)

Block 2 E P(2, 2) D P(5, 5) H P(4, 4) D P(5, 5) H P(4, 4) E P(2, 2)

Block 3 D P(5, 5) E P(2, 2) D P(5, 5) H P(4, 4) E P(2, 2) H P(4, 4) 5760 5760 5760 5760 5760 5760 34560

Alternate Solution

Note that in each row the qualities are anagrams of HED .

Alternate Solution

Note that in each row the qualities are anagrams of HED . So the number of rows must be C(3, 1)C(2, 1)C(1, 1) = 6.

Alternate Solution

Note that in each row the qualities are anagrams of HED . So the number of rows must be C(3, 1)C(2, 1)C(1, 1) = 6. Since each row has the same size, n[E ] = 6 5760 = 34560.

Alternate Solution

Note that in each row the qualities are anagrams of HED . So the number of rows must be C(3, 1)C(2, 1)C(1, 1) = 6. Since each row has the same size, n[E ] = 6 5760 = 34560.

This is an example of the Same Size Principle (SSP).

Block 1 H P(4, 4) H P(4, 4) E P(2, 2) E P(2, 2) D P(5, 5) D P(5, 5)

Block 2 E P(2, 2) D P(5, 5) H P(4, 4) D P(5, 5) H P(4, 4) E P(2, 2)

Block 3 D P(5, 5) E P(2, 2) D P(5, 5) H P(4, 4) E P(2, 2) H P(4, 4) 5760 5760 5760 5760 5760 5760 6 5760 = 34560

Example: In how many ways could Anne, Bill, Carol, Dan, and Ellen be arranged in a line for a picture if Anne and Dan insist on standing next to one another?

Example: In how many ways could Anne, Bill, Carol, Dan, and Ellen be arranged in a line for a picture if Anne and Dan insist on standing next to one another? Analysis: 1. Objects: 5 people 2. Qualities: Ann, Dan, Others 3. Roles: 5 positions 4. Replacement: no A = Ann D = Dan P = 3 people besides Ann and Dan

Then E = desired orders is 1st A 1 D 1 P 3 P P P P P 2nd D 1 A 1 A 1 D P P P P 3rd P 3 P 3 D 1 A A D P P 4th P 2 P 2 P 2 P D A A D 5th P 1 P 1 P 1 P P P D A 8 6 = 48 6 6 6

Alternative Method
There are 4 blocks of people: A D and B , C , and E .

Alternative Method
There are 4 blocks of people: A D and B , C , and E . 1st A D 2nd B 3rd C 4th E

P(4, 4) = 24

Alternative Method
There are 4 blocks of people: A D and B , C , and E . 1st A D 2 2nd B 1 3rd C 1 4th E 1 2

P(4, 4) = 24

Alternative Method
There are 4 blocks of people: A D and B , C , and E . 1st A D 2 E 1 2nd B 1 A D 2 3rd C 1 C 1 4th E 1 B 1 2 2

P(4, 4) = 24

Alternative Method
There are 4 blocks of people: A D and B , C , and E . 1st A D 2 E 1 . . . 2nd B 1 A D 2 . . . 3rd C 1 C 1 . . . 4th E 1 B 1 . . . 2 2 . . . 24 2 = 48

P(4, 4) = 24

Example: A committee has 5 freshmen, 5 sophomores, and 5 juniors. In how many dierent ways could a panel consisting of 2 people from each of 2 classes be selected?

Example: A committee has 5 freshmen, 5 sophomores, and 5 juniors. In how many dierent ways could a panel consisting of 2 people from each of 2 classes be selected? 1. 2. 3. 4. Objects: 15 students Qualities: Freshmen, Sophomores, Juniors Roles: none that matter Replacement: no Freshmen 2 C(5, 2) 2 C(5, 2) 0 C(5, 0) Sophomores 2 C(5, 2) 0 C(5, 0) 2 C(5, 2) Juniors 0 C(5, 0) 2 C(5, 2) 2 C(5, 2) 100 100 100 300

Example: A committee has 5 freshmen, 5 sophomores, and 5 juniors. In how many dierent ways could a panel consisting of 2 people from each of 2 classes be selected? 1. 2. 3. 4. Objects: 15 students Qualities: Freshmen, Sophomores, Juniors Roles: none that matter Replacement: no Freshmen 2 C(5, 2) 2 C(5, 2) 0 C(5, 0) Sophomores 2 C(5, 2) 0 C(5, 0) 2 C(5, 2) Juniors 0 C(5, 0) 2 C(5, 2) 2 C(5, 2) 100 100 100 300 Note the three cases are all anagrams of 2, 2, 0.

Example: A committee has 5 freshmen, 5 sophomores, 5 juniors, and 5 seniors. In how many dierent ways could a panel be chosen if it must consist of 2 people from one class and 1 person each from two other classes?

Example: A committee has 5 freshmen, 5 sophomores, 5 juniors, and 5 seniors. In how many dierent ways could a panel be chosen if it must consist of 2 people from one class and 1 person each from two other classes? Analysis: 1. Objects: 20 students 2. Qualities: Freshmen, Sophomores, Juniors, Seniors 3. Roles: none that matter 4. Replacement: no

The desired panels can be described as follows. Freshmen 2 C(5, 2) 1 C(5, 1) . . . Sophomores 1 C(5, 1) 1 C(5, 1) . . . Juniors 1 C(5, 1) 0 C(5, 0) . . . Seniors 0 C(5, 0) 2 C(5, 2) . . . 250 250 . . . 3000

12

The desired panels can be described as follows. Freshmen 2 C(5, 2) 1 C(5, 1) . . . Sophomores 1 C(5, 1) 1 C(5, 1) . . . Juniors 1 C(5, 1) 0 C(5, 0) . . . Seniors 0 C(5, 0) 2 C(5, 2) . . . 250 250 . . . 3000 Why 12? # anagrams of 2110 = C(4, 1)C(3, 2)C(1, 1) = 12

12

Example: How many dierent hands consisting of 5 cards from a regular deck of 52 cards (i.e., four suits spades, hearts, diamonds, and clubs with thirteen cards each and no jokers) have 3 cards from one suit and 2 cards from one other suit?

Example: How many dierent hands consisting of 5 cards from a regular deck of 52 cards (i.e., four suits spades, hearts, diamonds, and clubs with thirteen cards each and no jokers) have 3 cards from one suit and 2 cards from one other suit? Analysis: 1. Objects: 52 cards 2. Qualities: 13 , 13 , 13 , 13 3. Roles: none that matter 4. Replacement: no

The desired hands can be described as follows. 3 C(13, 3) 2 C(13, 2) . . . 2 C(13, 2) 0 C(13, 0) . . . 0 C(13, 0) 3 C(13, 3) . . . 0 C(13, 0) 0 C(13, 0) . . . 22308 22308 . . . 267696 12 = # anagrams of 3200 = C(4, 1)C(3, 1)C(2, 2)

12

Example: A small retail chain has vacancies for 4 store managers all in dierent cities. Because the positions are in dierent cities, no one could hold more than one of these positions. The company has 6 female assistant managers and 5 male assistant managers who are eligible for promotion to these store manager positions. How many outcomes are there in which 2 women and 2 men are promoted?

Example: A small retail chain has vacancies for 4 store managers all in dierent cities. Because the positions are in dierent cities, no one could hold more than one of these positions. The company has 6 female assistant managers and 5 male assistant managers who are eligible for promotion to these store manager positions. How many outcomes are there in which 2 women and 2 men are promoted? Analysis: 1. Objects: 11 managers 2. Qualities: 6 female, 5 male 3. Roles: 4 stores 4. Replacement: no

F = 6 females M = 5 males

Store 1 F 6 F 6 . . .

Store 2 F 5 M 5 . . .

Store 3 M 5 M 4 . . .

Store 4 M 4 F 5 . . . 600 600 . . . 3600

C(4, 2)C(2, 2) = 6

Example: 9 seniors and 4 juniors in English each submitted one poem for the English Departments annual poetry competition. The department will award 1st, 2nd, 3rd, 4th, and 5th prizes. In how many overall outcomes would seniors be selected for exactly 3 prizes?

Example: 9 seniors and 4 juniors in English each submitted one poem for the English Departments annual poetry competition. The department will award 1st, 2nd, 3rd, 4th, and 5th prizes. In how many overall outcomes would seniors be selected for exactly 3 prizes? Analysis: 1. Objects: 13 students 2. Qualities: 9 Seniors, 4 Juniors 3. Roles: 5 prizes 4. Replacement: no

N = 9 Seniors J = 4 Juniors

1st N 9 N 9 . . .

2nd N 8 J 4 . . .

3rd N 7 N 8 . . .

4th J 4 N 7 . . .

5th J 3 J 3 . . . 6048 6048 . . . 60480

C(5, 3) = 10

Equally Likely Outcomes


When all outcomes are equally likely, probabilities are easy to compute. Example: There are 15 people in a club, including Adrienne and Ben, who run the door prize committee. Two people are randomly chosen to receive door prizes, and it turns out to be Adrienne and Ben! What is the probability of that happening?

Equally Likely Outcomes


When all outcomes are equally likely, probabilities are easy to compute. Example: There are 15 people in a club, including Adrienne and Ben, who run the door prize committee. Two people are randomly chosen to receive door prizes, and it turns out to be Adrienne and Ben! What is the probability of that happening? Solution: For the sample space, n[S ] = C(15, 2) = 105. The event were interested in is E = A and B both get prizes and n[E ] = 1.

Equally Likely Outcomes


When all outcomes are equally likely, probabilities are easy to compute. Example: There are 15 people in a club, including Adrienne and Ben, who run the door prize committee. Two people are randomly chosen to receive door prizes, and it turns out to be Adrienne and Ben! What is the probability of that happening? Solution: For the sample space, n[S ] = C(15, 2) = 105. The event were interested in is E = A and B both get prizes and n[E ] = 1. Then Pr[E ] = 1 < 1%. 105

Formula for ELO Probability

In general, n[E ] . n[S ]

Pr[E ] =

Example: You are dealt 5 cards at random from a standard deck of cards. What is the probability the hand is a full house (3 cards of one rank and 2 of another)?

Example: You are dealt 5 cards at random from a standard deck of cards. What is the probability the hand is a full house (3 cards of one rank and 2 of another)? Analysis: S E = = all 5 card hands full houses

n[S ] = C(52, 5) = 2598960 n[E ] = ?

n[E ] = ?:

2 3 4 0 . . .

3 2 6 0 . . .

4 0 0 . . .

5 0 0 . . .

6 0 2 6 . . .

7 0 0 . . .

8 0 0 . . .

9 0 0 . . .

10 0 0 . . .

J 0 0 . . .

Q 0 0 . . .

K 0 0 . . .

A 0 24 3 4 . . . 3744 24

156

Note 4 = C(4, 3) 156 = C(13, 1)C(12, 1)C(11, 11) 6 = C(4, 2) 3744 = 156 24.

156 is the number of anagrams of 3200000000000.

Example: You are dealt 5 cards at random from a standard deck of cards. What is the probability the hand is a full house (3 cards of one rank and 2 of another)? Answer: Since n[E ] = 3744 and n[S ] = 2598960, therefore Pr[E ] = 3744 n[E ] = = .001441. n[S ] 2598960

Conditional Probability

Partial information about an outcome can change the probability. Example: If you know that someone won $60 million in the powerball last night, the probability the winner was an IU student is about 107, 160 = .01644 = 1.644%. 6, 516, 922

Conditional Probability

Partial information about an outcome can change the probability. Example: If you know that someone won $60 million in the powerball last night, the probability the winner was an IU student is about 107, 160 = .01644 = 1.644%. 6, 516, 922 But if you know the winner lived in Bloomington the probability 1 goes up to about = 33%. 3

Example: A special die has the numbers 13 in yellow and 46 in red, otherwise the die is normal and fair. You roll the die, but you dont have your glasses on, so all you tell is that the number is red. What is the probability it is even?

Example: A special die has the numbers 13 in yellow and 46 in red, otherwise the die is normal and fair. You roll the die, but you dont have your glasses on, so all you tell is that the number is red. What is the probability it is even? 1 Solution: Normally the probability of an even number is , but 2 here we have the extra information that the number is red.

Example: A special die has the numbers 13 in yellow and 46 in red, otherwise the die is normal and fair. You roll the die, but you dont have your glasses on, so all you tell is that the number is red. What is the probability it is even? 1 Solution: Normally the probability of an even number is , but 2 here we have the extra information that the number is red. 2 Of the 3 red numbers, 2 are even so the answer is . 3 Pr[ E | R ] = 2 3

Let E be the even numbers and R the red ones.

We know the number is red.

Were wondering whether it is even.

So the answer is Pr[ E | R ] =

2 = . 3

In general, Assuming ELO, Pr[ B | A ] = n[B A] . n[A]

Example: A city council has 5 Democratic members and 3 Republican members. A committee consisting of 3 members is selected at random. If the selected committee has at least 1 Democrat, then what is the probability that it has at least 1 Republican also?

Example: A city council has 5 Democratic members and 3 Republican members. A committee consisting of 3 members is selected at random. If the selected committee has at least 1 Democrat, then what is the probability that it has at least 1 Republican also? Solution: Let E be the event that the committee has at least 1 Democrat, and let F be the event that it has at least 1 Republican. Then we want Pr[ F | E ] = n[F E ] . n[E ]

Example: A city council has 5 Democratic members and 3 Republican members. A committee consisting of 3 members is selected at random. If the selected committee has at least 1 Democrat, then what is the probability that it has at least 1 Republican also? Solution: Let E be the event that the committee has at least 1 Democrat, and let F be the event that it has at least 1 Republican. Then we want Pr[ F | E ] = n[F E ] . n[E ]

So there are two subproblems: n[E ] and n[F E ].

Example: A city council has 5 Democratic members and 3 Republican members. A committee consisting of 3 members is selected at random. If the selected committee has at least 1 Democrat, then what is the probability that it has at least 1 Republican also? n[E ]: CP: E E = the committee has at least 1 Democrat = the committee has at no Democrats = the committee has 3 Republicans

Example: A city council has 5 Democratic members and 3 Republican members. A committee consisting of 3 members is selected at random. If the selected committee has at least 1 Democrat, then what is the probability that it has at least 1 Republican also? n[E ]: CP: E E = the committee has at least 1 Democrat = the committee has at no Democrats = the committee has 3 Republicans

n[E ] = C(3, 3) = 1 n[S ] = C(8, 3) = 56 n[E ] = 56 1 = 55

Example: A city council has 5 Democratic members and 3 Republican members. A committee consisting of 3 members is selected at random. If the selected committee has at least 1 Democrat, then what is the probability that it has at least 1 Republican also? n[F E ]: 5 Dem 1 C(5, 1) = 5 2 C(5, 2) = 10 3 Rep 2 C(3, 2) = 3 1 C(3, 1) = 3 size 15 30 45

Example: A city council has 5 Democratic members and 3 Republican members. A committee consisting of 3 members is selected at random. If the selected committee has at least 1 Democrat, then what is the probability that it has at least 1 Republican also? Therefore Pr[ F | E ] = 45 n[F E ] = . n[E ] 55

Example: A Republican member to member to

city council has 5 Democratic members and 3 members. The mayor appoints one randomly selected the Police Commission and another randomly selected the Zoning Commission.

1. What is the probability that the member appointed to the Zoning Commission is a Democrat? 2. What is the probability that the member appointed to the Zoning Commission is a Democrat given that the member appointed to the Police Commission is a Democrat? 3. What is the probability that the member appointed to the Zoning Commission is a Democrat given that the member appointed to the Police Commission is a Republican? Answers:

Example: A Republican member to member to

city council has 5 Democratic members and 3 members. The mayor appoints one randomly selected the Police Commission and another randomly selected the Zoning Commission.

1. What is the probability that the member appointed to the Zoning Commission is a Democrat? 2. What is the probability that the member appointed to the Zoning Commission is a Democrat given that the member appointed to the Police Commission is a Democrat? 3. What is the probability that the member appointed to the Zoning Commission is a Democrat given that the member appointed to the Police Commission is a Republican?
4 5 Answers: (1) 5 8 ; (2) 7 ; (3) 7 .

Example: A city council has 5 Democratic members and 3 Republican members. The mayor appoints one randomly selected member to the Police Commission and another randomly selected member to the Zoning Commission. What is the probability that the member appointed to the Zoning Commission is a Democrat? Solution: S E = = all possible appointees to two commissions appointee to Zoning Com is a Dem

Example: A city council has 5 Democratic members and 3 Republican members. The mayor appoints one randomly selected member to the Police Commission and another randomly selected member to the Zoning Commission. What is the probability that the member appointed to the Zoning Commission is a Democrat? Solution: S E = = all possible appointees to two commissions appointee to Zoning Com is a Dem

n[S ] = P(8, 2) = 56 n[E ] = 5 7 = 35

Example: A city council has 5 Democratic members and 3 Republican members. The mayor appoints one randomly selected member to the Police Commission and another randomly selected member to the Zoning Commission. What is the probability that the member appointed to the Zoning Commission is a Democrat? Solution: S E = = all possible appointees to two commissions appointee to Zoning Com is a Dem

n[S ] = P(8, 2) = 56 n[E ] = 5 7 = 35 Pr[E ] = 35 5 = 56 8

Example: A city council has 5 Democratic members and 3 Republican members. The mayor appoints one randomly selected member to the Police Commission and another randomly selected member to the Zoning Commission. What is the probability that the member appointed to the Zoning Commission is a Democrat given that the member appointed to the Police Commission is a Democrat? Solution: E F = = appointee to Zoning Com is a Dem appointee to Police Com is a Dem n[E F ] . n[F ]

We want Pr[ E | F ] =

Example: A city council has 5 Democratic members and 3 Republican members. The mayor appoints one randomly selected member to the Police Commission and another randomly selected member to the Zoning Commission. What is the probability that the member appointed to the Zoning Commission is a Democrat given that the member appointed to the Police Commission is a Democrat? Solution: E F = = appointee to Zoning Com is a Dem appointee to Police Com is a Dem n[E F ] . n[F ]

We want Pr[ E | F ] =

n[E F ] = 5 4 = 20 n[F ] = 5 7 = 35 20 4 Pr[ E | F ] = = 35 7

Example: A city council has 5 Democratic members and 3 Republican members. The mayor appoints one randomly selected member to the Police Commission and another randomly selected member to the Zoning Commission. What is the probability that the member appointed to the Zoning Commission is a Democrat given that the member appointed to the Police Commission is a Republican? Solution: E F = = appointee to Zoning Com is a Dem appointee to Police Com is a Dem n[E F ] . n[F ]

We want Pr[ E | F ] =

Example: A city council has 5 Democratic members and 3 Republican members. The mayor appoints one randomly selected member to the Police Commission and another randomly selected member to the Zoning Commission. What is the probability that the member appointed to the Zoning Commission is a Democrat given that the member appointed to the Police Commission is a Republican? Solution: E F = = appointee to Zoning Com is a Dem appointee to Police Com is a Dem n[E F ] . n[F ]

We want Pr[ E | F ] =

n[E F ] = 5 3 = 15 n[F ] = 3 7 = 21 5 15 Pr[ E | F ] = = 21 7

Independence
If the truth of an event has no inuence on the probability of another event being true, we say those two events are independent. Example: Suppose E is the event that you win the lottery today; and F is the event that it rains today.

Independence
If the truth of an event has no inuence on the probability of another event being true, we say those two events are independent. Example: Suppose E is the event that you win the lottery today; and F is the event that it rains today. Pr[ E | F ] = Pr[E ] Pr[ F | E ] = Pr[F ] The two equations are equivalent, and in fact theyre both equivalent to Pr[E F ] = Pr[E ]Pr[F ].

Pr[E F ] = Pr[E ] Pr[F ] n[E F ] n[S ] = n[E ] Pr[F ] n[S ]

n[E F ] = n[E ] Pr[F ] n[E F ] n[E ] = Pr[F ]

Pr[ F | E ] = Pr[F ]

Example: A special die has the numbers 13 in yellow and 46 in red, otherwise the die is normal and fair. Let E be the event the number is even and R the event the number is red. Are E and R independent?

Example: A special die has the numbers 13 in yellow and 46 in red, otherwise the die is normal and fair. Let E be the event the number is even and R the event the number is red. Are E and R independent? Solution: Since the die is fair we know Pr[E ] = Pr[R ] =
1/ 2 1/ . 2

Example: A special die has the numbers 13 in yellow and 46 in red, otherwise the die is normal and fair. Let E be the event the number is even and R the event the number is red. Are E and R independent? Solution: Since the die is fair we know Pr[E ] = Pr[R ] = In an earlier problem we calculated Pr[ E | R ] = 2/3 .
1/ 2 1/ . 2

Example: A special die has the numbers 13 in yellow and 46 in red, otherwise the die is normal and fair. Let E be the event the number is even and R the event the number is red. Are E and R independent? Solution: Since the die is fair we know Pr[E ] = Pr[R ] = In an earlier problem we calculated Pr[ E | R ] = 2/3 . Since Pr[E ] = Pr[ E | R ], then E and R are not independent.
1/ 2 1/ . 2

Example: A special die has the numbers 13 in yellow and 46 in red, otherwise the die is normal and fair. Let E be the event the number is even and R the event the number is red. Are E and R independent? Alternate Solution: Since the die is fair we know Pr[E ] = Pr[R ] =
1/ 2 1/ . 2

Example: A special die has the numbers 13 in yellow and 46 in red, otherwise the die is normal and fair. Let E be the event the number is even and R the event the number is red. Are E and R independent? Alternate Solution: Since the die is fair we know Pr[E ] = Pr[R ] = Also Pr[E R ] =
1/ 2 1/ . 2

n[E R ] 2 = = 1/3 . n[S ] 6

Example: A special die has the numbers 13 in yellow and 46 in red, otherwise the die is normal and fair. Let E be the event the number is even and R the event the number is red. Are E and R independent? Alternate Solution: Since the die is fair we know Pr[E ] = Pr[R ] = Also Pr[E R ] =
1/ 2 1/ . 2

n[E R ] 2 = = 1/3 . n[S ] 6

But Pr[E R ] = Pr[E ]Pr[R ], so E and R are not independent.

Example: 120 tennis balls have been numbered 1120. In addition, 1/3 of the balls are green and 2/3 are white. How many green balls with even numbers are there?

Example: 120 tennis balls have been numbered 1120. In addition, 1/3 of the balls are green and 2/3 are white. How many green balls with even numbers are there? Solution: Theres not enough information given to answer the question.

Example: 120 tennis balls have been numbered 1120. In addition, 1/3 of the balls are green and 2/3 are white. Suppose the events G E = = ball is green ball is even

are known to be independent. How many green balls with even numbers are there?

Example: 120 tennis balls have been numbered 1120. In addition, 1/3 of the balls are green and 2/3 are white. Suppose the events G E = = ball is green ball is even

are known to be independent. How many green balls with even numbers are there? Solution: We know that Pr[G ] = Pr[E ] =
1/ 3 1/ . 2

Example: 120 tennis balls have been numbered 1120. In addition, 1/3 of the balls are green and 2/3 are white. Suppose the events G E = = ball is green ball is even

are known to be independent. How many green balls with even numbers are there? Solution: We know that Pr[G ] = Pr[E ] = Since G and E are independent, Pr[G E ] = Pr[G ] Pr[E ] = 1/3 1/2 = 1/6 .
1/ 3 1/ . 2

Example: 120 tennis balls have been numbered 1120. In addition, 1/3 of the balls are green and 2/3 are white. Suppose the events G E = = ball is green ball is even

are known to be independent. How many green balls with even numbers are there? Solution: We know that Pr[G ] = Pr[E ] = Since G and E are independent, Pr[G E ] = Pr[G ] Pr[E ] = 1/3 1/2 = 1/6 . Therefore n[G E ] = Pr[G E ] n[S ] = 1/6 120 = 20.
1/ 3 1/ . 2

Example: 120 tennis balls have been numbered 1120. In addition, 1/3 of the balls are green and 2/3 are white. Suppose the events G E = = ball is green ball is even

are known to be independent. How many green balls with even numbers are there? Solution: We know that Pr[G ] = Pr[E ] = Since G and E are independent, Pr[G E ] = Pr[G ] Pr[E ] = 1/3 1/2 = 1/6 . Therefore n[G E ] = Pr[G E ] n[S ] = 1/6 120 = 20. (Because Pr[A] = n[A] .) n[S ]
1/ 3 1/ . 2

Example: Suppose you shue a deck of cards, then cut the deck to reveal one card. You do this 10 times. What is the probability you get exactly 3 face cards?

Example: Suppose you shue a deck of cards, then cut the deck to reveal one card. You do this 10 times. What is the probability you get exactly 3 face cards? Solution: In a standard deck there are 12 face cards (F) and 40 nonface cards (N). There are 10 slots to ll, each with one card. There are many cases to consider, for example: F F . . . F F F N N F N N N N N N N N N N N N

The number of cases is the number of anagrams of FFFNNNNNNN: C(10, 3) C(7, 7) = C(10, 3).

The number of cases is the number of anagrams of FFFNNNNNNN: C(10, 3) C(7, 7) = C(10, 3).

The number of outcomes in each case is 123 407 ,

The number of cases is the number of anagrams of FFFNNNNNNN: C(10, 3) C(7, 7) = C(10, 3).

The number of outcomes in each case is 123 407 , so the total number of outcomes is C(10, 3)123 407 .

The number of cases is the number of anagrams of FFFNNNNNNN: C(10, 3) C(7, 7) = C(10, 3).

The number of outcomes in each case is 123 407 , so the total number of outcomes is C(10, 3)123 407 . The size of the sample space is n[S ] = 5210 .

The number of cases is the number of anagrams of FFFNNNNNNN: C(10, 3) C(7, 7) = C(10, 3).

The number of outcomes in each case is 123 407 , so the total number of outcomes is C(10, 3)123 407 . The size of the sample space is n[S ] = 5210 .

Therefore the probability is C(10, 3)123 407 . 5210

C(10, 3)123 407 5210 This can be rewritten: C(10, 3) 123 407 523 527 = C(10, 3) 123 407 523 527 12 52
3

= C(10, 3)

40 52

C(10, 3)123 407 5210 This can be rewritten: C(10, 3) 123 407 523 527 = C(10, 3) 123 407 523 527 12 52
3

= C(10, 3)

40 52

Note that Pr[F ] = Pr[N ] =

12 52 40 . 52

Bernoulli Processes

Situations like that occur often. A Bernoulli Process is a repeated process of selecting an object with replacement, in which we keep track of the number of good objects selected. In the previous problem the good objects were face cards.

Bernoulli Formula
Suppose there are N objects to choose from, of which G are considered good. We select n objects with replacement. N G N G n g ng = = = = = = # objects # good objects # bad objects # selections # good selections # bad selections G = probability of getting a good object N N G = probability of getting a bad object N

p = 1p =

Then the probability of exactly g good objects selected is C(n, g ) G g (N G )(ng ) = C(n, g )p g (1 p )(ng ) . n N

The probability of g good objects in n picks with replacement is C(n, g )(prob of good) (# good picks) (prob of bad) (# bad picks) .

The probability of g good objects in n picks with replacement is C(n, g )(prob of good) (# good picks) (prob of bad) (# bad picks) .

So in the last problem, the probability of 3 face cards in 10 picks is C(10, 3) 12 52


3

40 52

Example: A fair standard die is rolled 12 times. Whats the probability of getting exactly 6 even numbers?

Example: A fair standard die is rolled 12 times. Whats the probability of getting exactly 6 even numbers? Solution: n g ng p 1p = = = = = # selections # good selections # bad selections probability of getting a good object probability of getting a bad object = 12 = 6 = 6 = 1/2 = 1/2

Example: A fair standard die is rolled 12 times. Whats the probability of getting exactly 6 even numbers? Solution: n g ng p 1p = = = = = # selections # good selections # bad selections probability of getting a good object probability of getting a bad object = 12 = 6 = 6 = 1/2 = 1/2

The probability of g = 6 even numbers and 6 odd numbers in n = 12 rolls is

C(n, g )(prob of good) (# good picks) (prob of bad) (# bad picks)

Example: A fair standard die is rolled 12 times. Whats the probability of getting exactly 6 even numbers? Solution: n g ng p 1p = = = = = # selections # good selections # bad selections probability of getting a good object probability of getting a bad object = 12 = 6 = 6 = 1/2 = 1/2

The probability of g = 6 even numbers and 6 odd numbers in n = 12 rolls is

C(n, g )(prob of good) (# good picks) (prob of bad) (# bad picks) = C(12, 6)(1/2 ) (1/2 ) = 0.2256.
6 6

Example: A roulette wheel has 38 slots numbered 0, 00, and 136. Suppose a fair roulette wheel is spun 100 times. Whats the probability of getting 17 exactly three times?

Example: A roulette wheel has 38 slots numbered 0, 00, and 136. Suppose a fair roulette wheel is spun 100 times. Whats the probability of getting 17 exactly three times? Solution: n g ng p 1p = = = = = # selections # good selections # bad selections probability of getting a good object probability of getting a bad object = 100 = 3 = 97 = 1/38 = 37/38

Example: A roulette wheel has 38 slots numbered 0, 00, and 136. Suppose a fair roulette wheel is spun 100 times. Whats the probability of getting 17 exactly three times? Solution: n g ng p 1p = = = = = # selections # good selections # bad selections probability of getting a good object probability of getting a bad object = 100 = 3 = 97 = 1/38 = 37/38

The probability of three 17s is

C(n, g )(prob of good) (# good picks) (prob of bad) (# bad picks)

Example: A roulette wheel has 38 slots numbered 0, 00, and 136. Suppose a fair roulette wheel is spun 100 times. Whats the probability of getting 17 exactly three times? Solution: n g ng p 1p = = = = = # selections # good selections # bad selections probability of getting a good object probability of getting a bad object = 100 = 3 = 97 = 1/38 = 37/38

The probability of three 17s is

C(n, g )(prob of good) (# good picks) (prob of bad) (# bad picks) = C(100, 3)(1/38 ) (37/38 ) = .2218.
3 97

Example: Suppose a fair roulette wheel is spun 100 times. Whats the probability of getting 17 at most three times?

Example: Suppose a fair roulette wheel is spun 100 times. Whats the probability of getting 17 at most three times? Solution: At most 3 means 0, 1, 2, or 3. So repeat the previous method for the cases g = 0, 1, 2, 3 and add.

Example: Suppose a fair roulette wheel is spun 100 times. Whats the probability of getting 17 at most three times? Solution: At most 3 means 0, 1, 2, or 3. So repeat the previous method for the cases g = 0, 1, 2, 3 and add. C(100, 0)(1/38 ) (37/38 ) 1 99 C(100, 1)(1/38 ) (37/38 ) 2 98 C(100, 2)(1/38 ) (37/38 ) 3 97 C(100, 3)(1/38 ) (37/38 ) .7302
0 100

+ + + =

NonELO Probability
If a sample space does not have ELO, then we have to make a few adjustments.

NonELO Probability
If a sample space does not have ELO, then we have to make a few adjustments. The Bernoulli formula can still be used. But any formula that uses n[ ] to measure the size of an outcome must use Pr[ ] instead.

NonELO Probability
If a sample space does not have ELO, then we have to make a few adjustments. The Bernoulli formula can still be used. But any formula that uses n[ ] to measure the size of an outcome must use Pr[ ] instead. ELO: Pr[ E | F ] = NonELO: Pr[ E | F ] = n[E F ] n[F ] Pr[E F ] Pr[F ]

NonELO Probability
If a sample space does not have ELO, then we have to make a few adjustments. The Bernoulli formula can still be used. But any formula that uses n[ ] to measure the size of an outcome must use Pr[ ] instead. ELO: Pr[ E | F ] = NonELO: Pr[ E | F ] = n[E F ] n[F ] Pr[E F ] Pr[F ]

The nonELO version can be used even in ELO problems, but not vice-versa.

The MP for counting outcomes can be translated using the conditional probability formula. Pr[E F ] Pr[F ] = Pr[ E | F ]

The MP for counting outcomes can be translated using the conditional probability formula. Pr[E F ] Pr[F ] = Pr[ E | F ]

Pr[E F ] = Pr[F ] Pr[ E | F ]

The MP for counting outcomes can be translated using the conditional probability formula. Pr[E F ] Pr[F ] = Pr[ E | F ]

Pr[E F ] = Pr[F ] Pr[ E | F ] Pr[E and F both happen] = Pr[F happens rst] Pr[then E happens]

Example: A standard deck of cards is shued and then 2 cards are drawn without replacement. Whats the probability that both cards are Aces? Solution: Let E F Then Pr[E ] = = = rst card is an Ace second card is an Ace. 4 52

Example: A standard deck of cards is shued and then 2 cards are drawn without replacement. Whats the probability that both cards are Aces? Solution: Let E F Then Pr[E ] = Pr[ F | E ] = = = rst card is an Ace second card is an Ace. 4 52 3 51

Example: A standard deck of cards is shued and then 2 cards are drawn without replacement. Whats the probability that both cards are Aces? Solution: Let E F Then Pr[E ] = Pr[ F | E ] = Pr[F ] = = = rst card is an Ace second card is an Ace. 4 52 3 51 4 52

Example: A standard deck of cards is shued and then 2 cards are drawn without replacement. Whats the probability that both cards are Aces? Solution: Let E F Then Pr[E ] = Pr[ F | E ] = Pr[F ] = Therefore Pr[E F ] = Pr[E ] Pr[ F | E ] = 4 3 = .004525. 52 51 = = rst card is an Ace second card is an Ace. 4 52 3 51 4 52

Venn diagrams can still be used, but sizes must be indicated by probability rather than # elements.

Venn diagrams can still be used, but sizes must be indicated by probability rather than # elements. Example: Suppose that Pr[A] = .6, Pr[B ] = .3, and Pr[A B ] = .8. Find Pr[ B | A ].

Venn diagrams can still be used, but sizes must be indicated by probability rather than # elements. Example: Suppose that Pr[A] = .6, Pr[B ] = .3, and Pr[A B ] = .8. Find Pr[ B | A ]. Venn Diagram:

Then Pr[ B | A ] =

Pr[B A] Pr[A]

Then Pr[ B | A ] =

Pr[B A] Pr[A] .1 1 = /6 . .6

Tree Diagrams: The Last Resort


If nothing else works you can try a tree diagram. Tree diagrams are often useful for multistage processes in which later stages depend on earlier ones.

Tree Diagrams: The Last Resort


If nothing else works you can try a tree diagram. Tree diagrams are often useful for multistage processes in which later stages depend on earlier ones. Example: Suppose you have 5 candies in your pocket, 2 of which are butterscotch and the other 3 cherry. You grab one candy at random and eat it. Of course you are only aware of the avor of the candy.

Tree Diagrams: The Last Resort


If nothing else works you can try a tree diagram. Tree diagrams are often useful for multistage processes in which later stages depend on earlier ones. Example: Suppose you have 5 candies in your pocket, 2 of which are butterscotch and the other 3 cherry. You grab one candy at random and eat it. Of course you are only aware of the avor of the candy. Tree Diagram: There are two possible outcomes.

Begin, B, and C are called nodes, the connecting lines are called edges.

We label the edges with probabilities.

For clarity, Ill indicate under each node the number of candies of each avor available.

Now suppose you grab and eat another candy. Now the possibilities look like this.

Notice that the probabilities at the rst stage are ordinary probabilities.

If B1 is the event the rst candy was butterscotch, then Pr[B1 ] = 2/5 .

But the probabilities at the second stage are conditional probabilities.

If C2 is the event the second candy was cherry, then Pr[ C2 | B1 ] = 3/4 .

Everywhere, the probabilities from a single node sum to 1.

If we keep track of the avor of each candy as we eat it, there are 4 possible outcomes.

The probability of each outcome can be computed using the MP: Pr[E ]Pr[ F | E ] = Pr[E F ].

The probability of each outcome can be computed using the MP: Pr[E ]Pr[ F | E ] = Pr[E F ].

The sum of those four probabilities should be 1.

Now we can answer questions. Example: Whats the probability of eating two cherry candies? Solution:

Answer: 6/20

Example: Whats the probability of eating one of each avor? Solution:

Answer: 6/20 + 6/20 = 12/20

Example: Whats the conditional probability that the second is cherry given that the rst was butterscotch? Solution:

Example: Whats the conditional probability that the second is cherry given that the rst was butterscotch? Solution:

Answer:

3 4

Example: Whats the conditional probability that the rst was cherry given that the second was butterscotch? Solution:

Example: Whats the conditional probability that the rst was cherry given that the second was butterscotch? Solution:

Pr[ 1C | 2B ] =

Pr[1C 2B ] Pr[2B ]

Example: Whats the conditional probability that the rst was cherry given that the second was butterscotch? Solution:

Pr[ 1C | 2B ] =

6/ Pr[1C 2B ] 20 =2 Pr[2B ] /20 + 6/20

Example: Whats the conditional probability that the rst was cherry given that the second was butterscotch? Solution:

Pr[ 1C | 2B ] =

6/ Pr[1C 2B ] 6 3 20 =2 = = 6 Pr[2B ] /20 + /20 8 4

Random Variables
A random variable is a variable whose value depends on the outcome of a random process. Example: Let X be the value that comes up when a die is rolled.

Random Variables
A random variable is a variable whose value depends on the outcome of a random process. Example: Let X be the value that comes up when a die is rolled. Whether a die is physically rolled or a computer generates a random number, the only important thing is the probability distribution (p.d.f.), the table of values of X and their probabilities. X 1 2 3 4 5 6 Pr[X ]
1/ 6 1/ 6 1/ 6 1/ 6 1/ 6 1/ 6

Example: Suppose there are 3 red marbles and 2 green marbles in a bowl. Two marbles are taken simultaneously at random from the bowl. Let X be the number of green marbles taken. Write out the p.d.f. of X .

Example: Suppose there are 3 red marbles and 2 green marbles in a bowl. Two marbles are taken simultaneously at random from the bowl. Let X be the number of green marbles taken. Write out the p.d.f. of X . X 0 1 2 Pr[X ] ? ? ?

Example: Suppose there are 3 red marbles and 2 green marbles in a bowl. Two marbles are taken simultaneously at random from the bowl. Let X be the number of green marbles taken. Write out the p.d.f. of X . X 0 1 2 Pr[X ] ? ? ?

Each individual probability like Pr[X = 1] can be computed using the C function.

Example: Suppose there are 3 red marbles and 2 green marbles in a bowl. Two marbles are taken simultaneously at random from the bowl. Let X be the number of green marbles taken. Write out the p.d.f. of X . X 0 1 2 Pr[X ] ? ? ?

Each individual probability like Pr[X = 1] can be computed using the C function. Pr[X = 1] = 6 3 C(3, 1) C(2, 1) = = C(5, 2) 10 5

Example: Suppose there are 3 red marbles and 2 green marbles in a bowl. Two marbles are taken simultaneously at random from the bowl. Let X be the number of green marbles taken. Write out the p.d.f. of X . X 0 1 2
C(3,2) C(5,2) C(3,1) C(2,1) C(5,2) C(2,2) C(5,2)

Pr[X ]
3/ 10 6/ 10 1/ 10

Example: Suppose there are 3 red marbles and 2 green marbles in a bowl. Two marbles are taken simultaneously at random from the bowl. Let X be the number of green marbles taken. Write out the p.d.f. of X . X 0 1 2
C(3,2) C(5,2) C(3,1) C(2,1) C(5,2) C(2,2) C(5,2)

Pr[X ]
3/ 10 6/ 10 1/ 10

1 Note the sum of the probabilities must be 1.

Lets play a game. We both put $1 in the pot. Then we roll a fair die. If the die comes up 6, you win the pot. If anything else comes up, I win.

Lets play a game. We both put $1 in the pot. Then we roll a fair die. If the die comes up 6, you win the pot. If anything else comes up, I win. Is this game fair?

Lets play a game. We both put $1 in the pot. Then we roll a fair die. If the die comes up 6, you win the pot. If anything else comes up, I win. Is this game fair? No . Exactly how unfair is it?

We both put $1 in the pot. Then we roll a fair die. If the die comes up 6, you win the pot. If anything else comes up, I win. Let X be my net winnings. Then X can take only the values 1. Heres the p.d.f.

We both put $1 in the pot. Then we roll a fair die. If the die comes up 6, you win the pot. If anything else comes up, I win. Let X be my net winnings. Then X can take only the values 1. Heres the p.d.f. X -1 1 Pr[X ]
1/ 6 5/ 6

X -1 1

Pr[X ]
1/ 6 5/ 6

The expected value of X , denoted E(X ), is the weighted average of the X values: E(X ) = 1 (1/6 ) + 1 (5/6 ) = 4/6 = 2/3 .

X -1 1

Pr[X ]
1/ 6 5/ 6

The expected value of X , denoted E(X ), is the weighted average of the X values: E(X ) = 1 (1/6 ) + 1 (5/6 ) = 4/6 = 2/3 . This means that on average I win 66.6 every time we play.

X -1 1

Pr[X ]
1/ 6 5/ 6

The expected value of X , denoted E(X ), is the weighted average of the X values: E(X ) = 1 (1/6 ) + 1 (5/6 ) = 4/6 = 2/3 . This means that on average I win 66.6 every time we play. If we play 100 times, I should win about $67.

The easiest way to compute E(X ) is to add an extra column to the p.d.f. X -1 1 Pr[X ]
1/ 6 5/ 6

X Pr[X ] 1/6 5/ 6
4/ 6

= E(X )

Then E(X ) is the sum of the entries in the X Pr[X ] column.

Example: Suppose there are 3 red marbles and 2 green marbles in a bowl. Two marbles are taken simultaneously at random from the bowl. Let X be the number of green marbles taken. Find the expected value of X .

Example: Suppose there are 3 red marbles and 2 green marbles in a bowl. Two marbles are taken simultaneously at random from the bowl. Let X be the number of green marbles taken. Find the expected value of X . Solution: Recall the p.d.f. X 0 1 2 Add the extra column. Pr[X ]
3/ 10 6/ 10 1/ 10

Example: Suppose there are 3 red marbles and 2 green marbles in a bowl. Two marbles are taken simultaneously at random from the bowl. Let X be the number of green marbles taken. Find the expected value of X . Solution: Recall the p.d.f. X 0 1 2 Pr[X ]
3/ 10 6/ 10 1/ 10

X Pr[X ] 0
6/ 10 2/ 10 8/ 10

Add the extra column.

Example: Suppose there are 3 red marbles and 2 green marbles in a bowl. Two marbles are taken simultaneously at random from the bowl. Let X be the number of green marbles taken. Find the expected value of X . Solution: Recall the p.d.f. X 0 1 2 Pr[X ]
3/ 10 6/ 10 1/ 10

X Pr[X ] 0
6/ 10 2/ 10 8/ 10

Add the extra column. So E(X ) = 8/10 = 4/5 .

Example: Suppose there are 3 red marbles and 2 green marbles in a bowl. Two marbles are taken simultaneously at random from the bowl. Let X be the number of green marbles taken. Find the expected value of X . E(X ) = 4/5 Shouldnt that have been obvious?

Example: Suppose there are 3 red marbles and 2 green marbles in a bowl. Two marbles are taken simultaneously at random from the bowl. Let X be the number of green marbles taken. Find the expected value of X . E(X ) = 4/5 Shouldnt that have been obvious? Of all the marbles, 2/5 are green. So 2/5 of the 2 marbles we take should be green. E(X ) = 2/5 2

This shortcut works for sampling problems, i.e. when objects are chosen from the universe with ELO, with or without replacement.

This shortcut works for sampling problems, i.e. when objects are chosen from the universe with ELO, with or without replacement. In general, If n samples are taken from a universe where the fraction of good objects is p , and X is the number of good objects in the sample, then E(X ) = p n.

Example: You sit at the roulette wheel for 36 hours, betting 1000 times on your lucky number, 17. What is the expected number of times that you win?

Example: You sit at the roulette wheel for 36 hours, betting 1000 times on your lucky number, 17. What is the expected number of times that you win? Solution: E(X ) = 1 1000 = 26.32 38

Linear Algebra
A linear equation is one in which variables are used in the simplest possible way: multiplied by a constant and added to the other terms.

Linear Algebra
A linear equation is one in which variables are used in the simplest possible way: multiplied by a constant and added to the other terms. Examples Linear: 2x + 3y = 4 .1a 5b = 137c

Linear Algebra
A linear equation is one in which variables are used in the simplest possible way: multiplied by a constant and added to the other terms. Examples Linear: 2x + 3y = 4 .1a 5b = 137c Nonlinear: x y xy 2 x 3y 2 = z = 1 = 5

Linear equations get their name from their graphs. Here is the graph of 3x 2y = 6.
6 4 2 0 -2 -4 -6 -6 -4 -2 0 y 2 4 6 x

Linear equations get their name from their graphs. Here is the graph of 3x 2y = 6.
6 4 2 0 -2 -4 -6 -6 -4 -2 0 y 2 4 6 x

Each point on the blue line has coordinates that satisfy 3x 2y = 6.

The points where the graph crosses the axes are called intercepts.
6 4 2 0 -2 -4 -6 -6 -4 -2 0 y 2 4 6 x

The points where the graph crosses the axes are called intercepts.
6 4 2 0 -2 -4 -6 -6 -4 -2 0 y 2 4 6 x

For the line 3x 2y = 6 the x -intercept is (2, 0) and the y -intercept is (0, 3).

To nd the intercepts for a linear equation: x -intercept This is where the line crosses the x -axis, so y = 0. Plug in y = 0 and solve for x . y -intercept This is where the line crosses the y -axis, so x = 0. Plug in x = 0 and solve for y .

To nd the intercepts for a linear equation: x -intercept This is where the line crosses the x -axis, so y = 0. Plug in y = 0 and solve for x . y -intercept This is where the line crosses the y -axis, so x = 0. Plug in x = 0 and solve for y . Example: For 3x 2y = 6, x : 3x 2(0) = 6 = x = 2 y : 3(0) 2y = 6 = y = 3.

If youre asked to graph a linear equation, all you need are two distinct points satisfying the equation (for example, the intercepts). Just plot them and draw a line through them.

If youre asked to graph a linear equation, all you need are two distinct points satisfying the equation (for example, the intercepts). Just plot them and draw a line through them. Example: Graph the linear equation x + 3y = 4. Solution: Here its easy to nd one solution: (1, 1). You can nd others without much trouble, e.g. (2, 2). Or you can nd the intercepts: x : x + 3(0) = 4 = x = 4 y: 0 + 3y = 4 = y = 4/3 . That gives us (4, 0) and (0, 4/3 ).

It doesnt matter which two points you use since they all lie on the same line.
6 4 2 0 -2 -4 -6 -6 -4 -2 0 2 4 6

It doesnt matter which two points you use since they all lie on the same line.
6 4 2 0 -2 -4 -6 -6 -4 -2 0 2 4 6

But you might want to plot extra points as a check.

Slope

Slope is a measure of the steepness of a line.

Slope

Slope is a measure of the steepness of a line. If the line rises from left to right, slope is +. If the line falls from left to right, slope is .

Slope

Slope is a measure of the steepness of a line. If the line rises from left to right, slope is +. If the line falls from left to right, slope is . The formula for slope is slope = rise vertical change = . run horizontal change

Consider the line we just graphed.


6 4 2 0 -2 -4 -6 -6 -4 -2 0 2 4 6

As the line travels from (2, 2) to (4, 0): the vertical change is 0 2 = 2 and the horizontal change is 4 (2) = 6.

Consider the line we just graphed.


6 4 2 0 -2 -4 -6 -6 -4 -2 0 2 4 6

As the line travels from (2, 2) to (4, 0): the vertical change is 0 2 = 2 and the horizontal change is 4 (2) = 6.

So the slope of the line is rise 2 1 = = . run 6 3

Consider the line we just graphed.


6 4 2 0 -2 -4 -6 -6 -4 -2 0 2 4 6

As the line travels from (2, 2) to (4, 0): the vertical change is 0 2 = 2 and the horizontal change is 4 (2) = 6.

So the slope of the line is rise 2 1 = = . run 6 3 We could have used any two points because they all give the same result.

Jack and Jill have a date on a hill.

Jack is climbing up the line from (1,5) to Jills picnic site at (4,14).

Jack and Jill have a date on a hill.

Jack is climbing up the line from (1,5) to Jills picnic site at (4,14). Jacks path; 1. Straight line; use Pythagorean Theorem to nd distance;

Jack and Jill have a date on a hill.

Jack is climbing up the line from (1,5) to Jills picnic site at (4,14). Jacks path; 1. Straight line; use Pythagorean Theorem to nd distance; 2. Horizontal and vertical components.

Jack and Jill have a date on a hill.

Jack is climbing up the line from (1,5) to Jills picnic site at (4,14). Jacks path; 1. Straight line; use Pythagorean Theorem to nd distance; 2. Horizontal and vertical components.
2.1 Horizontal: 4 1 = 3

Jack and Jill have a date on a hill.

Jack is climbing up the line from (1,5) to Jills picnic site at (4,14). Jacks path; 1. Straight line; use Pythagorean Theorem to nd distance; 2. Horizontal and vertical components.
2.1 Horizontal: 4 1 = 3 2.2 Vertical: 14 5 = 9

In general, if a line passes through the points (x1 , y1 ) and (x2 , y2 ) then its slope is Slope = m = y2 y1 x2 x1

In general, if a line passes through the points (x1 , y1 ) and (x2 , y2 ) then its slope is Slope = m = y2 y1 y1 y2 = . x2 x1 x1 x2

In general, if a line passes through the points (x1 , y1 ) and (x2 , y2 ) then its slope is Slope = m = y2 y1 y1 y2 = . x2 x1 x1 x2

You may also remember this rule from high school: The slope of the line Ax + By = C is B/A .

You may also remember this rule from high school: The slope of the line Ax + By = C is B/A . Example: Find the slope of the line 2x 3y = 5.

You may also remember this rule from high school: The slope of the line Ax + By = C is B/A . Example: Find the slope of the line 2x 3y = 5. Solution: We have A = 2, B = 3, and C = 5, so m = (3/2 ) = 3/2 .

Whats the formula for the line with the following properties?

Whats the formula for the line with the following properties? 1. The vertical intercept is 2, i.e. it passes through (0, 2); 2. The slope is 3.

Whats the formula for the line with the following properties? 1. The vertical intercept is 2, i.e. it passes through (0, 2); 2. The slope is 3. Fact 2 means the height grows like 3x : one unit of x produces three units of y .

Whats the formula for the line with the following properties? 1. The vertical intercept is 2, i.e. it passes through (0, 2); 2. The slope is 3. Fact 2 means the height grows like 3x : one unit of x produces three units of y . So y = 3x + ?.

Whats the formula for the line with the following properties? 1. The vertical intercept is 2, i.e. it passes through (0, 2); 2. The slope is 3. Fact 2 means the height grows like 3x : one unit of x produces three units of y . So y = 3x + ?. To nd the value of ?, use Fact 1:

Whats the formula for the line with the following properties? 1. The vertical intercept is 2, i.e. it passes through (0, 2); 2. The slope is 3. Fact 2 means the height grows like 3x : one unit of x produces three units of y . So y = 3x + ?. To nd the value of ?, use Fact 1: 2 = 3 0 + ?.

Whats the formula for the line with the following properties? 1. The vertical intercept is 2, i.e. it passes through (0, 2); 2. The slope is 3. Fact 2 means the height grows like 3x : one unit of x produces three units of y . So y = 3x + ?. To nd the value of ?, use Fact 1: 2 = 3 0 + ?. So ? has a value of 2. y = 3x + 2

Whats the formula for the line with the following properties? 1. The vertical intercept is 2, i.e. it passes through (0, 2); 2. The slope is 3. Fact 2 means the height grows like 3x : one unit of x produces three units of y . So y = 3x + ?. To nd the value of ?, use Fact 1: 2 = 3 0 + ?. So ? has a value of 2. y = 3x + 2 In general, the equation of a line can be written y = mx + b (Slope-Intercept Form)

where m is the slope and b is from the vertical intercept.

Heres another form of the linear equation:

Heres another form of the linear equation: Start with the denition of slope: Slope = m = y2 y1 . x2 x1

Heres another form of the linear equation: Start with the denition of slope: Slope = m = y2 y1 . x2 x1

Now rearrange terms to get rid of the fraction: m(x2 x1 ) = y2 y1 .

Heres another form of the linear equation: Start with the denition of slope: Slope = m = y2 y1 . x2 x1

Now rearrange terms to get rid of the fraction: m(x2 x1 ) = y2 y1 . Since this holds for any (x2 , y2 ) on the line, we can think of it as the dening equation of the line: m(x x1 ) = y y1 .

Heres another form of the linear equation: Start with the denition of slope: Slope = m = y2 y1 . x2 x1

Now rearrange terms to get rid of the fraction: m(x2 x1 ) = y2 y1 . Since this holds for any (x2 , y2 ) on the line, we can think of it as the dening equation of the line: m(x x1 ) = y y1 .

Heres another form of the linear equation: Start with the denition of slope: Slope = m = y2 y1 . x2 x1

Now rearrange terms to get rid of the fraction: m(x2 x1 ) = y2 y1 . Since this holds for any (x2 , y2 ) on the line, we can think of it as the dening equation of the line: m(x x1 ) = y y1 . This is called point-slope form.

Heres another form of the linear equation: Start with the denition of slope: Slope = m = y2 y1 . x2 x1

Now rearrange terms to get rid of the fraction: m(x2 x1 ) = y2 y1 . Since this holds for any (x2 , y2 ) on the line, we can think of it as the dening equation of the line: m(x x1 ) = y y1 . This is called point-slope form. Both forms have their strengths and weaknesses for problem solving.

Types of problems:

Types of problems: 1. Given information about a line, gure out its equation.

Types of problems: 1. Given information about a line, gure out its equation.

Types of problems: 1. Given information about a line, gure out its equation.

Point-Slope form is usually best.

Types of problems: 1. Given information about a line, gure out its equation.

Point-Slope form is usually best. 2. Given the equation of a line, determine facts about it.

Types of problems: 1. Given information about a line, gure out its equation.

Point-Slope form is usually best. 2. Given the equation of a line, determine facts about it.

Types of problems: 1. Given information about a line, gure out its equation.

Point-Slope form is usually best. 2. Given the equation of a line, determine facts about it.

Slope-Intercept form is usually best.

Example:

Example: Given the equation 4y + 2x + 8 = 0, nd the slope and y -intercept.

Example: Given the equation 4y + 2x + 8 = 0, nd the slope and y -intercept.

Slope-Intercept form is usually best.

Example: Given the equation 4y + 2x + 8 = 0, nd the slope and y -intercept.

Slope-Intercept form is usually best. Solution Rewrite in Slope-Intercept form: 4y = 2x 8 1 y = x + 2. 2 Now we can read o the info we need: m= 1 2 b=2

Example:

Example: A line passes through the points (4, 5) and (2, 1). What is the equation of the line?

Example: A line passes through the points (4, 5) and (2, 1). What is the equation of the line?

Point-Slope form is usually best.

Example: A line passes through the points (4, 5) and (2, 1). What is the equation of the line?

Point-Slope form is usually best. Solution: First nd the slope. m= y2 y1 1 5 6 = = =3 x2 x1 24 2

Example: A line passes through the points (4, 5) and (2, 1). What is the equation of the line?

Point-Slope form is usually best. Solution: First nd the slope. m= y2 y1 1 5 6 = = =3 x2 x1 24 2

Now apply the point-slope form. y 5 = 3(x 4)

There are exceptions.

There are exceptions. Example:

There are exceptions. Example: The population was 30,700 and grew at a rate of 850 per year.

There are exceptions. Example: The population was 30,700 and grew at a rate of 850 per year. 1. Give the linear equation for p and t (years since 2000).

There are exceptions. Example: The population was 30,700 and grew at a rate of 850 per year. 1. Give the linear equation for p and t (years since 2000). 2. What is the population in the year 2010?

There are exceptions. Example: The population was 30,700 and grew at a rate of 850 per year. 1. Give the linear equation for p and t (years since 2000). 2. What is the population in the year 2010? 3. When will the population be 45,000?

There are exceptions. Example: The population was 30,700 and grew at a rate of 850 per year. 1. Give the linear equation for p and t (years since 2000). 2. What is the population in the year 2010? 3. When will the population be 45,000? Solution

There are exceptions. Example: The population was 30,700 and grew at a rate of 850 per year. 1. Give the linear equation for p and t (years since 2000). 2. What is the population in the year 2010? 3. When will the population be 45,000? Solution 1. p = 850t + 30, 700.

There are exceptions. Example: The population was 30,700 and grew at a rate of 850 per year. 1. Give the linear equation for p and t (years since 2000). 2. What is the population in the year 2010? 3. When will the population be 45,000? Solution 1. p = 850t + 30, 700.

There are exceptions. Example: The population was 30,700 and grew at a rate of 850 per year. 1. Give the linear equation for p and t (years since 2000). 2. What is the population in the year 2010? 3. When will the population be 45,000? Solution 1. p = 850t + 30, 700. 2. Plug in t = 10. p = 850 10 + 30,700 = 39,200

There are exceptions. Example: The population was 30,700 and grew at a rate of 850 per year. 1. Give the linear equation for p and t (years since 2000). 2. What is the population in the year 2010? 3. When will the population be 45,000? Solution 1. p = 850t + 30, 700. 2. Plug in t = 10. p = 850 10 + 30,700 = 39,200 3. Plug in p = 45,000. 45,000 = 850t + 30,700

There are exceptions. Example: The population was 30,700 and grew at a rate of 850 per year. 1. Give the linear equation for p and t (years since 2000). 2. What is the population in the year 2010? 3. When will the population be 45,000? Solution 1. p = 850t + 30, 700. 2. Plug in t = 10. p = 850 10 + 30,700 = 39,200 3. Plug in p = 45,000. 45,000 = 850t + 30,700 14,300 = 850t

There are exceptions. Example: The population was 30,700 and grew at a rate of 850 per year. 1. Give the linear equation for p and t (years since 2000). 2. What is the population in the year 2010? 3. When will the population be 45,000? Solution 1. p = 850t + 30, 700. 2. Plug in t = 10. p = 850 10 + 30,700 = 39,200 3. Plug in p = 45,000. 45,000 = 850t + 30,700 14,300 = 850t t 16.8

Example:

Example: Latitude 11 44 # Species 34 26

Example: Latitude 11 44 (a) First nd the slope: m= N2 N1 34 26 8 = = . l2 l1 11 44 33 # Species 34 26

Example: Latitude 11 44 (a) First nd the slope: m= N2 N1 34 26 8 = = . l2 l1 11 44 33 # Species 34 26

Now apply point-slope form: N 26 = 8 (l 44). 33

8 (l 44). 33 (b) Find the slope and y -intercept. N 26 =

8 (l 44). 33 (b) Find the slope and y -intercept. Lets rst rewrite the equation in slope-intercept form: N 26 = N= 8 8 8 110 l + 44 + 26 = l + . 33 33 33 3

8 (l 44). 33 (b) Find the slope and y -intercept. Lets rst rewrite the equation in slope-intercept form: N 26 = N= Now we can say:
8 species per degree; m = 33 110 b = 3 species at the equator.

8 8 8 110 l + 44 + 26 = l + . 33 33 33 3

8 (l 44). 33 (b) Find the slope and y -intercept. Lets rst rewrite the equation in slope-intercept form: N 26 = N= Now we can say:
8 species per degree; m = 33 110 b = 3 species at the equator.

8 8 8 110 l + 44 + 26 = l + . 33 33 33 3

(c) Graph the line.

8 (l 44). 33 (b) Find the slope and y -intercept. Lets rst rewrite the equation in slope-intercept form: N 26 = N= Now we can say:
8 species per degree; m = 33 110 b = 3 species at the equator.

8 8 8 110 l + 44 + 26 = l + . 33 33 33 3

(c) Graph the line.

Problem: A coee shop sells coee to go in three sizes.

Problem: A coee shop sells coee to go in three sizes.

Problem: A coee shop sells coee to go in three sizes.

Assuming the relationship between price and size is linear, what does the 12 oz. coee cost?

Size in oz. x 8 12 20

Price in $ y 1.00 ? 2.08

Size in oz. x 8 12 20

Price in $ y 1.00 ? 2.08

(Its OK to reverse x and y , just be consistent.)

Size in oz. x 8 12 20

Price in $ y 1.00 ? 2.08

(Its OK to reverse x and y , just be consistent.) Solution: First nd the slope: m= 1.08 2.08 1.00 = = 0.09. 20 8 12

Size in oz. x 8 12 20

Price in $ y 1.00 ? 2.08

(Its OK to reverse x and y , just be consistent.) Solution: First nd the slope: m= Therefore y 1.00 = 0.09(x 8) 1.08 2.08 1.00 = = 0.09. 20 8 12

Size in oz. x 8 12 20

Price in $ y 1.00 ? 2.08

(Its OK to reverse x and y , just be consistent.) Solution: First nd the slope: m= Therefore y 1.00 = 0.09(x 8) and y = 0.09x + 0.28. 1.08 2.08 1.00 = = 0.09. 20 8 12

Size in oz. x 8 12 20

Price in $ y 1.00 ? 2.08

(Its OK to reverse x and y , just be consistent.) Solution: First nd the slope: m= Therefore y 1.00 = 0.09(x 8) and y = 0.09x + 0.28. Finally, when x = 12, y = 0.09 12 + 0.28 = $1.36. 1.08 2.08 1.00 = = 0.09. 20 8 12

y = 0.09x + 0.28.

y = 0.09x + 0.28. Problem: What do m = 0.09 and b = 0.28 (the y -intercept) mean in this problem?

y = 0.09x + 0.28. Problem: What do m = 0.09 and b = 0.28 (the y -intercept) mean in this problem? Solution: The slope is rise/run, in this case $/oz.

y = 0.09x + 0.28. Problem: What do m = 0.09 and b = 0.28 (the y -intercept) mean in this problem? Solution: The slope is rise/run, in this case $/oz. So evidently coee costs 9 per ounce.

y = 0.09x + 0.28. Problem: What do m = 0.09 and b = 0.28 (the y -intercept) mean in this problem? Solution: The slope is rise/run, in this case $/oz. So evidently coee costs 9 per ounce. The value of b is the value that y takes when x = 0.

y = 0.09x + 0.28. Problem: What do m = 0.09 and b = 0.28 (the y -intercept) mean in this problem? Solution: The slope is rise/run, in this case $/oz. So evidently coee costs 9 per ounce. The value of b is the value that y takes when x = 0. So if you buy 0 ounces of coee it still costs you 28.

y = 0.09x + 0.28. Problem: What do m = 0.09 and b = 0.28 (the y -intercept) mean in this problem? Solution: The slope is rise/run, in this case $/oz. So evidently coee costs 9 per ounce. The value of b is the value that y takes when x = 0. So if you buy 0 ounces of coee it still costs you 28. Evidently that is the price of renting a mug.

y = 0.09x + 0.28. Problem: What do m = 0.09 and b = 0.28 (the y -intercept) mean in this problem? Solution: The slope is rise/run, in this case $/oz. So evidently coee costs 9 per ounce. The value of b is the value that y takes when x = 0. So if you buy 0 ounces of coee it still costs you 28. Evidently that is the price of renting a mug. b = xed costs mx = variable costs

Systems of Equations
Sometimes its easier to set up story problems using more than one variable.

Systems of Equations
Sometimes its easier to set up story problems using more than one variable. Example: Suppose that a mother is 25 years older than her son, and that in 15 years she will be twice as old as her son. How old are they now?

Systems of Equations
Sometimes its easier to set up story problems using more than one variable. Example: Suppose that a mother is 25 years older than her son, and that in 15 years she will be twice as old as her son. How old are they now? Solution: Let x y = Mothers age = Sons age.

Systems of Equations
Sometimes its easier to set up story problems using more than one variable. Example: Suppose that a mother is 25 years older than her son, and that in 15 years she will be twice as old as her son. How old are they now? Solution: Let x y The rst phrase says that x = y + 25, = Mothers age = Sons age.

Systems of Equations
Sometimes its easier to set up story problems using more than one variable. Example: Suppose that a mother is 25 years older than her son, and that in 15 years she will be twice as old as her son. How old are they now? Solution: Let x y The rst phrase says that x = y + 25, and the second phrase says that x + 15 = 2(y + 15). = Mothers age = Sons age.

Systems of Equations
Sometimes its easier to set up story problems using more than one variable. Example: Suppose that a mother is 25 years older than her son, and that in 15 years she will be twice as old as her son. How old are they now? Solution: Let x y The rst phrase says that x = y + 25, and the second phrase says that x + 15 = 2(y + 15). = Mothers age = Sons age.

This gives us the system of Equations x = y + 25 x + 15 = 2(y + 15).

This gives us the system of Equations x = y + 25 x + 15 = 2(y + 15).

This particular system can be solved easily using the substitution method.

This gives us the system of Equations x = y + 25 x + 15 = 2(y + 15).

This particular system can be solved easily using the substitution method. (y + 25) + 15 = 2(y + 15)

This gives us the system of Equations x = y + 25 x + 15 = 2(y + 15).

This particular system can be solved easily using the substitution method. (y + 25) + 15 = 2(y + 15) y + 40 = 2y + 30

This gives us the system of Equations x = y + 25 x + 15 = 2(y + 15).

This particular system can be solved easily using the substitution method. (y + 25) + 15 = 2(y + 15) y + 40 = 2y + 30 y 30 = y 30

This gives us the system of Equations x = y + 25 x + 15 = 2(y + 15).

This particular system can be solved easily using the substitution method. (y + 25) + 15 = 2(y + 15) y + 40 = 2y + 30 y 30 = y 30 10 = y

This gives us the system of Equations x = y + 25 x + 15 = 2(y + 15).

This particular system can be solved easily using the substitution method. (y + 25) + 15 = 2(y + 15) y + 40 = 2y + 30 y 30 = y 30 10 = y Then x = 35.

Substitution Method
1. 2. 3. 4. Solve for one variable in one equation. Plug that result into the other equation. Solve for the remaining variable. Go back to (1) to evaluate the rst variable.

Substitution Method
1. 2. 3. 4. Solve for one variable in one equation. Plug that result into the other equation. Solve for the remaining variable. Go back to (1) to evaluate the rst variable. x +y x y = 2 = 0

Example:

Substitution Method
1. 2. 3. 4. Solve for one variable in one equation. Plug that result into the other equation. Solve for the remaining variable. Go back to (1) to evaluate the rst variable. x +y x y 1. x = 2 y = 2 = 0

Example:

Substitution Method
1. 2. 3. 4. Solve for one variable in one equation. Plug that result into the other equation. Solve for the remaining variable. Go back to (1) to evaluate the rst variable. x +y x y 1. x = 2 y 2. (2 y ) y = 0 = 2 = 0

Example:

Substitution Method
1. 2. 3. 4. Solve for one variable in one equation. Plug that result into the other equation. Solve for the remaining variable. Go back to (1) to evaluate the rst variable. x +y x y 1. x = 2 y 2. (2 y ) y = 0 3. 2 2y 2y y = 0 = 2 = 1 = 2 = 0

Example:

Substitution Method
1. 2. 3. 4. Solve for one variable in one equation. Plug that result into the other equation. Solve for the remaining variable. Go back to (1) to evaluate the rst variable. x +y x y 1. x = 2 y 2. (2 y ) y = 0 3. 2 2y 2y y 4. x = 2 y = 2 1 = 1 = 0 = 2 = 1 = 2 = 0

Example:

Elimination Method
1. 2. 3. 4. 5. Put both equations in standard form (variables on left). Multiply either or both equations by constant. Add the equations together, eliminating one variable. Solve for the remaining variable. Plug that into the rst equation and solve for the rst variable.

Elimination Method
1. 2. 3. 4. 5. Put both equations in standard form (variables on left). Multiply either or both equations by constant. Add the equations together, eliminating one variable. Solve for the remaining variable. Plug that into the rst equation and solve for the rst variable. x +y x y = 2 = 0

Example:

Elimination Method
1. 2. 3. 4. 5. Put both equations in standard form (variables on left). Multiply either or both equations by constant. Add the equations together, eliminating one variable. Solve for the remaining variable. Plug that into the rst equation and solve for the rst variable. x +y x y 1. Already done. = 2 = 0

Example:

Elimination Method
1. 2. 3. 4. 5. Put both equations in standard form (variables on left). Multiply either or both equations by constant. Add the equations together, eliminating one variable. Solve for the remaining variable. Plug that into the rst equation and solve for the rst variable. x +y x y 1. Already done. 2. Already done. = 2 = 0

Example:

Elimination Method
1. 2. 3. 4. 5. Put both equations in standard form (variables on left). Multiply either or both equations by constant. Add the equations together, eliminating one variable. Solve for the remaining variable. Plug that into the rst equation and solve for the rst variable. x +y x y 1. Already done. 2. Already done. 3. 2x = 2. = 2 = 0

Example:

Elimination Method
1. 2. 3. 4. 5. Put both equations in standard form (variables on left). Multiply either or both equations by constant. Add the equations together, eliminating one variable. Solve for the remaining variable. Plug that into the rst equation and solve for the rst variable. x +y x y 1. 2. 3. 4. 5. Already done. Already done. 2x = 2. x = 1. 1+y y = 2 = 1 = 2 = 0

Example:

Example: Solve the system 2x 4y = 4 3y = 5 3x .

Example: Solve the system 2x 4y Solution: 1. 2x + 3 y 3x + 4 y = 4 = 5. = 4 3y = 5 3x .

Example: Solve the system 2x 4y Solution: 1. 2x + 3 y 3x + 4 y = 4 = 5. = 4 3y = 5 3x .

2. Multiply the rst equation by 3, the second by 2. 6x + 9 y 6x 8y = 12 = 10.

Example: Solve the system 2x 4y Solution: 1. 2x + 3 y 3x + 4 y = 4 = 5. = 4 3y = 5 3x .

2. Multiply the rst equation by 3, the second by 2. 6x + 9 y 6x 8y 3. y = 2 4. y = 2 = 12 = 10.

Example: Solve the system 2x 4y Solution: 1. 2x + 3 y 3x + 4 y = 4 = 5. = 4 3y = 5 3x .

2. Multiply the rst equation by 3, the second by 2. 6x + 9 y 6x 8y 3. y = 2 4. y = 2 5. 2x 2x x = 4 3(2) = 2 = 1 = 12 = 10.

Graphic Systems
2x 4y
3

= 4 3y = 5 3x .

2.5

1.5

0.5 -2 -1.5 -1 -0.5 0 0.5

Graphic Systems
2x 4y
3

= 4 3y = 5 3x .

2.5

1.5

0.5 -2 -1.5 -1 -0.5 0 0.5

Solution: (1, 2) The solution is the point of intersection.

System Weirdness
Things can go wrong.

System Weirdness
Things can go wrong. Example: x +y y = 2 = 4x

System Weirdness
Things can go wrong. Example: x +y y Substitution produces x + (4 x ) = 2 = 2 = 4x

System Weirdness
Things can go wrong. Example: x +y y Substitution produces x + (4 x ) = 2 4 = 2 Contradiction. = 2 = 4x

System Weirdness
Things can go wrong. Example: x +y y Substitution produces x + (4 x ) = 2 4 = 2 Contradiction.
3 2.5 2 1.5 1 0.5 0 0 0.5 1 1.5 2 2.5 3

= 2 = 4x

So this system of equations has no solution, because the lines are parallel.

Heres another avor of weirdness. x +y y = 2 = 2x

Heres another avor of weirdness. x +y y = 2 = 2x

This time substitution produces x + (2 x ) = 2

Heres another avor of weirdness. x +y y = 2 = 2x

This time substitution produces x + (2 x ) = 2 2 = 2.

Heres another avor of weirdness. x +y y = 2 = 2x

This time substitution produces x + (2 x ) = 2 2 = 2. This is an identity.

Heres another avor of weirdness. x +y y = 2 = 2x

This time substitution produces x + (2 x ) = 2 2 = 2. This is an identity. The interpretation is that the two equations must have been equivalent.

Heres another avor of weirdness. x +y y = 2 = 2x

This time substitution produces x + (2 x ) = 2 2 = 2. This is an identity. The interpretation is that the two equations must have been equivalent. So the lines are actually the same.

Heres another avor of weirdness. x +y y = 2 = 2x

This time substitution produces x + (2 x ) = 2 2 = 2. This is an identity. The interpretation is that the two equations must have been equivalent. So the lines are actually the same. So in this case there are innitely many solutions: every point on the line satises both equations.

x +y y Innitely many solutions.

= 2 = 2x

x +y y Innitely many solutions.

= 2 = 2x

Even in this case we can do a little more:

x +y y Innitely many solutions.

= 2 = 2x

Even in this case we can do a little more: provide a recipe for producing as many solutions as desired.

x +y y Innitely many solutions.

= 2 = 2x

Even in this case we can do a little more: provide a recipe for producing as many solutions as desired. Note that the second equation gives y as a function of x . y =2x

x +y y Innitely many solutions.

= 2 = 2x

Even in this case we can do a little more: provide a recipe for producing as many solutions as desired. Note that the second equation gives y as a function of x . y =2x This is a recipe for y in terms of x .

x +y y Innitely many solutions.

= 2 = 2x

Even in this case we can do a little more: provide a recipe for producing as many solutions as desired. Note that the second equation gives y as a function of x . y =2x This is a recipe for y in terms of x . If we think of x as the input, then y is the output.

x +y y Innitely many solutions.

= 2 = 2x

Even in this case we can do a little more: provide a recipe for producing as many solutions as desired. Note that the second equation gives y as a function of x . y =2x This is a recipe for y in terms of x . If we think of x as the input, then y is the output. x is called the free variable or arbitrary parameter.

Reduction Method

Reduction Method = Elimination Method - Cleverness

Reduction Method

Reduction Method = Elimination Method - Cleverness Or alternately, Reduction is Elimination by brute force.

Reduction Method

Reduction Method = Elimination Method - Cleverness Or alternately, Reduction is Elimination by brute force. This is not pretty.

Example 2x + 3 y 3x + 4 y = 4 = 5

Example 2x + 3 y 3x + 4 y Lets change that 2x into a 1x . = 4 = 5

Example 2x + 3 y 3x + 4 y Lets change that 2x into a 1x .


1 2 (2x

= 4 = 5

+ 3y

= 4) = = 5

3 x+2 y

= 2 = 5

3x + 4 y

3x + 4y

Example 2x + 3 y 3x + 4 y Lets change that 2x into a 1x .


1 2 (2x

= 4 = 5

+ 3y

= 4) = = 5

3 x+2 y

= 2 = 5

3x + 4 y

3x + 4y

Note that we always write both equations together, but we modify only one at a time.

x+3 2y 3x + 4 y

= 2 = 5

x+3 2y 3x + 4 y Now lets eliminate the x . 3(x + 3 2y 3x + 4 y = 2) = = 5

= 2 = 5

9 3x 2 y 8 y 3x + 2

= 6 = 5

x+3 2y 3x + 4 y Now lets eliminate the x . 3(x + 3 2y 3x + 4 y = 2) = = 5

= 2 = 5

9 3x 2 y 8 y 3x + 2 1 y 2

= 6 = 5 = 1

x+3 2y 3x + 4 y Now lets eliminate the x . 3(x + 3 2y 3x + 4 y = 2) = = 5

= 2 = 5

9 3x 2 y 8 y 3x + 2 1 y 2

= 6 = 5 = 1

Lets regard the result as a new improved equation 2, and lets keep the simpler old equation 1.
3 x+2 y 1 2 y

= 2 = 1

3 y x+2 1 2 y

= 2 = 1

3 y x+2 1 2 y

= 2 = 1

Now lets nish solving for y . x+3 2y


1 2 2 y

= 2 = = 1)

x+3 2y y

= 2 = 2

3 y x+2 y

= 2 = 2

3 y x+2 y

= 2 = 2

Finally, lets eliminate y from equation 1. x+3 2y 3 2 (y = 2 = = 2) 3 2y = 3


3 x+2 y

= 2

3 y x+2 y

= 2 = 2

Finally, lets eliminate y from equation 1. x+3 2y 3 2 (y = 2 = = 2) x 3 2y = 3 = 1


3 x+2 y

= 2

3 y x+2 y

= 2 = 2

Finally, lets eliminate y from equation 1. x+3 2y 3 2 (y = 2 = = 2) x 3 2y = 3 = 1


3 x+2 y

= 2

So the complete simplication (and solution) is x y = 1 = 2.

Note there were only two things we had to know to do: Multiply an equation by a constant. Add a multiple of one equation to another.
+

Note there were only two things we had to know to do: Multiply an equation by a constant. Add a multiple of one equation to another. You can also do: Reverse the equations.
+

Now lets work on speed. Shorthand notation: Instead of 2x + 3 y 3x + 4 y well write 2 3 4 3 4 5 . = 4 = 5

2 3 4 3 4 5 The rst step: get a 1 in upper left.


1/ 2

R1 :

1 3

3/ 2

2 5

2 3 4 3 4 5 The rst step: get a 1 in upper left.


1/ 2

R1 :

1 3

3/ 2

2 5

Now use the 1 to cancel the 3. 3R1 R2 :


+ 3/ 1 2 2 0 1/2 1

2 3 4 3 4 5 The rst step: get a 1 in upper left.


1/ 2

R1 :

1 3

3/ 2

2 5

Now use the 1 to cancel the 3. 3R1 R2 : Now get a 1 in the lower right. 2 R2 :
+ 3/ 1 2 2 0 1/2 1

1 0

3/ 2

2 1 2

2 3 4 3 4 5 The rst step: get a 1 in upper left.


1/ 2

R1 :

1 3

3/ 2

2 5

Now use the 1 to cancel the 3. 3R1 R2 : Now get a 1 in the lower right. 2 R2 :
+ 3/ 1 2 2 0 1/2 1

1 0

3/ 2

2 1 2

Finally use the 1 in the lower right to cancel the 3/2 . 3/2 R2 R1 :
+

1 0 1 0 1 2

Note that the goal is to get it looking like 1 0 0 1 .

Note that the goal is to get it looking like 1 0 0 1 .

We do it one column at a time, left to right.

Note that the goal is to get it looking like 1 0 0 1 .

We do it one column at a time, left to right. In each column, rst get the 1, then use the 1 to get the 0s.

Operations

You use the three operations , , to get what you want.

Operations

You use the three operations , , to get what you want. 1. To get a 1,

Operations

You use the three operations , , to get what you want. 1. To get a 1, usually;

Operations

You use the three operations , , to get what you want. 1. To get a 1, usually; sometimes.

Operations

You use the three operations , , to get what you want. 1. To get a 1, usually; sometimes. 2. to get a 0, use , using 1s to make 0s.
+

Example: Solve the system 2x + 2 y + 6 z x + y + 2z 2x + 3 y + z using matrix methods. = 1 = 1 = 2

Example: Solve the system 2x + 2 y + 6 z x + y + 2z 2x + 3 y + z using matrix methods. Solution: First translate the problem into matrix form. 2 2 6 1 1 1 2 1 . 2 3 1 2 = 1 = 1 = 2

2 2 6 1 1 1 2 1 . 2 2 3 1 Now we need a 1 in the upper left. We could use 1/2 R1 ,

2 2 6 1 1 1 2 1 . 2 2 3 1 Now we need a 1 in the upper left. We could use 1/2 R1 , but that creates a fraction, and there is an alternative.

2 2 6 1 1 1 2 1 . 2 2 3 1 Now we need a 1 in the upper left. We could use 1/2 R1 , but that creates a fraction, and there is 1 1 2 2 R1 R2 : 2 3 an alternative. 2 1 6 1 . 1 2

1 1 2 1 2 2 6 1 2 2 3 1 Now get 0s down the rest of the rst column.

1 1 2 1 2 2 6 1 2 2 3 1 Now get 0s down the rest of the rst 1 + 0 2R1 R2 : 2 column. 1 2 1 0 2 3 3 1 2

1 1 2 1 2 2 6 1 2 2 3 1 Now get 0s down the rest of the rst 1 + 0 2R1 R2 : 2 2R1 R3 :


+

column. 1 2 1 0 2 3 3 1 2

1 1 2 1 0 0 2 3 0 1 3 0

1 1 1 2 0 0 2 3 0 0 1 3 Now we need a 1 in the 2,2 position.

1 1 1 2 0 0 2 3 0 0 1 3 Now we need a 1 in the 2,2 position. Normally wed use , but that wont work here. So instead we use 1 1 2 1 0 1 3 0 . R2 R3 : 0 0 2 3

1 1 1 2 0 0 2 3 0 0 1 3 Now we need a 1 in the 2,2 position. Normally wed use , but that wont work here. So instead we use 1 1 2 1 0 1 3 0 . R2 R3 : 0 0 2 3 Now nish the second column: 1R2 R1 :
+

1 0 5 1 0 1 3 0 . 0 0 2 3

1 0 5 1 0 1 3 0 . 0 0 2 3 Finally the third column, where we need a 1 in the 3,3 position.

1 0 5 1 0 1 3 0 . 0 0 2 3 Finally the third column, where we need a 1 in the 3,3 position. 1 0 5 1 1/ 0 1 3 0 . R3 : 2 3 0 0 1 /2

1 1 0 5 0 1 3 0 3 0 0 1 /2 Then clear out the upper entries.

1 1 0 5 0 1 3 0 3 0 0 1 /2 Then clear out the upper entries. 3R3 R2 :


+

1 0 5 1 0 1 0 9/ 2 0 0 1 3/2

1 1 0 5 0 1 3 0 3 0 0 1 /2 Then clear out the upper entries. 3R3 R2 :


+

1 0 5 1 0 1 0 9/ 2 0 0 1 3/2 1 0 0 17/2 0 1 0 9/ 2 0 0 1 3/2

5R3 R1 :

1 1 0 5 0 1 3 0 3 0 0 1 /2 Then clear out the upper entries. 3R3 R2 :


+

1 0 5 1 0 1 0 9/ 2 0 0 1 3/2 1 0 0 17/2 0 1 0 9/ 2 0 0 1 3/2

5R3 R1 : So we get the solution x y z

= 17/2 = 9/2 = 3/2 .

Example: Here is a variation on the last problem. 2x + 2 y + 6 z x + y + 2z 2x + 2 y + z = 1 = 1 = 2

Example: Here is a variation on the last problem. 2x + 2 y + 6 z x + y + 2z 2x + 2 y + z Start o the same way. R1 R2 : 1 1 1 2 2 2 6 1 2 2 2 1 = 1 = 1 = 2

Example: Here is a variation on the last problem. 2x + 2 y + 6 z x + y + 2z 2x + 2 y + z Start o the same way. R1 R2 : 1 1 1 2 2 2 6 1 2 2 2 1 2R1 R2 :
+

= 1 = 1 = 2

1 1 2 1 0 0 2 3 2 2 1 2

Example: Here is a variation on the last problem. 2x + 2 y + 6 z x + y + 2z 2x + 2 y + z Start o the same way. R1 R2 : 1 1 1 2 2 2 6 1 2 2 2 1 2R1 R2 :
+

= 1 = 1 = 2

1 1 2 1 0 0 2 3 2 2 1 2 1 1 2 1 0 0 2 3 0 0 3 0

2R1 R3 :

1 1 2 1 0 0 2 3 0 0 3 0 Now we have a another problem.

1 1 2 1 0 0 2 3 0 0 3 0 Now we have a another problem. Theres no way to make progress on column 2.

1 1 2 1 0 0 2 3 0 0 3 0 Now we have a another problem. Theres no way to make progress on column 2. So we have no choice but to skip it and move on to column 3.

1 1 2 1 0 0 2 3 0 0 3 0 Now we have a another problem. Theres no way to make progress on column 2. So we have no choice but to skip it and move on to column 3. But where should the 1 go?

1 1 2 1 0 0 2 3 0 0 3 0 Now we have a another problem. Theres no way to make progress on column 2. So we have no choice but to skip it and move on to column 3. But where should the 1 go? Just below the previous 1 in column 1. 1 1 2 1 1/ 0 0 1 3/2 R2 : 2 0 0 3 0

1 1 2 1 0 0 2 3 0 0 3 0 Now we have a another problem. Theres no way to make progress on column 2. So we have no choice but to skip it and move on to column 3. But where should the 1 go? Just below the previous 1 in column 1. 1 1 2 1 1/ 0 0 1 3/2 R2 : 2 0 0 3 0 Those are called leading ones.

1 1 1 2 0 0 1 3/2 0 0 3 0 Now the 0s. 2R2 R1 :


+

1 1 0 4 0 0 1 3/2 0 0 3 0

1 1 1 2 0 0 1 3/2 0 0 3 0 Now the 0s. 2R2 R1 :


+

1 1 0 4 0 0 1 3/2 0 0 3 0 1 1 0 4 0 0 1 3/ 2 0 0 0 9/2

3R2 R3 :

1 1 0 4 0 0 1 3/ 2 0 0 0 9/2 Now were done with the row reduction, so we can translate back to equation form.

1 1 0 4 0 0 1 3/ 2 0 0 0 9/2 Now were done with the row reduction, so we can translate back to equation form. x +y = 4 3 z = /2 0 = 9/2

1 1 0 4 0 0 1 3/ 2 0 0 0 9/2 Now were done with the row reduction, so we can translate back to equation form. x +y = 4 3 z = /2 0 = 9/2

That last equation is trouble.

1 1 0 4 0 0 1 3/ 2 0 0 0 9/2 Now were done with the row reduction, so we can translate back to equation form. x +y = 4 3 z = /2 0 = 9/2

That last equation is trouble. Conclusion: The system has no solution.

Suppose, hypothetically, that 9/2 had been 0 instead. 4 1 1 0 0 0 1 3/ 2 0 0 0 0

Suppose, hypothetically, that 9/2 had been 0 instead. 4 1 1 0 0 0 1 3/ 2 0 0 0 0 Then we get x +y = 4 z = 3/2 0 = 0

and the third equation is OK.

Suppose, hypothetically, that 9/2 had been 0 instead. 4 1 1 0 0 0 1 3/ 2 0 0 0 0 Then we get x +y = 4 z = 3/2 0 = 0

and the third equation is OK. On the other hand, it doesnt tell us anything useful, so lets ignore it.

x +y z

= 4 = 3/2

Note that we now have more variables than equations. That usually means innitely many solutions.

x +y z

= 4 = 3/2

Note that we now have more variables than equations. That usually means innitely many solutions. Solve for the leading variables. x z = 4y = 3/2

x +y z

= 4 = 3/2

Note that we now have more variables than equations. That usually means innitely many solutions. Solve for the leading variables. x z = 4y = 3/2

We say that y is arbitrary or free. It can take any value you want.

x +y z

= 4 = 3/2

Note that we now have more variables than equations. That usually means innitely many solutions. Solve for the leading variables. x z = 4y = 3/2

We say that y is arbitrary or free. It can take any value you want. But once you decide on a value for y , then x and z are determined by the equations.

x z For example, suppose y = 17.

= 4y = 3/2

x z

= 4y = 3/2

For example, suppose y = 17. Then x = 4 17 = 13

x z

= 4y = 3/2

For example, suppose y = 17. Then x = 4 17 = 13 giving the complete solution x y z = 13 = 17 3 = /2 .

Row Reduced Form


To recap the last example: Sometimes you cannot achieve your 1 0 0 0 1 0 0 0 1 goal of .

Row Reduced Form


To recap the last example: Sometimes you cannot achieve your 1 0 0 0 1 0 0 0 1 goal of .

In that case you have to settle for row-reduced form.

Row Reduced Form


To recap the last example: Sometimes you cannot achieve your 1 0 0 0 1 0 0 0 1 goal of .

In that case you have to settle for row-reduced form. 1. Each row has a leading 1; 2. every column with a leading 1 has 0s elsewhere; 3. leading 1s do not zigzag.

Row-reduced: 1 0 2 3 0 1 4 5 1 2 0 3 0 0 1 4 1 2 3 4

Row-reduced: 1 0 2 3 0 1 4 5 1 2 0 3 0 0 1 4 1 2 3 4 Not row-reduced: 0 1 4 5 1 0 2 3

Row-reduced: 1 0 2 3 0 1 4 5 1 2 0 3 0 0 1 4 1 2 3 4 Not row-reduced: 0 1 4 5 1 0 2 3 1 1 0 0 0 1 0 1

Row-reduced: 1 0 2 3 0 1 4 5 1 2 0 3 0 0 1 4 1 2 3 4 Not row-reduced: 0 1 4 5 1 0 2 3 1 1 0 0 0 1 0 1 1 0 2 3 0 2 4 5

Having gotten your matrx in row-reduced form: 1. check for nonsense like 0 = 5, which means no solution; 2. if the equations all look good, then solve for the leading variables; 3. the remaining variables (now on the right) are free.

Chapter 1: Matrices

A matrix is a rectangular array of numbers. 1 2 3 4 5 6

Chapter 1: Matrices

A matrix is a rectangular array of numbers. 1 2 3 4 5 6 The size of a matrix is described by the number of rows and columns. These numbers are called the dimensions of the matrix. (Rows are horizontal; columns are vertical.) This matrix is 2 3. (# rows # columns)

Matrices are usually denoted by capital letters. A= 1 2 3 4 5 6

Matrices are usually denoted by capital letters. A= 1 2 3 4 5 6

Individual entries in a matrix are denoted by the corresponding lower-case letter, with subscripts to indicate the position. a1,1 = 1 a1,2 = 2 a1,3 = 3 a2,1 = 4 a2,2 = 5 a2,3 = 6

Matrices are usually denoted by capital letters. A= 1 2 3 4 5 6

Individual entries in a matrix are denoted by the corresponding lower-case letter, with subscripts to indicate the position. a1,1 = 1 a1,2 = 2 a1,3 = 3 a2,1 = 4 a2,2 = 5 a2,3 = 6 The commas can be omitted from the subscripts if it doesnt cause confusion. a11 = 1 a12 = 2 a13 = 3 a21 = 4 a22 = 5 a23 = 6

A vector is a skinny matrix: a matrix with only 1 row or only 1 column. row vector: [1 2 3] 1 column vector: 2 3

Matrix Operations: +,

Total no-brainers.

Example: 1 4 2 5 3 6 1 1 1 7 3 4 =

Matrix Operations: +,

Total no-brainers.

Example: 1 4 2 5 3 6 1 1 1 7 3 4 = 2 3 1 4 2 10

Matrix Operations: +,

Total no-brainers.

Example: 1 4 2 5 3 6 1 1 1 7 3 4 = 2 3 1 4 2 10

If the dimensions dont match, the result is undened. Example: 1 4 2 5 + 1 1 1 7 3 4 = undened

Matrix Operations: Scalar

There are 2 kinds of matrix multiplication, this is the easy one. Example: 3 1 2 3 4 =

Matrix Operations: Scalar

There are 2 kinds of matrix multiplication, this is the easy one. Example: 3 1 2 3 4 = 3 6 9 12

Matrix Operations: Scalar

There are 2 kinds of matrix multiplication, this is the easy one. Example: 3 1 2 3 4 = 3 6 9 12

An ordinary number can be called a scalar. Scalar multiplication is always dened.

Linear Combinations
Its common to combine addition and scalar multiplication. Example: Suppose that A= What is 2A 3B ? 2 1 0 3 B= 3 5 2 1 .

Linear Combinations
Its common to combine addition and scalar multiplication. Example: Suppose that A= What is 2A 3B ? Solution: 2A 3B = 2 2 1 0 3 3 3 5 2 1 2 1 0 3 B= 3 5 2 1 .

Linear Combinations
Its common to combine addition and scalar multiplication. Example: Suppose that A= What is 2A 3B ? Solution: 2A 3B = 2 2 1 4 2 0 3 0 6 3 3 5 2 1 9 15 6 3 2 1 0 3 B= 3 5 2 1 .

Linear Combinations
Its common to combine addition and scalar multiplication. Example: Suppose that A= What is 2A 3B ? Solution: 2A 3B = 2 2 1 4 2 0 3 0 6 3 3 5 2 1 9 15 6 3 . 2 1 0 3 B= 3 5 2 1 .

13 15 8 9

Example: Suppose that A= What is 2A 3B ? Alternate Solution: 2A 3B = 2 2 1 0 3 + 3 3 5 2 1 2 1 0 3 B= 3 5 2 1 .

Example: Suppose that A= What is 2A 3B ? Alternate Solution: 2A 3B = 2 2 1 4 2 0 3 0 6 + 3 3 5 2 1 9 15 6 3 2 1 0 3 B= 3 5 2 1 .

Example: Suppose that A= What is 2A 3B ? Alternate Solution: 2A 3B = 2 2 1 4 2 0 3 0 6 + 3 3 5 2 1 9 15 6 3 . 2 1 0 3 B= 3 5 2 1 .

+ 13 15 8 9

Matrix Multiplication
This one is confusing.

Matrix Multiplication
This one is confusing.

Imagine that vectors are cannons, and numbers are cannon balls.

Matrix Multiplication

This one is confusing.

Imagine that vectors are cannons, and numbers are cannon balls. Think of this problem 2 1 2

Matrix Multiplication
This one is confusing.

Imagine that vectors are cannons, and numbers are cannon balls. like this: 2 1 2 4 2 3 .

Matrix Multiplication
This one is confusing.

Imagine that vectors are cannons, and numbers are cannon balls. like this: 2 1 2 4 2 3 .

1 2

2 + 6

2 + 6

8 2 + 6

Thus, 4 2 3 2 1 = 2 4

Thus, 4 Example: 3 1 0 1 5 = ? 8 2 3 2 1 = 2 4

Thus, 4 Example: 3 1 0 1 5 = ? 8 2 3 2 1 = 2 4

Solution: 1 5 8 3 1 0

Thus, 4 Example: 3 1 0 1 5 = ? 8 2 3 2 1 = 2 4

Solution: 1 5 8 3 1 1(5) + 0 8

3 1

Thus, 4 Example: 3 1 0 1 5 = ? 8 2 3 2 1 = 2 4

Solution: 1 5 8 3 1 1(5) + 0 8

3 1

Note that both vectors must be of the same length! Example: 1 2 4 5 = Undened 6

The same idea works for double barreled cannons. 1 2 3 4 2 3

Fire! 1 9 2 4

Fire! 2 4 11

Reload. 2 4 2 3 11

Fire, and re again.

11 16

So we get 2 3 1 2 3 4 = 11 16 .

In fact this works for almost any sizes of matrix: The only restriction is that the length of the rows (2nd dimension) in the left matrix = the length of the columns (1st dimension) in the right matrix.

In fact this works for almost any sizes of matrix: The only restriction is that the length of the rows (2nd dimension) in the left matrix = the length of the columns (1st dimension) in the right matrix. Example: 4 5 1 2 3 6 7 8 9 Can we multiply a 1 3 matrix by a 3 2 matrix?

In fact this works for almost any sizes of matrix: The only restriction is that the length of the rows (2nd dimension) in the left matrix = the length of the columns (1st dimension) in the right matrix. Example: 4 5 1 2 3 6 7 8 9 Can we multiply a 1 3 matrix by a 3 2 matrix? Yes, because the 2nd dimension of the rst = the 1st dimension of the second.

In fact this works for almost any sizes of matrix: The only restriction is that the length of the rows (2nd dimension) in the left matrix = the length of the columns (1st dimension) in the right matrix. Example: 4 5 1 2 3 6 7 8 9 Can we multiply a 1 3 matrix by a 3 2 matrix? Yes, because the 2nd dimension of the rst = the 1st dimension of the second. Moreover the result is 1 2. (1 3)(3 2) = (1 3)( 3 2) = 1 2

If both dimensions are the same we say the matrix is square. 2 1 4 3 2 3 0 1

First row on left against rst column on right. 2 1 4 3 2 3 0 1

Fire both rounds. 1 3 0 1 16

Second row on left against second column on right. 1 3 16 0 1

Fire both rounds.

16 3

Reload. 2 1 4 3 2 3 0 1 16 3

First row on left against second column on right. 2 1 4 3 16 3

2 3 0 1

Fire. 2 4 0 1 16 11 3

Second row on left against rst column on right. 2 4 16 0 1

11 3

Fire.

16 11 4 3

Fire.

So 2 3 0 1 2 1 4 3

16 11 4 3 16 11 4 3

Example: 3 1 1 2 1 5 3 2 =?

Example: 3 1 1 2 Solution: 1 5 3 2 3 1 1 2 6 17 5 1 1 5 3 2 =?

Example: 3 1 1 2 Solution: 1 5 3 2 3 1 1 2 Note: 3 1 3 1 (1) + (5) + 1 2 1 2 (3) = (2) = 6 5 17 1 6 17 5 1 1 5 3 2 =?

So the each column in the answer is a linear combination of the columns in the left factor.

Matrix multiplication is not commutative!

Matrix multiplication is not commutative! This means that usually AB = BA.

Matrix multiplication is not commutative! This means that usually AB = BA. This is very unlike ordinary arithmetic. You must take care with the order in which matrices are multiplied!

Special Matrices

In ordinary arithmetic there are two special numbers: 0, the additive identity, for which 0 + x = x;

and 1, the multiplicative identity, for which 1 x = x.

There are special matrices with the same properties: the zero matrix 0 with the property that 0 + X = X;

and the identity matrix I with the property that IX =X and X I = X .

All the entries of the zero matrix 0 are 0.

All the entries of the zero matrix 0 are 0.

Example: Suppose that X = 1 2 3 4 5 6 .

Since X is 2 3, the corresponding zero matrix should be 0= 0 0 0 0 0 0 .

All the entries of the zero matrix 0 are 0.

Example: Suppose that X = 1 2 3 4 5 6 .

Since X is 2 3, the corresponding zero matrix should be 0= Then 0 + X = X + 0 = X. 0 0 0 0 0 0 .

The identity matrix is trickier. It also comes in dierent sizes, but is always square. I1 = I2 = 1 , ,

1 0 0 1

I3 etc.

1 0 0 = 0 1 0 , 0 0 1

Example: Suppose that X = Then I2 X = X (2 2) (2 3) and X I3 = X. (2 3) (3 3) 1 2 3 4 5 6 .

Notice that we need a dierent sized I on each side of X .

Example: Suppose that X = Then I2 X = X (2 2) (2 3) and X I3 = X. (2 3) (3 3) 1 2 3 4 5 6 .

Notice that we need a dierent sized I on each side of X . But since the size of I is determined by its neighbor, we can omit the subscript. IX = X and X I = X .

Matrix Inverses

In ordinary algebra the multiplicative inverse is used constantly. For example, 2x = 6 1 1 2 2x = 2 6 1x = 3 x = 3. Wouldnt it be nice if we had the same for matrices?

1 2 3 4

x y

5 6

1 2 3 4 2
3 2

x y x y

5 6 2
3 2

1 1 2

1 2 3 4

1 1 2

5 6

1 2 3 4 2
3 2

x y x y x y

5 6 2
3 2

1 1 2

1 2 3 4 1 0 0 1

1 1 2

5 6

4
9 2

1 2 3 4 2
3 2

x y x y x y x y

5 6 2
3 2

1 1 2

1 2 3 4 1 0 0 1

1 1 2

5 6

4
9 2

4
9 2

2
3 2

1 1 2

is called the inverse matrix of 2


3 2

1 2 3 4 1 0 0 1 1 0 0 1

because

1 1 2 2
3 2

1 2 3 4 1 1 2

and 1 2 3 4 = .

2
3 2

1 1 2

is called the inverse matrix of 2


3 2

1 2 3 4 1 0 0 1 1 0 0 1

because

1 1 2 2
3 2

1 2 3 4 1 1 2

and 1 2 3 4 As shorthand we can write 1 2 3 4


1

2
3 2

1 1 2

2
3 2

1 1 2

is called the inverse matrix of 2


3 2

1 2 3 4 1 0 0 1 1 0 0 1

because

1 1 2 2
3 2

1 2 3 4 1 1 2

and 1 2 3 4 As shorthand we can write 1 2 3 4 Note that also 2


3 2 1

2
3 2

1 1 2

1
1 2

1 2 3 4

In general, if AB = I = BA then we say B is the inverse of A. Almost every square matrix has an inverse. Only square matrices can have inverses, but not every one does.

In general, if AB = I = BA then we say B is the inverse of A. Almost every square matrix has an inverse. Only square matrices can have inverses, but not every one does. If a matrix does not have an inverse, we say it is singular or Examples: 1 1 2 1 2 3 1 0 4 , , , 2 4 4 5 6 0 0 7 noninvertible. 2 3 5 6 8 9

How do you calculate the inverse of a matrix? Row reduction.

How do you calculate the inverse of a matrix? Row reduction. Each of those row operations are equivalent to a multiplication. 3 0 0 0 1 0 3 R1 : 0 0 1

How do you calculate the inverse of a matrix? Row reduction. Each of those row operations are equivalent to a multiplication. 3 0 0 0 1 0 3 R1 : 0 0 1 R2 R3 : 1 0 0 0 0 1 0 1 0

How do you calculate the inverse of a matrix? Row reduction. Each of those row operations are equivalent to a multiplication. 3 0 0 0 1 0 3 R1 : 0 0 1 R2 R3 : 1 0 0 0 0 1 0 1 0 5R2 R3 :
+

1 0 0 0 1 0 0 5 1

So row reduction is equivalent to multiplication. That means if 1 2 3 1 0 0 RR 4 5 6 0 1 0 7 8 9 0 0 1

So row reduction is equivalent to multiplication. That means if 1 2 3 1 0 0 RR 4 5 6 0 1 0 7 8 9 0 0 1 then then same result can be gotten by 1 0 M7 M6 M5 M4 M3 M2 M1 2 1 1 0 matrix multiplication: 1 1 0 0 2 = 0 1 0 1 0 0 1

with the M s equivalent to the row reduction.

So row reduction is equivalent to multiplication. That means if 1 2 3 1 0 0 RR 4 5 6 0 1 0 7 8 9 0 0 1 then then same result can be gotten by 1 0 M7 M6 M5 M4 M3 M2 M1 2 1 1 0 matrix multiplication: 1 1 0 0 2 = 0 1 0 1 0 0 1

with the M s equivalent to the row reduction. Then 1 1 0 1 2 1 2 = M7 M6 M5 M4 M3 M2 M1 . 1 0 1

The only problem is: how do we keep track of the M s while doing the row reduction? Simple trick: Row reduce 1 0 2 1 1 0 instead. Then the right side keeps track of the row operations performed on the left side. 1 2 1 1 0 0 0 1 0 0 0 1

Calculating the Matrix Inverse


If A is a square matrix, then to calculate its inverse: 1. Set up the augmented matrix A I .

2. Row reduce. 3. If you succeed in getting I B then B = A1 . 4. If you fail to get the identity on the left, then A is not invertible.

Example: Find the matrix inverse of A= 1 2 3 4 .

Example: Find the matrix inverse of A= 1 2 3 4 .

Solution: Start o with the augmented matrix 1 2 1 0 3 4 0 1 then row reduce.

1 2 1 0 3 4 0 1 3R1 R2 :
+

1 2 1 0 0 2 3 1

1 2 1 0 3 4 0 1 3R1 R2 :
+

1 2 1 0 0 2 3 1 1 2 0 1 1
3/ 2

1/2 R2 :

0 1/2

1 2 1 0 3 4 0 1 3R1 R2 :
+

1 2 1 0 0 2 3 1 1 2 0 1 1
3/ 2

1/2 R2 :
+

0 1/2

2R2 R1 :

1 0 2 1 0 1 3/2 1/2

1 2 1 0 3 4 0 1 3R1 R2 :
+

1 2 1 0 0 2 3 1 1 2 0 1 1
3/ 2

1/2 R2 :
+

0 1/2

2R2 R1 : So A1 =

1 0 2 1 0 1 3/2 1/2 2 1 3/ 1/ 2 2

1 2 1 0 3 4 0 1 3R1 R2 :
+

1 2 1 0 0 2 3 1 1 2 0 1 1
3/ 2

1/2 R2 :
+

0 1/2

2R2 R1 : So A1 =

1 0 2 1 0 1 3/2 1/2 2 1 3/ 1/ 2 2

You can check your answer: is AA1 = I?

Solving Systems with Inverses

Note that the matrix equation 1 2 3 x 2 4 5 6 y = 3 7 8 9 z 6 is equivalent to the system of equations x + 2y + 3z 4x + 5y + 6z 7x + 8y + 9z = 3 = 6 = 9.

So matrix inverses can also be used to solve systems.

Example: Solve the following system. 2x + z y + 2z x +z = 2 = 1 = 3.

Example: Solve the following system. 2x + z y + 2z x +z Solution: First translate the 2 0 0 1 1 0 = 2 = 1 = 3.

system into matrix equation form. 1 x 2 2 y = 1 1 z 3

2 0 1 x 2 0 1 2 y = 1 1 0 1 z 3 Now compute the inverse matrix. 2 0 1 1 0 0 0 1 2 0 1 0 1 0 1 0 0 1

2 0 1 x 2 0 1 2 y = 1 1 0 1 z 3 Now compute the inverse matrix. 2 0 1 1 0 0 0 1 2 0 1 0 1 0 1 0 0 1 R1 R3 1 0 1 0 0 1 0 1 2 0 1 0 2 0 1 1 0 0

2 0 1 x 2 0 1 2 y = 1 1 0 1 z 3 Now compute the inverse matrix. 2 0 1 1 0 0 0 1 2 0 1 0 1 0 1 0 0 1 R1 R3 1 0 1 0 0 1 0 1 2 0 1 0 2 0 1 1 0 0 2R1 R3


+

1 0 1 0 0 1 0 1 2 0 1 0 0 0 1 1 0 2

1 0 1 0 0 1 0 1 2 0 1 0 0 0 1 1 0 2

1 0 1 0 0 1 0 1 2 0 1 0 0 0 1 1 0 2

1 R2

1 0 1 0 0 1 0 1 2 0 1 0 0 0 1 1 0 2

1 0 1 0 0 1 0 1 2 0 1 0 0 0 1 1 0 2

1 R2

1 0 1 0 0 1 0 1 2 0 1 0 0 0 1 1 0 2 1 0 1 0 0 1 0 1 2 0 1 0 0 0 1 1 0 2

1 R3

1 0 1 0 0 1 0 1 2 0 1 0 0 0 1 1 0 2

1 R2

1 0 1 0 0 1 0 1 2 0 1 0 0 0 1 1 0 2 1 0 1 0 0 1 0 1 2 0 1 0 0 0 1 1 0 2 1 0 0 1 0 1 0 1 2 0 1 0 0 0 1 1 0 2

1 R3

1R3 R1

1 0 1 0 0 1 0 1 2 0 1 0 0 0 1 1 0 2

1 R2

1 0 1 0 0 1 0 1 2 0 1 0 0 0 1 1 0 2 1 0 1 0 0 1 0 1 2 0 1 0 0 0 1 1 0 2 1 0 0 1 0 1 0 1 2 0 1 0 0 0 1 1 0 2 1 0 0 1 0 1 0 1 0 2 1 4 0 0 1 1 0 2

1 R3

1R3 R1

2R3 R2

So the inverse is

1 0 1 2 1 4 . 1 0 2

Now we can solve the matrix equation. 1 0 1 2 0 1 x 1 0 1 2 2 1 4 0 1 2 y = 2 1 4 1 1 0 2 1 0 1 z 1 0 2 3

So the inverse is

1 0 1 2 1 4 . 1 0 2

Now we can solve the matrix equation. 1 0 1 2 0 1 x 1 0 1 2 2 1 4 0 1 2 y = 2 1 4 1 1 0 2 1 0 1 z 1 0 2 3 x 1 y = 9 z 4

So the inverse is

1 0 1 2 1 4 . 1 0 2

Now we can solve the matrix equation. 1 0 1 2 0 1 x 1 0 1 2 2 1 4 0 1 2 y = 2 1 4 1 1 0 2 1 0 1 z 1 0 2 3 x 1 y = 9 z 4 So the solution to the original system is x y z = 1 = 9 = 4.

Example: Solve the following system. 2x + z y + 2z x +z = 3 = 2 = 1.

Example: Solve the following system. 2x + z y + 2z x +z Solution: First translate the 2 0 0 1 1 0 = 3 = 2 = 1.

system into matrix equation form. 1 x 3 2 y = 2 1 z 1

Example: Solve the following system. 2x + z y + 2z x +z Solution: First translate the 2 0 0 1 1 0 = 3 = 2 = 1.

system into matrix equation form. 1 x 3 2 y = 2 1 z 1

Now reuse the inverse from the last example.

1 0 1 2 0 1 x 1 0 1 3 2 1 4 0 1 2 y 2 1 4 2 = 1 0 2 1 0 1 z 1 0 2 1

1 0 1 2 0 1 x 1 0 1 3 2 1 4 0 1 2 y 2 1 4 2 = 1 0 2 1 0 1 z 1 0 2 1 x 2 y = 4 z 1

1 0 1 2 0 1 x 1 0 1 3 2 1 4 0 1 2 y 2 1 4 2 = 1 0 2 1 0 1 z 1 0 2 1 x 2 y = 4 z 1 So the solution to the system is x y z = 2 = 4 = 1.

Augmented matrix versus Matrix Equation

A system can be solved either by 1. converting it to an augmented matrix and row reducing; or 2. converting it to a matrix equation and using a matrix inverse.

Augmented matrix versus Matrix Equation

A system can be solved either by 1. converting it to an augmented matrix and row reducing; or 2. converting it to a matrix equation and using a matrix inverse. Method 1 is usually easier, and its more general: it even handles cases where there are innitely many solutions.

Augmented matrix versus Matrix Equation

A system can be solved either by 1. converting it to an augmented matrix and row reducing; or 2. converting it to a matrix equation and using a matrix inverse. Method 1 is usually easier, and its more general: it even handles cases where there are innitely many solutions. Method 2 only makes sense if youll reuse the inverse.

Recipes

Recipes
Example: Suppose that a recipe for a dozen biscuits calls for 4 cups of our and 2 eggs; a recipe for a dozen pancakes calls for 5 cups of our and 1 egg. Now suppose you need to make 2 dozen biscuits and 3 dozen pancakes. How much our and how many eggs do you need?

Recipes
Example: Suppose that a recipe for a dozen biscuits calls for 4 cups of our and 2 eggs; a recipe for a dozen pancakes calls for 5 cups of our and 1 egg. Now suppose you need to make 2 dozen biscuits and 3 dozen pancakes. How much our and how many eggs do you need? Solution: Dene R1 = 4 2 R2 = 5 1 .

Recipes
Example: Suppose that a recipe for a dozen biscuits calls for 4 cups of our and 2 eggs; a recipe for a dozen pancakes calls for 5 cups of our and 1 egg. Now suppose you need to make 2 dozen biscuits and 3 dozen pancakes. How much our and how many eggs do you need? Solution: Dene R1 = 4 2 R2 = 5 1 .

Then the our and eggs required are given by 2R1 + 3R2 = 2 4 2 +3 5 1 .

Recipes
Example: Suppose that a recipe for a dozen biscuits calls for 4 cups of our and 2 eggs; a recipe for a dozen pancakes calls for 5 cups of our and 1 egg. Now suppose you need to make 2 dozen biscuits and 3 dozen pancakes. How much our and how many eggs do you need? Solution: Dene R1 = 4 2 R2 = 5 1 .

Then the our and eggs required are given by 2R1 + 3R2 = 2 This can also be expressed as 4 5 2 1 2 3 . 4 2 +3 5 1 .

A= X =

4 5 2 1 2 3

is called the production matrix. is called the production schedule.

R denotes the matrix of required our and eggs, the resource vector. The fundamental equation relating these is R = AX .

A= X =

4 5 2 1 2 3

is called the production matrix. is called the production schedule.

R denotes the matrix of required our and eggs, the resource vector. The fundamental equation relating these is R = AX . In our case, R = AX = 4 5 2 1 2 3 = 23 7 .

A= X =

4 5 2 1 2 3

is called the production matrix. is called the production schedule.

R denotes the matrix of required our and eggs, the resource vector. The fundamental equation relating these is R = AX . In our case, R = AX = 4 5 2 1 2 3 = 23 7 .

This is called a linear production model.

Note in our problem R = r1 r2 x1 x2 where r 1 = our in cups r 2 = eggs x 1 = biscuits in dozens x 2 = pancakes in dozens.

where

Note in our problem R = r1 r2 x1 x2 where r 1 = our in cups r 2 = eggs x 1 = biscuits in dozens x 2 = pancakes in dozens.

where

What would be dierent if biscuits and pancakes were counted individually?

R =

r1 r2 x1 x2
4/ 12 2/ 12

where

r 1 = our in cups r 2 = eggs x 1 = # biscuits x 2 = # pancakes


1/ 3 1/ 6 5/ 12 1/ 12

where

A=

5/ 12 1/ 12

R =

r1 r2 x1 x2
4/ 12 2/ 12

where

r 1 = our in cups r 2 = eggs x 1 = # biscuits x 2 = # pancakes


1/ 3 1/ 6 5/ 12 1/ 12

where

A=

5/ 12 1/ 12

What if eggs were in dozens and our in kilograms? (Assume 1 cup our is 125 grams.)

R =

r1 r2 x1 x2

where

r 1 = our in kg. r 2 = eggs in dozens x 1 = # biscuits x 2 = # pancakes .042 .052 1/ 1/ 72 144

where

A=

.125 1/3 .125 5/12 1/ 1/ 1/ 1/ 12 6 12 12

Example: Solve for X in 1 2 3 4 5 6

x1 x2

10 = 24 . 37

Example: Solve for X in 1 2 3 4 5 6 Solution: Row reducing

x1 x2

10 = 24 . 37

1 2 10 3 4 24 5 6 37 yields 1 0 0 0 1 0 0 0 1

Example: Solve for X in 1 2 3 4 5 6 Solution: Row reducing

x1 x2

10 = 24 . 37

1 2 10 3 4 24 5 6 37 yields 1 0 0 0 1 0 0 0 1 which translates into x1 = 0 x2 = 0 0 = 1

which has no solution.

Example: Solve for X in 1 2 3 4 5 6

x1 x2

10 = 24 . 38

Example: Solve for X in 1 2 3 4 5 6

x1 x2

10 = 24 . 38

Solution: Row reduction yields 1 0 4 0 1 3 , 0 0 0

Example: Solve for X in 1 2 3 4 5 6

x1 x2

10 = 24 . 38

Solution: Row reduction yields 1 0 4 0 1 3 , 0 0 0 which translates into x1 x2 0 = 4 = 3 = 0.

Example: Solve for X in 1 2 2 4 x1 x2 = 3 6 .

Example: Solve for X in 1 2 2 4 Solution: Row reducing 3 1 2 2 4 6 yields 1 2 0 0 3 0 , x1 x2 = 3 6 .

Example: Solve for X in 1 2 2 4 Solution: Row reducing 3 1 2 2 4 6 yields 1 2 0 0 which translates into 1x 1 2x 2 = 3 0 + 0 = 0. 3 0 , x1 x2 = 3 6 .

Example: Solve for X in 1 2 2 4 Solution: Row reducing 3 1 2 2 4 6 yields 1 2 0 0 which translates into 1x 1 2x 2 = 3 0 + 0 = 0. Solving the rst equation for x 1 gives x 1 = 2x 2 + 3 and we interpret x 2 as a free parameter. 3 0 , x1 x2 = 3 6 .

x 1 = 2x 2 + 3 x 2 = free A free parameter is a variable that can take any value.

x 1 = 2x 2 + 3 x 2 = free A free parameter is a variable that can take any value. For example, suppose x 2 = 17. Then x 1 = 2(17) + 3 = 31. And that is a valid solution to the initial problem: 1x 1 2x 1 + 2x 2 = 3 4x 2 = 6

x 1 = 2x 2 + 3 x 2 = free A free parameter is a variable that can take any value. For example, suppose x 2 = 17. Then x 1 = 2(17) + 3 = 31. And that is a valid solution to the initial problem: 1x 1 2x 1 + 2x 2 = 3 4x 2 = 6

1 (31) 2 (17) = 3 2 (31) + 4 (17) = 6.

Example: Solve for X in 1 2 2 4 x1 x2 = 3 6 .

Example: Solve for X in 1 2 2 4 x1 x2 = 3 6 .

Solution: Row reducing 1 2 2 4 yields 1 2 0 0 0 1 , 3 6

Example: Solve for X in 1 2 2 4 x1 x2 = 3 6 .

Solution: Row reducing 1 2 2 4 yields 1 2 0 0 which translates into 1x 1 2x 2 = 0 0 + 0 = 1, 0 1 , 3 6

Example: Solve for X in 1 2 2 4 x1 x2 = 3 6 .

Solution: Row reducing 1 2 2 4 yields 1 2 0 0 which translates into 1x 1 2x 2 = 0 0 + 0 = 1, which has no solution. 0 1 , 3 6

Example: Solve for X in 1 2 3 4 5 6 x1 x2 = x3 21 24

Example: Solve for X in 1 2 3 4 5 6 x1 x2 = x3 21 24

Solution: Row reducing 1 2 3 21 4 5 6 24 yields 1 0 0 1 19 1 2 20 ,

Example: Solve for X in 1 2 3 4 5 6 x1 x2 = x3 21 24

Solution: Row reducing 1 2 3 21 4 5 6 24 yields 1 0 which translates into x1 x2 x3 = + 2x 3 = 19 20. 0 1 19 1 2 20 ,

x1

x3 = x 2 + 2x 3 =

19 20.

Here we will solve for x 1 and x 2 x1 = x 3 19 x 2 = 2x 3 + 20 and interpret x 3 as a free variable.

x1

x3 = x 2 + 2x 3 =

19 20.

Here we will solve for x 1 and x 2 x1 = x 3 19 x 2 = 2x 3 + 20 and interpret x 3 as a free variable. For example, if x 3 = 5 then x1 = 5 19 = 14 x 2 = 2(5) + 20 = 10 x3 = 5 is a solution.

Number of Solutions

General Observations:

Number of Solutions

General Observations: 1. zeros on left & nonzero on right = no solution; otherwise

Number of Solutions

General Observations: 1. zeros on left & nonzero on right = no solution; otherwise 2. #rows = #variables = unique solution;

Number of Solutions

General Observations: 1. zeros on left & nonzero on right = no solution; otherwise 2. #rows = #variables = unique solution; 3. #rows < #variables = innitely many solutions.

Row reduction produces reduced 1 0 0 RREF is the closest to I. RREF satises the following.

row echelon form. 0 4 1 3 . 0 0

Row reduction produces reduced 1 0 0 RREF is the closest to I. RREF satises the following. 1. Each row has a leading 1;

row echelon form. 0 4 1 3 . 0 0

Row reduction produces reduced 1 0 0 RREF is the closest to I. RREF satises the following. 1. Each row has a leading 1;

row echelon form. 0 4 1 3 . 0 0

2. each column with a leading 1 has 0s elsewhere;

Row reduction produces reduced 1 0 0 RREF is the closest to I. RREF satises the following. 1. Each row has a leading 1;

row echelon form. 0 4 1 3 . 0 0

2. each column with a leading 1 has 0s elsewhere; 3. the leading 1s form a staircase (no zigzags).

Leontiev Models
Now we apply linear production models to self-sustaining economies. Think of an Amish oat farmer who plows with horses.

Leontiev Models
Now we apply linear production models to self-sustaining economies. Think of an Amish oat farmer who plows with horses. He uses horse manure to fertilize his oat elds, and oats to feed his horses.

Leontiev Models
Now we apply linear production models to self-sustaining economies. Think of an Amish oat farmer who plows with horses. He uses horse manure to fertilize his oat elds, and oats to feed his horses. Suppose that it takes 1/10 ton of oats and half a ton of manure to grow one ton of oats.

Leontiev Models
Now we apply linear production models to self-sustaining economies. Think of an Amish oat farmer who plows with horses. He uses horse manure to fertilize his oat elds, and oats to feed his horses. Suppose that it takes 1/10 ton of oats and half a ton of manure to grow one ton of oats. And it takes one ton of oats to produce a ton of manure. Then if x1 = x2 = we get A= 0.1 1 0.5 0 . # tons of oats # tons of manure

x1 = x2 =

# tons of oats # tons of manure Outputs Oats Manure

Inputs

Oats Manure

0.1 0.5

1 0

A=

0.1 1 0.5 0

So if X = then the resources consumed are R = AX = 0.1 1 0.5 0 x1 x2 . x1 x2

So if X = then the resources consumed are R = AX = 0.1 1 0.5 0 x1 x2 . x1 x2

Now the farmer would like to sell what he can: excess oats to Quaker, excess manure to other farmers.

How much does he have at the end of the harvest that doesnt need himself for next year?

How much does he have at the end of the harvest that doesnt need himself for next year? D = X AX D is called the demand vector and denotes the excess of each product that can be sold to meet external D emand.

How much does he have at the end of the harvest that doesnt need himself for next year? D = X AX D is called the demand vector and denotes the excess of each product that can be sold to meet external D emand. The equation above is the fundamental one for Leontiev theory.

The Fundamental Equation of Leontiev Theory

This equation can be written in three dierent ways, and each has its own uses. D = X AX (1)

The Fundamental Equation of Leontiev Theory

This equation can be written in three dierent ways, and each has its own uses. D = X AX D = (I A)X (1) (2)

The Fundamental Equation of Leontiev Theory

This equation can be written in three dierent ways, and each has its own uses. D = X AX D = (I A)X (I A)1 = X (1) (2) (3)

The Fundamental Equation of Leontiev Theory

This equation can be written in three dierent ways, and each has its own uses. D = X AX D = (I A)X (I A)1 = X 1. Form (1) is easy to understand; 2. form (2) can be solved for X using row reduction; 3. form (3) has been solved using a matrix inverse. (1) (2) (3)

Back to the Amish oat farmer: IA= 1 0 0 1 0.1 1 0.5 0 = 0.9 1 0.5 1 .

Back to the Amish oat farmer: IA= 1 0 0 1 0.1 1 0.5 0 = 0.9 1 0.5 1 .

Suppose the farmer is currently growing 100 tons of oats and producing 60 tons of horse manure. How much will he be able to sell?

Back to the Amish oat farmer: IA= 1 0 0 1 0.1 1 0.5 0 = 0.9 1 0.5 1 .

Suppose the farmer is currently growing 100 tons of oats and producing 60 tons of horse manure. How much will he be able to sell? X = 100 60 ,

Back to the Amish oat farmer: IA= 1 0 0 1 0.1 1 0.5 0 = 0.9 1 0.5 1 .

Suppose the farmer is currently growing 100 tons of oats and producing 60 tons of horse manure. How much will he be able to sell? X = 100 60 ,

D = (I A)X = 0.9 1 0.5 1 100 60

Back to the Amish oat farmer: IA= 1 0 0 1 0.1 1 0.5 0 = 0.9 1 0.5 1 .

Suppose the farmer is currently growing 100 tons of oats and producing 60 tons of horse manure. How much will he be able to sell? X = 100 60 ,

D = (I A)X = 0.9 1 0.5 1 30 10 100 60

Harder Question: Suppose the farmer wants to be able to sell 50 tons of oats and 15 tons of manure. How much of each should he produce?

Harder Question: Suppose the farmer wants to be able to sell 50 tons of oats and 15 tons of manure. How much of each should he produce? Solution: Here D is given but X is the unknown. We can use either form (2) or form (3). Here I use form (2). D = 50 15 ,

Harder Question: Suppose the farmer wants to be able to sell 50 tons of oats and 15 tons of manure. How much of each should he produce? Solution: Here D is given but X is the unknown. We can use either form (2) or form (3). Here I use form (2). D = 50 15 ,

D = (I A)X 50 15 = 0.9 1 0.5 1 X

Harder Question: Suppose the farmer wants to be able to sell 50 tons of oats and 15 tons of manure. How much of each should he produce? Solution: Here D is given but X is the unknown. We can use either form (2) or form (3). Here I use form (2). D = 50 15 ,

D = (I A)X 50 15 = 0.9 1 0.5 1 X

This translates into the augmented matrix 0.9 1 50 0.5 1 15 .

0.9 1 50 0.5 1 15 Now row reduce. R1 R2 :

0.5 1 15 0.9 1 50

0.9 1 50 0.5 1 15 Now row reduce. R1 R2 :

0.5 1 15 0.9 1 50 1 2 30 50 0.9 1

2 R1 :

0.9 1 50 0.5 1 15 Now row reduce. R1 R2 :

0.5 1 15 0.9 1 50 1 2 30 50 0.9 1 1 2 30 0 0.8 77

2 R1 :
+

0.9R1 R2 :

0.9 1 50 0.5 1 15 Now row reduce. R1 R2 :

0.5 1 15 0.9 1 50 1 2 30 50 0.9 1 1 2 30 0 0.8 77 1 2 30 0 1 385/4

2 R1 :
+

0.9R1 R2 :
5/ 4

R2 :

0.9 1 50 0.5 1 15 Now row reduce. R1 R2 :

0.5 1 15 0.9 1 50 1 2 30 50 0.9 1 1 2 30 0 0.8 77 1 2 30 0 1 385/4 1 0 0 1


445/ 2 385/ 4

2 R1 :
+

0.9R1 R2 :
5/ 4

R2 :

2R2 R1 :

1 0 0 1 So the solution is # tons of oats # tons of manure

445/ 2 385/ 4

= x1 = = x2 =

445/ 2 385/ 4

= 222.5 = 96.25.

Linear Programming

Bad news: this chapter is all about story problems.

Linear Programming

Bad news: this chapter is all about story problems. Good news: the only math you need is high school algebra.

Linear Programming

Bad news: this chapter is all about story problems. Good news: the only math you need is high school algebra. Bad news: solving a single problem is a 3 step process that can take 20 minutes.

Linear Programming

Bad news: this chapter is all about story problems. Good news: the only math you need is high school algebra. Bad news: solving a single problem is a 3 step process that can take 20 minutes. Good news: all 3 steps are easy and most do well on these problems.

Example: Suppose youre taking a dozen 10 year olds out to see Ice Age IV. You gure youll need 44 ounces of popcorn and 56 ounces of soda to see them through the movie. The snack bar oers two snack combos that look interesting: the Gulpalicious, which oers a 12 oz. popcorn with an 8 oz. soda for $8, and the Slurpalicious, which oers an 8 oz. popcorn and a 12 oz. soda for $7. How many of each combo should you buy in order to get all the popcorn and soda you need at the lowest price?

Example: Suppose youre taking a dozen 10 year olds out to see Ice Age IV. You gure youll need 44 ounces of popcorn and 56 ounces of soda to see them through the movie. The snack bar oers two snack combos that look interesting: the Gulpalicious, which oers a 12 oz. popcorn with an 8 oz. soda for $8, and the Slurpalicious, which oers an 8 oz. popcorn and a 12 oz. soda for $7. How many of each combo should you buy in order to get all the popcorn and soda you need at the lowest price? Strategy: Theres a lot of info here. First identify exactly what the question is asking for.

Example: Suppose youre taking a dozen 10 year olds out to see Ice Age IV. You gure youll need 44 ounces of popcorn and 56 ounces of soda to see them through the movie. The snack bar oers two snack combos that look interesting: the Gulpalicious, which oers a 12 oz. popcorn with an 8 oz. soda for $8, and the Slurpalicious, which oers an 8 oz. popcorn and a 12 oz. soda for $7. How many of each combo should you buy in order to get all the popcorn and soda you need at the lowest price?

Example: Suppose youre taking a dozen 10 year olds out to see Ice Age IV. You gure youll need 44 ounces of popcorn and 56 ounces of soda to see them through the movie. The snack bar oers two snack combos that look interesting: the Gulpalicious, which oers a 12 oz. popcorn with an 8 oz. soda for $8, and the Slurpalicious, which oers an 8 oz. popcorn and a 12 oz. soda for $7. How many of each combo should you buy in order to get all the popcorn and soda you need at the lowest price? Strategy: So the answer should be two numbers: x y = = # Gulpaliciouses # Slurpaliciouses.

Example: Suppose youre taking a dozen 10 year olds out to see Ice Age IV. You gure youll need 44 ounces of popcorn and 56 ounces of soda to see them through the movie. The snack bar oers two snack combos that look interesting: the Gulpalicious, which oers a 12 oz. popcorn with an 8 oz. soda for $8, and the Slurpalicious, which oers an 8 oz. popcorn and a 12 oz. soda for $7. How many of each combo should you buy in order to get all the popcorn and soda you need at the lowest price? Strategy: In all these problems there will be some quantity youre trying to optimize.

Example: Suppose youre taking a dozen 10 year olds out to see Ice Age IV. You gure youll need 44 ounces of popcorn and 56 ounces of soda to see them through the movie. The snack bar oers two snack combos that look interesting: the Gulpalicious, which oers a 12 oz. popcorn with an 8 oz. soda for $8, and the Slurpalicious, which oers an 8 oz. popcorn and a 12 oz. soda for $7. How many of each combo should you buy in order to get all the popcorn and soda you need at the lowest price? Strategy: In all these problems there will be some quantity youre trying to optimize. Youre not given a denite target for this quantity, you just want to do as well as you can.

Example: Suppose youre taking a dozen 10 year olds out to see Ice Age IV. You gure youll need 44 ounces of popcorn and 56 ounces of soda to see them through the movie. The snack bar oers two snack combos that look interesting: the Gulpalicious, which oers a 12 oz. popcorn with an 8 oz. soda for $8, and the Slurpalicious, which oers an 8 oz. popcorn and a 12 oz. soda for $7. How many of each combo should you buy in order to get all the popcorn and soda you need at the lowest price? Strategy: In all these problems there will be some quantity youre trying to optimize. Youre not given a denite target for this quantity, you just want to do as well as you can. What is that quantity here?

Example: Suppose youre taking a dozen 10 year olds out to see Ice Age IV. You gure youll need 44 ounces of popcorn and 56 ounces of soda to see them through the movie. The snack bar oers two snack combos that look interesting: the Gulpalicious, which oers a 12 oz. popcorn with an 8 oz. soda for $8, and the Slurpalicious, which oers an 8 oz. popcorn and a 12 oz. soda for $7. How many of each combo should you buy in order to get all the popcorn and soda you need at the lowest price? Strategy: Here we want the lowest price possible.

Example: Suppose youre taking a dozen 10 year olds out to see Ice Age IV. You gure youll need 44 ounces of popcorn and 56 ounces of soda to see them through the movie. The snack bar oers two snack combos that look interesting: the Gulpalicious, which oers a 12 oz. popcorn with an 8 oz. soda for $8, and the Slurpalicious, which oers an 8 oz. popcorn and a 12 oz. soda for $7. How many of each combo should you buy in order to get all the popcorn and soda you need at the lowest price? Strategy: Here we want the lowest price possible. This is called the objective function, because its actual value depends on x and y .

Example: Suppose youre taking a dozen 10 year olds out to see Ice Age IV. You gure youll need 44 ounces of popcorn and 56 ounces of soda to see them through the movie. The snack bar oers two snack combos that look interesting: the Gulpalicious, which oers a 12 oz. popcorn with an 8 oz. soda for $8, and the Slurpalicious, which oers an 8 oz. popcorn and a 12 oz. soda for $7. How many of each combo should you buy in order to get all the popcorn and soda you need at the lowest price? Strategy: Here we want the lowest price possible. This is called the objective function, because its actual value depends on x and y . Here the objective is minimize 8x + 7 y .

Example: Suppose youre taking a dozen 10 year olds out to see Ice Age IV. You gure youll need 44 ounces of popcorn and 56 ounces of soda to see them through the movie. The snack bar oers two snack combos that look interesting: the Gulpalicious, which oers a 12 oz. popcorn with an 8 oz. soda for $8, and the Slurpalicious, which oers an 8 oz. popcorn and a 12 oz. soda for $7. How many of each combo should you buy in order to get all the popcorn and soda you need at the lowest price? Strategy: There are other quantities for which we do have targets.

Example: Suppose youre taking a dozen 10 year olds out to see Ice Age IV. You gure youll need 44 ounces of popcorn and 56 ounces of soda to see them through the movie. The snack bar oers two snack combos that look interesting: the Gulpalicious, which oers a 12 oz. popcorn with an 8 oz. soda for $8, and the Slurpalicious, which oers an 8 oz. popcorn and a 12 oz. soda for $7. How many of each combo should you buy in order to get all the popcorn and soda you need at the lowest price? Strategy: There are other quantities for which we do have targets. The precise amounts of popcorn and soda depends on x and y , but each must satisfy at least the given requirements.

Example: Suppose youre taking a dozen 10 year olds out to see Ice Age IV. You gure youll need 44 ounces of popcorn and 56 ounces of soda to see them through the movie. The snack bar oers two snack combos that look interesting: the Gulpalicious, which oers a 12 oz. popcorn with an 8 oz. soda for $8, and the Slurpalicious, which oers an 8 oz. popcorn and a 12 oz. soda for $7. How many of each combo should you buy in order to get all the popcorn and soda you need at the lowest price? Strategy: There are other quantities for which we do have targets. The precise amounts of popcorn and soda depends on x and y , but each must satisfy at least the given requirements. # oz.s popcorn # oz.s soda 44 56

Example: Suppose youre taking a dozen 10 year olds out to see Ice Age IV. You gure youll need 44 ounces of popcorn and 56 ounces of soda to see them through the movie. The snack bar oers two snack combos that look interesting: the Gulpalicious, which oers a 12 oz. popcorn with an 8 oz. soda for $8, and the Slurpalicious, which oers an 8 oz. popcorn and a 12 oz. soda for $7. How many of each combo should you buy in order to get all the popcorn and soda you need at the lowest price? Strategy: We dont know exactly how much of each well get, but we can express the quantities in terms of x and y . 12x + 8y 8x + 12y = # oz.s popcorn = # oz.s soda 44 56

Example: Suppose youre taking a dozen 10 year olds out to see Ice Age IV. You gure youll need 44 ounces of popcorn and 56 ounces of soda to see them through the movie. The snack bar oers two snack combos that look interesting: the Gulpalicious, which oers a 12 oz. popcorn with an 8 oz. soda for $8, and the Slurpalicious, which oers an 8 oz. popcorn and a 12 oz. soda for $7. How many of each combo should you buy in order to get all the popcorn and soda you need at the lowest price? Strategy: Some people nd it helpful in these Materials & Products problems to make a table. Gulp. popcorn soda 12 oz. 8 oz. Slurp. 8 oz. 12 oz. Total 44 oz. 56 oz.

Gulp. popcorn soda 12 oz. 8 oz.

Slurp. 8 oz. 12 oz.

Total 44 oz. 56 oz.

From this table we get the inequalities 12x + 8y 8x + 12y 44 56.

Gulp. popcorn soda 12 oz. 8 oz.

Slurp. 8 oz. 12 oz.

Total 44 oz. 56 oz.

From this table we get the inequalities 12x + 8y 8x + 12y 44 56.

The direction of the inequalities is because the numbers on the right are minimum requirements .

Gulp. popcorn soda 12 oz. 8 oz.

Slurp. 8 oz. 12 oz.

Total 44 oz. 56 oz.

From this table we get the inequalities 12x + 8y 8x + 12y 44 56.

The direction of the inequalities is because the numbers on the right are minimum requirements . In other problems they may go the other way.

Example: Suppose youre taking a dozen 10 year olds out to see Ice Age IV. You gure youll need 44 ounces of popcorn and 56 ounces of soda to see them through the movie. The snack bar oers two snack combos that look interesting: the Gulpalicious, which oers a 12 oz. popcorn with an 8 oz. soda for $8, and the Slurpalicious, which oers an 8 oz. popcorn and a 12 oz. soda for $7. How many of each combo should you buy in order to get all the popcorn and soda you need at the lowest price? Strategy: You cant buy a negative number of snack combos. Therefore x and y must satisfy the natural constraints: x y 0 0.

Example: Suppose youre taking a dozen 10 year olds out to see Ice Age IV. You gure youll need 44 ounces of popcorn and 56 ounces of soda to see them through the movie. The snack bar oers two snack combos that look interesting: the Gulpalicious, which oers a 12 oz. popcorn with an 8 oz. soda for $8, and the Slurpalicious, which oers an 8 oz. popcorn and a 12 oz. soda for $7. How many of each combo should you buy in order to get all the popcorn and soda you need at the lowest price? Strategy: You cant buy a negative number of snack combos. Therefore x and y must satisfy the natural constraints: x y 0 0.

The 4 inequalities together are called the constraints.

Putting together everything weve done so far, we get the following. x y We want to minimize subject to the constraints x y 12x + 8y 8x + 12y 0 0 44 56. 8x + 7y = = # Gulpaliciouses # Slurpaliciouses

Putting together everything weve done so far, we get the following. x y We want to minimize subject to the constraints x y 12x + 8y 8x + 12y 0 0 44 56. 8x + 7y = = # Gulpaliciouses # Slurpaliciouses

This is called the mathematical formulation of the problem.

Formulation of a Linear Programming Problem

1. Dene the variables, including the units of measure; 2. write out the objective function in terms of the variables, and specify whether you want to maximize or minimize it; 3. write out the system of constraints, including the natural constraints if applicable.

Example: (7.1 #12) A purchasing agent for a college nds it necessary to decide how much hard and soft chalk should be purchased each month. He knows that a typical instructor will use two pieces of soft chalk or one piece of hard chalk for each class. In addition, he has observed that hard chalk always amounts to at least one-quarter of the total used. Finally, his supplier has limited him to a purchase of at most 60 gross (8640 pieces) of soft chalk. There are 3600 classes to be taught each month. If soft chalk is $1.50 per gross and hard chalk is $3.50 per gross, how much of each should be purchased to meet the needs and to keep costs to a minimum?

Example: (7.1 #12) A purchasing agent for a college nds it necessary to decide how much hard and soft chalk should be purchased each month. He knows that a typical instructor will use two pieces of soft chalk or one piece of hard chalk for each class. In addition, he has observed that hard chalk always amounts to at least one-quarter of the total used. Finally, his supplier has limited him to a purchase of at most 60 gross (8640 pieces) of soft chalk. There are 3600 classes to be taught each month. If soft chalk is $1.50 per gross and hard chalk is $3.50 per gross, how much of each should be purchased to meet the needs and to keep costs to a minimum? Variables: x y = = # pieces of soft chalk # pieces of hard chalk

Other units could be used, e.g. gross.

Example: (7.1 #12) A purchasing agent for a college nds it necessary to decide how much hard and soft chalk should be purchased each month. He knows that a typical instructor will use two pieces of soft chalk or one piece of hard chalk for each class. In addition, he has observed that hard chalk always amounts to at least one-quarter of the total used. Finally, his supplier has limited him to a purchase of at most 60 gross (8640 pieces) of soft chalk. There are 3600 classes to be taught each month. If soft chalk is $1.50 per gross and hard chalk is $3.50 per gross, how much of each should be purchased to meet the needs and to keep costs to a minimum? Objective Function: Since gross = dozen dozen = 144, minimize Cost = 1.5 = y x + 3.5 144 144 x+ 3.5 144 y.

1.5 144

Example: (7.1 #12) A purchasing agent for a college nds it necessary to decide how much hard and soft chalk should be purchased each month. He knows that a typical instructor will use two pieces of soft chalk or one piece of hard chalk for each class. In addition, he has observed that hard chalk always amounts to at least one-quarter of the total used. Finally, his supplier has limited him to a purchase of at most 60 gross (8640 pieces) of soft chalk. There are 3600 classes to be taught each month. If soft chalk is $1.50 per gross and hard chalk is $3.50 per gross, how much of each should be purchased to meet the needs and to keep costs to a minimum? Constraints: The natural constraints apply here: x y 0 0.

Example: (7.1 #12) A purchasing agent for a college nds it necessary to decide how much hard and soft chalk should be purchased each month. He knows that a typical instructor will use two pieces of soft chalk or one piece of hard chalk for each class. In addition, he has observed that hard chalk always amounts to at least one-quarter of the total used. Finally, his supplier has limited him to a purchase of at most 60 gross (8640 pieces) of soft chalk. There are 3600 classes to be taught each month. If soft chalk is $1.50 per gross and hard chalk is $3.50 per gross, how much of each should be purchased to meet the needs and to keep costs to a minimum? Constraints: classes per piece soft chalk classes per piece hard chalk = 1/2 = 1

Example: (7.1 #12) A purchasing agent for a college nds it necessary to decide how much hard and soft chalk should be purchased each month. He knows that a typical instructor will use two pieces of soft chalk or one piece of hard chalk for each class. In addition, he has observed that hard chalk always amounts to at least one-quarter of the total used. Finally, his supplier has limited him to a purchase of at most 60 gross (8640 pieces) of soft chalk. There are 3600 classes to be taught each month. If soft chalk is $1.50 per gross and hard chalk is $3.50 per gross, how much of each should be purchased to meet the needs and to keep costs to a minimum? Constraints: classes per piece soft chalk classes per piece hard chalk # classes with all soft chalk # classes with all hard chalk = 1/2 = 1 = 1/2 x = 1y

Example: (7.1 #12) A purchasing agent for a college nds it necessary to decide how much hard and soft chalk should be purchased each month. He knows that a typical instructor will use two pieces of soft chalk or one piece of hard chalk for each class. In addition, he has observed that hard chalk always amounts to at least one-quarter of the total used. Finally, his supplier has limited him to a purchase of at most 60 gross (8640 pieces) of soft chalk. There are 3600 classes to be taught each month. If soft chalk is $1.50 per gross and hard chalk is $3.50 per gross, how much of each should be purchased to meet the needs and to keep costs to a minimum? Constraints: classes per piece soft chalk classes per piece hard chalk # classes with all soft chalk # classes with all hard chalk total # classes with all chalk = 1/2 = 1 = 1/2 x = 1y =
1/ x 2

+ y 3600

Example: (7.1 #12) A purchasing agent for a college nds it necessary to decide how much hard and soft chalk should be purchased each month. He knows that a typical instructor will use two pieces of soft chalk or one piece of hard chalk for each class. In addition, he has observed that hard chalk always amounts to at least one-quarter of the total used. Finally, his supplier has limited him to a purchase of at most 60 gross (8640 pieces) of soft chalk. There are 3600 classes to be taught each month. If soft chalk is $1.50 per gross and hard chalk is $3.50 per gross, how much of each should be purchased to meet the needs and to keep costs to a minimum? Constraints: total chalk used =

Example: (7.1 #12) A purchasing agent for a college nds it necessary to decide how much hard and soft chalk should be purchased each month. He knows that a typical instructor will use two pieces of soft chalk or one piece of hard chalk for each class. In addition, he has observed that hard chalk always amounts to at least one-quarter of the total used. Finally, his supplier has limited him to a purchase of at most 60 gross (8640 pieces) of soft chalk. There are 3600 classes to be taught each month. If soft chalk is $1.50 per gross and hard chalk is $3.50 per gross, how much of each should be purchased to meet the needs and to keep costs to a minimum? Constraints: total chalk used = x +y

Example: (7.1 #12) A purchasing agent for a college nds it necessary to decide how much hard and soft chalk should be purchased each month. He knows that a typical instructor will use two pieces of soft chalk or one piece of hard chalk for each class. In addition, he has observed that hard chalk always amounts to at least one-quarter of the total used. Finally, his supplier has limited him to a purchase of at most 60 gross (8640 pieces) of soft chalk. There are 3600 classes to be taught each month. If soft chalk is $1.50 per gross and hard chalk is $3.50 per gross, how much of each should be purchased to meet the needs and to keep costs to a minimum? Constraints: total chalk used one-quarter the total chalk used = x +y =

Example: (7.1 #12) A purchasing agent for a college nds it necessary to decide how much hard and soft chalk should be purchased each month. He knows that a typical instructor will use two pieces of soft chalk or one piece of hard chalk for each class. In addition, he has observed that hard chalk always amounts to at least one-quarter of the total used. Finally, his supplier has limited him to a purchase of at most 60 gross (8640 pieces) of soft chalk. There are 3600 classes to be taught each month. If soft chalk is $1.50 per gross and hard chalk is $3.50 per gross, how much of each should be purchased to meet the needs and to keep costs to a minimum? Constraints: total chalk used one-quarter the total chalk used = x +y = 1 4 (x + y )

Example: (7.1 #12) A purchasing agent for a college nds it necessary to decide how much hard and soft chalk should be purchased each month. He knows that a typical instructor will use two pieces of soft chalk or one piece of hard chalk for each class. In addition, he has observed that hard chalk always amounts to at least one-quarter of the total used. Finally, his supplier has limited him to a purchase of at most 60 gross (8640 pieces) of soft chalk. There are 3600 classes to be taught each month. If soft chalk is $1.50 per gross and hard chalk is $3.50 per gross, how much of each should be purchased to meet the needs and to keep costs to a minimum? Constraints: total chalk used one-quarter the total chalk used y = x +y = 1 4 (x + y )

Example: (7.1 #12) A purchasing agent for a college nds it necessary to decide how much hard and soft chalk should be purchased each month. He knows that a typical instructor will use two pieces of soft chalk or one piece of hard chalk for each class. In addition, he has observed that hard chalk always amounts to at least one-quarter of the total used. Finally, his supplier has limited him to a purchase of at most 60 gross (8640 pieces) of soft chalk. There are 3600 classes to be taught each month. If soft chalk is $1.50 per gross and hard chalk is $3.50 per gross, how much of each should be purchased to meet the needs and to keep costs to a minimum? Constraints: total chalk used one-quarter the total chalk used y = x +y = 1 4 (x + y )
1 4 (x

+ y)

Example: (7.1 #12) A purchasing agent for a college nds it necessary to decide how much hard and soft chalk should be purchased each month. He knows that a typical instructor will use two pieces of soft chalk or one piece of hard chalk for each class. In addition, he has observed that hard chalk always amounts to at least one-quarter of the total used. Finally, his supplier has limited him to a purchase of at most 60 gross (8640 pieces) of soft chalk. There are 3600 classes to be taught each month. If soft chalk is $1.50 per gross and hard chalk is $3.50 per gross, how much of each should be purchased to meet the needs and to keep costs to a minimum? Constraints: x 8640

Variables: x y Objective Function: minimize 1.5 144 x+ 3.5 144 y = = # pieces of soft chalk # pieces of hard chalk

Constraints: x y 1/ x + y 2 y x 0 0 3600 1 4 (x + y ) 8640

Graphing Inequalities
To graph the inequality x + 2y 4 means to shade the region of the plane containing the points that satisfy it.

Graphing Inequalities
To graph the inequality x + 2y 4 means to shade the region of the plane containing the points that satisfy it. Here is the graph of the equation x + 2y = 4.
5 4 3 2 1 0 -1 -2 -3 -4 -4 -3 -2 -1 0 1 2 3 4 5

All the points on the line satisfy the inequality.

What other points satisfy x + 2y 4? Lets try out a few. (2, 3) : 2 + 2(3) 4 is false, so no.

What other points satisfy x + 2y 4? Lets try out a few. (2, 3) : 2 + 2(3) 4 is false, so no. (2, 3) : 2 + 2(3) 4 is true, so yes.

What other points satisfy x + 2y 4? Lets try out a few. (2, 3) : 2 + 2(3) 4 is false, so no. (2, 3) : 2 + 2(3) 4 is true, so yes. (3, 2) : 3 + 2(2) 4 is true, so yes.

What other points satisfy x + 2y 4? Lets try out a few. (2, 3) : 2 + 2(3) 4 is false, so no. (2, 3) : 2 + 2(3) 4 is true, so yes. (3, 2) : 3 + 2(2) 4 is true, so yes. (1, 4) : 1 + 2(4) 4 is false, so no.

What other points satisfy x + 2y 4? Lets try out a few. (2, 3) : 2 + 2(3) 4 is false, so no. (2, 3) : 2 + 2(3) 4 is true, so yes. (3, 2) : 3 + 2(2) 4 is true, so yes. (1, 4) : 1 + 2(4) 4 is false, so no. (3, 2) : 3 + 2(2) 4 is true, so yes.

What other points satisfy x + 2y 4? Lets try out a few. (2, 3) : 2 + 2(3) 4 is false, so no. (2, 3) : 2 + 2(3) 4 is true, so yes. (3, 2) : 3 + 2(2) 4 is true, so yes. (1, 4) : 1 + 2(4) 4 is false, so no. (3, 2) : 3 + 2(2) 4 is true, so yes. (1, 1) : 1 + 2(1) 4 is true, so yes.

What other points satisfy x + 2y 4? Lets try out a few. (2, 3) : 2 + 2(3) 4 is false, so no. (2, 3) : 2 + 2(3) 4 is true, so yes. (3, 2) : 3 + 2(2) 4 is true, so yes. (1, 4) : 1 + 2(4) 4 is false, so no. (3, 2) : 3 + 2(2) 4 is true, so yes. (1, 1) : 1 + 2(1) 4 is true, so yes.
5 4 3 2 1 0 -1 -2 -3 -4 -4 -3 -2 -1 0 1 2 3 4 5

Theres a pattern here.

5 4 3 2 1 0 -1 -2 -3 -4 -4 -3 -2 -1 0 1 2 3 4 5

The line x + 2y = 4 divides the plane into two half-planes.

5 4 3 2 1 0 -1 -2 -3 -4 -4 -3 -2 -1 0 1 2 3 4 5

The line x + 2y = 4 divides the plane into two half-planes. In one half-plane all the points satisfy x + 2y 4; in the other half-plane none do.

5 4 3 2 1 0 -1 -2 -3 -4 -4 -3 -2 -1 0 1 2 3 4 5

The line x + 2y = 4 divides the plane into two half-planes. In one half-plane all the points satisfy x + 2y 4; in the other half-plane none do. So once we have graphed the line, it only remains to determine which half-plane should be shaded.

There are two methods for doing that.

There are two methods for doing that. The rst method is the test point method: test any point which is not on the line itself. Any of the six points we looked at would do. For example, (2, 3) does not satisfy the inequality; therefore (2, 3) is in the wrong half-plane;

There are two methods for doing that. The rst method is the test point method: test any point which is not on the line itself. Any of the six points we looked at would do. For example, (2, 3) does not satisfy the inequality; therefore (2, 3) is in the wrong half-plane;
5 4 3 2 1 0 -1 -2 -3 -4 -4 -3 -2 -1 0 1 2 3 4 5

therefore we shade the other half-plane.

The other method for determining which half-plane to shade is to solve the inequality for one of the variables.

The other method for determining which half-plane to shade is to solve the inequality for one of the variables. x + 2y x 4 4 2y

The other method for determining which half-plane to shade is to solve the inequality for one of the variables. x + 2y x 4 4 2y

This means that x is less than equal to the other side:

The other method for determining which half-plane to shade is to solve the inequality for one of the variables. x + 2y x 4 4 2y

This means that x is less than equal to the other side: the variable x gets smaller towards the left,

The other method for determining which half-plane to shade is to solve the inequality for one of the variables. x + 2y x 4 4 2y

This means that x is less than equal to the other side: the variable x gets smaller towards the left,
5 4 3 2 1 0 -1 -2 -3 -4 -4 -3 -2 -1 0 1 2 3 4 5

right
so we should shade the left side of the line x +2y = 4.

left

Remember the algebra about inequalities: Multiplication by a negative reverses the inequality!

Remember the algebra about inequalities: Multiplication by a negative reverses the inequality!

For example, 2x + 4y 6

Remember the algebra about inequalities: Multiplication by a negative reverses the inequality!

For example, 2x + 4y 2x 6 6 4y

Remember the algebra about inequalities: Multiplication by a negative reverses the inequality!

For example, 2x + 4y 2x x 6 6 4y 3 + 2 y .

Now lets complicate things. Lets graph the system of inequalities x + 2y y 4 7 3x I II.

Weve already done I, so lets look at II.

Now lets complicate things. Lets graph the system of inequalities x + 2y y 4 7 3x I II.

Weve already done I, so lets look at II. The line y = 7 3x has slope 3 and y -intercept at (0, 7).
8 7 6 5 4 3 2 1 0 -1 -1 0 1 2 3 4 5

We can use (0, 0) as a test point: it makes the inequality true, so we shade the lower left half-plane.

Inequality I:
8 7 6 5 4 3 2 1 0 -1 -1 0 1 2 3 4 5

Inequalities I and II together:


8 7 6 5 4 3 2 1 0 -1 -1 0 1 2 3 4 5

Final Answer:
8 7 6 5 4 3 2 1 0 -1 -1 0 1 2 3 4 5

The nal answer is the region that satises all the inequalities from the original system.

The nal answer is the region that satises all the inequalities from the original system. There are two methods for putting the separate inequalities together. 1. Use a dierent color for eachthen the nal answer is the darkest region; 2. use each inequality to successively eliminate part of the planethe the nal answer is what is left over.

Heres the last problem done using successive elimination. Inequality I (red means eliminated):
8 7 6 5 4 3 2 1 0 -1 -1 0 1 2 3 4 5

Inequality II (the nal answer is white):


8 7 6 5 4 3 2 1 0 -1 -1 0 1 2 3 4 5

Were interested in using these graphs to solve LP problems so our systems will usually include the natural constraints. Example: x y x + 2y y 0 0 4 7 3x

I II

Were interested in using these graphs to solve LP problems so our systems will usually include the natural constraints. Example: x y x + 2y y 0 0 4 7 3x

I II

The addition of the natural constraints produces a slight change: since x and y must both be nonnegative, the answer includes only points from the rst quadrant.

8 7 6 5 4 3 2 1 0 -1 -1 0 1 2 3 4 5

II I

8 7 6 5 4 3 2 1 0 -1 -1 0 1 2 3 4 5

II I

In the context of LP problems, this is called the feasible set.

We need to know a little bit more about the feasible set: We also need to know the coordinates of the corners.
4 3

D
2

II C

1 0

I A B
0 1 2 3 4

A is obviously (0, 0). B and D are also easy: theyre intercepts of the lines.

-1 -1

4 3

D
2

II C

B is the x -intercept of II: y = 7 3x .

1 0

I A B
0 1 2 3 4

-1 -1

4 3

D
2

II C

B is the x -intercept of II: y = 7 3x . So


I
7/ 3

1 0

0 = 7 3x = x,

A
-1 -1 0 1

B
2 3 4

4 3

D
2

II C

B is the x -intercept of II: y = 7 3x . So


I
7/ 3

1 0

0 = 7 3x = x,

A
-1 -1 0 1

B
2 3 4

and B = (7/3 , 0).

4 3

D
2

II C

B is the x -intercept of II: y = 7 3x . So


I
7/ 3

1 0

0 = 7 3x = x,

A
-1 -1 0 1

B
2 3 4

and B = (7/3 , 0).

Similarly, D is the y -intercept of I: x + 2y = 4. 0 + 2y = 4 y = 2 D = (0, 2)

4 3

D
2

II C

1 0

I A B
0 1 2 3 4

C is the only hard one. It is formed by the intersection of lines I and II, so we must solve a system of linear equations to nd it: x + 2y y = 4 = 7 3x .

-1 -1

4 3

D
2

II C

1 0

I A B
0 1 2 3 4

C is the only hard one. It is formed by the intersection of lines I and II, so we must solve a system of linear equations to nd it: x + 2y y = 4 = 7 3x .

-1 -1

By substitution, x + 2(7 3x ) 5x + 14 5x x = = = = 4 4 10 2

4 3

D
2

II C

1 0

I A B
0 1 2 3 4

C is the only hard one. It is formed by the intersection of lines I and II, so we must solve a system of linear equations to nd it: x + 2y y = 4 = 7 3x .

-1 -1

By substitution, x + 2(7 3x ) 5x + 14 5x x y = = = = = 4 4 10 2 7 3(2) = 1.

So C = (2, 1). Now we have all the corners. A B C D (0, 0) (7/3 , 0) (2, 1) (0, 2)

This table will be crucial in actually solving LP problems.

Solving LP Problems
Now suppose our objective is to maximize the value of 3x + 2y on the feasible set. Here is the previous feasible set together with the values of 3x + 2y indicated by color: green for high values and red for low values.

Of all the points in the feasible set, which is the greenest?

Solving LP Problems
Now suppose our objective is to maximize the value of 3x + 2y on the feasible set. Here is the previous feasible set together with the values of 3x + 2y indicated by color: green for high values and red for low values.

Of all the points in the feasible set, which is the greenest? Suppose we were looking for the minimum?

Solving LP Problems
Now suppose the objective function is x y . Remember: green for high values and red for low values.

Of all the points in the feasible set, which is the greenest?

Solving LP Problems
Now suppose the objective function is x y . Remember: green for high values and red for low values.

Of all the points in the feasible set, which is the greenest? Which is the reddest?

It seems that both the max and min values are at corner points.

It seems that both the max and min values are at corner points. In fact this always true if the objective function is linear . With a linear objective function, the color gradient is straight.

It seems that both the max and min values are at corner points. In fact this always true if the objective function is linear . With a linear objective function, the color gradient is straight. So to solve an LP problem, all you have to do is to compare values of the objective function at the corner points of the feasible set: one of those points must be the answer.

Point A B C D (0, 0) (7/3 , 0) (2, 1) (0, 2)

3x + 2y 0 7 8 4

x y 0
7/ 3

1 2

This problem contains an additional complication. Example: Find the max and min of 3x + 2y subject to the constraints I x x 2y 0 6 II y x 3y 0 2.

This problem contains an additional complication. Example: Find the max and min of 3x + 2y subject to the constraints I Graph:
4

x x 2y

0 6

II

y x 3y

0 2.

C
3

The corners B and C are just intercepts. A = (0, 0) B = (2, 0) C = (0, 3)

I
2 1

A
-1 -1 0

II
1

B
2 3 4

This problem contains an additional complication. Example: Find the max and min of 3x + 2y subject to the constraints I Graph:
4

x x 2y

0 6

II

y x 3y

0 2.

C
3

The corners B and C are just intercepts. A = (0, 0) B = (2, 0) C = (0, 3)

I
2 1

A
-1 -1 0

II
1

B
2 3 4

Note that the feasible set continues forever to the upper right.

Here the objective function 3x + 2y is overlaid. See a problem?

Here the objective function 3x + 2y is overlaid. See a problem?

There is no maximum, because the objective function just keeps getting bigger toward the upper right.

Here the objective function 3x + 2y is overlaid. See a problem?

There is no maximum, because the objective function just keeps getting bigger toward the upper right.

There is a minimum at (0, 0).

Here the objective function 3x + 2y is overlaid. See a problem?

There is no maximum, because the objective function just keeps getting bigger toward the upper right.

There is a minimum at (0, 0). How could we get those answers without using a computer to graph the objective function?

This feasible set is said to be unbounded.


6 5

D
4 3 2 1

C I E

When the feasible set is unbounded we need additional sample points, one from each unbounded edge.

A
-1 -1 0

II
1

B
2 3 4 5 6

This feasible set is said to be unbounded.


6 5

D
4 3 2 1

C I E

When the feasible set is unbounded we need additional sample points, one from each unbounded edge.

A
-1 -1 0

II
1

B
2 3 4 5 6

Ill call D and E U-points. They can be anywhere inside the unbounded edges.

6 5

Corners
D C I E

4 3 2 1 0

A B C

(0, 0) (2, 0) (0, 3)

U-points D E

A
-1 -1 0

II
1

B
2 3 4 5 6

To get coordinates for D , note that: [h] (a) D is on line I: x 2y = 6; (b) D is to the upper right of C .

6 5

Corners
D C I E

4 3 2 1 0

A B C

(0, 0) (2, 0) (0, 3)

U-points D E

A
-1 -1 0

II
1

B
2 3 4 5 6

To get coordinates for D , note that: [h] (a) D is on line I: x 2y = 6; (b) D is to the upper right of C . We can use (b) to get one coordinate; then use (a) to get the other.

Corners A B C (0, 0) (2, 0) (0, 3)

[h]

U-points D E (a) D is on line I: x 2y = 6; (b) D is to the upper right of C . By (b) we could use e.g. either x = 1 or y = 4. Ill use the latter because it makes the algebra easier.

Corners A B C (0, 0) (2, 0) (0, 3)

[h]

U-points D E (a) D is on line I: x 2y = 6; (b) D is to the upper right of C . By (b) we could use e.g. either x = 1 or y = 4. Ill use the latter because it makes the algebra easier. By (a),

For E the method is the same. Corners A B C (0, 0) (2, 0) (0, 3)

[h]

U-points D E (2, 4)

(a) E is on line II: x 3y = 2; (b) E is to the upper right of B .

For E the method is the same. Corners A B C (0, 0) (2, 0) (0, 3)

[h]

U-points D E (2, 4)

(a) E is on line II: x 3y = 2; (b) E is to the upper right of B . By (b) we could use e.g. either x = 3 or y = 1. Ill use the latter because it makes the algebra easier.

For E the method is the same. Corners A B C (0, 0) (2, 0) (0, 3)

[h]

U-points D E (2, 4)

(a) E is on line II: x 3y = 2; (b) E is to the upper right of B . By (b) we could use e.g. either x = 3 or y = 1. Ill use the latter because it makes the algebra easier.

Now complete the table. Corners A B C (0, 0) (2, 0) (0, 3) 3x + 2y 0 6 6

U-points D E (2, 4) (5, 1) 14 17

Now complete the table. Corners A B C (0, 0) (2, 0) (0, 3) 3x + 2y 0 6 6

U-points D E (2, 4) (5, 1) 14 17

The max value is at a U-point; the min value is at a corner. The rule is: 1. if the best value is at a corner, thats the answer; 2. if the best value is at a U-point, there is no answer.

Heres the whole graph with all sample points and the objective function overlaid.

Corners A B C (0, 0) (2, 0) (0, 3)

3x + 2y 0 6 6

U-points D E (2, 4) (5, 1) 14 17

The fact that the U-points take the highest values indicates that the values will increase forever.

Heres the same feasible set with a dierent objective function, y x.

Corners A B C (0, 0) (2, 0) (0, 3)

y x 0 2 3

U-points D E (2, 4) (5, 1) 2 4

Heres the same feasible set with a dierent objective function, y x.

Corners A B C (0, 0) (2, 0) (0, 3)

y x 0 2 3

U-points D E (2, 4) (5, 1) 2 4

Here the max is 3 but there is no min.

Markov Chains
A Markov Chain is a system that can be in dierent states at dierent times. Example: Weather: Rainy, Sunny, Cloudy; times: days.

Markov Chains
A Markov Chain is a system that can be in dierent states at dierent times. Example: Weather: Rainy, Sunny, Cloudy; times: days. In this case the weather is observed every day, and the state of the weather is assigned to one of the three states and recorded.

Markov Chains
A Markov Chain is a system that can be in dierent states at dierent times. Example: Weather: Rainy, Sunny, Cloudy; times: days. In this case the weather is observed every day, and the state of the weather is assigned to one of the three states and recorded. As this continues a chain of successive states is recorded. R S S C ...

Markov Chains
A Markov Chain is a system that can be in dierent states at dierent times. Example: Weather: Rainy, Sunny, Cloudy; times: days. In this case the weather is observed every day, and the state of the weather is assigned to one of the three states and recorded. As this continues a chain of successive states is recorded. R S S C ...

Note that in this case the weather one day might have some inuence on the next days weather.

Another way to describe a Markov Chain: its like a Bernoulli Process with two dierences.

Another way to describe a Markov Chain: its like a Bernoulli Process with two dierences. 1. The trials can have more outcomes than just S or F .

Another way to describe a Markov Chain: its like a Bernoulli Process with two dierences. 1. The trials can have more outcomes than just S or F . 2. The trials are not independent: one observation may aect the next.

Another way to describe a Markov Chain: its like a Bernoulli Process with two dierences. 1. The trials can have more outcomes than just S or F . 2. The trials are not independent: one observation may aect the next. In a Bernoulli process the probabilities are always the same; in a M.C. they may depend on the previous state.

Transition Diagrams
A good visual representation of a M.C. is by a tree diagram with loops. Example: Suppose daily observations of the Dow Jones Average are made and classied as either Rising, Falling, or Level.

As with a tree diagram, we can also indicate the conditional probabilities of moving along a particular edge.

As with a tree diagram, we can also indicate the conditional probabilities of moving along a particular edge.

For example, the .7 from R to L means: if today the DJ was rising, there is a 70% chance that tomorrow it will be level.

We can trim o any edges with 0 probability.

We can trim o any edges with 0 probability.

This looks nice, but it can be hard to nd the number you want.

Heres another way to present the same information. End F L .3 .5 .6 .7 .1 .2

R R F L 0 .4 .2 Start

Heres another way to present the same information. End F L .3 .5 .6 .7 .1 .2

R R F L 0 .4 .2 Start

So for example, if today the DJ is falling, the probability that it will rise tomorrow is

Heres another way to present the same information. End F L .3 .5 .6 .7 .1 .2

R R F L 0 .4 .2 Start

So for example, if today the DJ is falling, the probability that it will rise tomorrow is 40%.

Heres another way to present the same information. End F L .3 .5 .6 .7 .1 .2

R R F L 0 .4 .2 Start Its usually denoted by P .

So for example, if today the DJ is falling, the probability that it will rise tomorrow is 40%. This table is called the transition matrix of the M.C.

0 .3 .7 P = .4 .5 .1 .2 .6 .2 Things to remember: 1. rows correspond to starting states; 2. columns correspond to ending states;

0 .3 .7 P = .4 .5 .1 .2 .6 .2 Things to remember: 1. rows correspond to starting states; 2. columns correspond to ending states; 3. both have the states in the same order (here R , F , L, left to right or top to bottom);

0 .3 .7 P = .4 .5 .1 .2 .6 .2 Things to remember: 1. rows correspond to starting states; 2. columns correspond to ending states; 3. both have the states in the same order (here R , F , L, left to right or top to bottom); 4. rows always sum to 1;

0 .3 .7 P = .4 .5 .1 .2 .6 .2 Things to remember: 1. rows correspond to starting states; 2. columns correspond to ending states; 3. both have the states in the same order (here R , F , L, left to right or top to bottom); 4. rows always sum to 1; 5. shorthand: probability of a transition from state i to state j is pij . For example, the probability of a transition from F to L is p23 , which is the entry in the second row and third column of P .

State 1 is Rise, State 2 is Fall, State 3 is Level. 0 .3 .7 P = .4 .5 .1 .2 .6 .2 A lot of simple questions can be answered from P alone. 1. If tomorrow the DJ is level, what is the probability it will rise the day after?

State 1 is Rise, State 2 is Fall, State 3 is Level. 0 .3 .7 P = .4 .5 .1 .2 .6 .2 A lot of simple questions can be answered from P alone. 1. If tomorrow the DJ is level, what is the probability it will rise the day after? 20% 2. If the DJ fell today, what is most likely to happen tomorrow?

State 1 is Rise, State 2 is Fall, State 3 is Level. 0 .3 .7 P = .4 .5 .1 .2 .6 .2 A lot of simple questions can be answered from P alone. 1. If tomorrow the DJ is level, what is the probability it will rise the day after? 20% 2. If the DJ fell today, what is most likely to happen tomorrow? Fall 3. If the DJ fell today, what is the probability of it falling the next two days?

State 1 is Rise, State 2 is Fall, State 3 is Level. 0 .3 .7 P = .4 .5 .1 .2 .6 .2 A lot of simple questions can be answered from P alone. 1. If tomorrow the DJ is level, what is the probability it will rise the day after? 20% 2. If the DJ fell today, what is most likely to happen tomorrow? Fall 3. If the DJ fell today, what is the probability of it falling the next two days? 0.5 0.5 = 0.25 4. If the DJ fell today, what is the probability of it being level the next two days?

State 1 is Rise, State 2 is Fall, State 3 is Level. 0 .3 .7 P = .4 .5 .1 .2 .6 .2 A lot of simple questions can be answered from P alone. 1. If tomorrow the DJ is level, what is the probability it will rise the day after? 20% 2. If the DJ fell today, what is most likely to happen tomorrow? Fall 3. If the DJ fell today, what is the probability of it falling the next two days? 0.5 0.5 = 0.25 4. If the DJ fell today, what is the probability of it being level the next two days? 0.1 0.2 = 0.02 F L L
.1 .2

State 1 is Rise, State 2 is Fall, State 3 is Level. 0 .3 .7 P = .4 .5 .1 .2 .6 .2 Heres a harder question. If the DJ fell today, what is the probability it will be level two days from now? In shorthand, p23 (2) = ?

State 1 is Rise, State 2 is Fall, State 3 is Level. 0 .3 .7 P = .4 .5 .1 .2 .6 .2 Heres a harder question. If the DJ fell today, what is the probability it will be level two days from now? In shorthand, p23 (2) = ? ? ? F ? L

State 1 is Rise, State 2 is Fall, State 3 is Level. 0 .3 .7 P = .4 .5 .1 .2 .6 .2 Heres a harder question. If the DJ fell today, what is the probability it will be level two days from now? In shorthand, p23 (2) = ? ? ? F ? L There are three cases: F R L
.4 .7

F F L

.5

.1

F L L

.1

.2

State 1 is Rise, State 2 is Fall, State 3 is Level. 0 .3 .7 P = .4 .5 .1 .2 .6 .2 Heres a harder question. If the DJ fell today, what is the probability it will be level two days from now? In shorthand, p23 (2) = ? ? ? F ? L There are three cases: F R L
.4 .7

F F L

.5

.1

F L L

.1

.2

So the answer is 0.4 0.7 + 0.5 0.1 + 0.1 0.2.

State 1 is Rise, State 2 is Fall, State 3 is Level. 0 .3 .7 P = .4 .5 .1 .2 .6 .2 Notice anything here? 0.4 0.7 + 0.5 0.1 + 0.1 0.2

State 1 is Rise, State 2 is Fall, State 3 is Level. 0 .3 .7 P = .4 .5 .1 .2 .6 .2 Notice anything here? 0.4 0.7 + 0.5 0.1 + 0.1 0.2 Its the product of the second row of P with the third column. .7 p23 (2) = .4 .5 .1 .1 .2

Ill write Ri for the i th row of P , and Cj for the j th column of P .

Ill write Ri for the i th row of P , and Cj for the j th column of P . We have just seen that R2 C3 = p23 (2).

Ill write Ri for the i th row of P , and Cj for the j th column of P . We have just seen that R2 C3 = p23 (2). How about this? R3 C1 =

Ill write Ri for the i th row of P , and Cj for the j th column of P . We have just seen that R2 C3 = p23 (2). How about this? R3 C1 = p31 (2)

Ill write Ri for the i th row of P , and Cj for the j th column of P . We have just seen that R2 C3 = p23 (2). How about this? R3 C1 = p31 (2) In general, Ri Cj = pij (2).

Ill write Ri for the i th row of P , and Cj for the j th column of P . We have just seen that R2 C3 = p23 (2). How about this? R3 C1 = p31 (2) In general, Ri Cj = pij (2).

Because of this, a nice pattern arises when we compute P P .

Using the cannon method, P P becomes C1 R1 C2 C3

R2 R3

Using the cannon method, P P becomes C1 R1 p11 (2) C2 C3

R2 R3

Using the cannon method, P P becomes C1 R1 C2 C3

p11 (2) p12 (2)

R2 R3

Using the cannon method, P P becomes C1 R1 C2 C3

p11 (2) p12 (2) p13 (2)

R2 R3

Using the cannon method, P P becomes C1 R1 C2 C3

p11 (2) p12 (2) p13 (2) R2 p21 (2) R3

Using the cannon method, P P becomes C1 R1 C2 C3

p11 (2) p12 (2) p13 (2)

R2 p21 (2) p22 (2) p23 (2) R3 p31 (2) p32 (2) p33 (2)

Using the cannon method, P P becomes C1 R1 C2 C3

p11 (2) p12 (2) p13 (2)

R2 p21 (2) p22 (2) p23 (2) R3 p31 (2) p32 (2) p33 (2)

That is, P 2 is the matrix of two-step transition probabilities. This can be denoted by P (2). P (2) = P 2

Summary
The probability of a transition from state i to state j in 2 steps is pij (2) = the i , j entry in P 2 .

Summary
The probability of a transition from state i to state j in 2 steps is pij (2) = the i , j entry in P 2 .

More generally, The probability of a transition from state i to state j in k steps is pij (k ) = the i , j entry in P k .

Example: Suppose IU is playing basketball against Purdue. State 1 is IU has just scored; state 2 is Purdue has just scored. When IU scores, Purdue gets the ball, so in that case Purdue has a 70% chance of scoring next. When Purdue scores, IU gets the ball, and then IU has an 80% of scoring next. Write out the transition matrix P and answer the following questions. 1. If Purdue has just scored, what is the probability that IU scores the next 2 baskets? 2. If Purdue has just scored, what is the probability that IU scores the basket after the next? 3. If Purdue has just scored, what is the probability that Purdue scores the basket after the next? 4. If Purdue has just scored, what is the probability that IU scores the 4th basket from now?

Example: Suppose IU is playing basketball against Purdue. State 1 is IU has just scored; state 2 is Purdue has just scored. When IU scores, Purdue gets the ball, so in that case Purdue has a 70% chance of scoring next. When Purdue scores, IU gets the ball, and then IU has an 80% of scoring next. Write out the transition matrix P and answer the following questions. Transition matrix: IU IU Purdue

Example: Suppose IU is playing basketball against Purdue. State 1 is IU has just scored; state 2 is Purdue has just scored. When IU scores, Purdue gets the ball, so in that case Purdue has a 70% chance of scoring next. When Purdue scores, IU gets the ball, and then IU has an 80% of scoring next. Write out the transition matrix P and answer the following questions. Transition matrix: IU IU 0.3 Purdue

Example: Suppose IU is playing basketball against Purdue. State 1 is IU has just scored; state 2 is Purdue has just scored. When IU scores, Purdue gets the ball, so in that case Purdue has a 70% chance of scoring next. When Purdue scores, IU gets the ball, and then IU has an 80% of scoring next. Write out the transition matrix P and answer the following questions. Transition matrix: IU IU Purdue 0.3 Purdue 0.7

Example: Suppose IU is playing basketball against Purdue. State 1 is IU has just scored; state 2 is Purdue has just scored. When IU scores, Purdue gets the ball, so in that case Purdue has a 70% chance of scoring next. When Purdue scores, IU gets the ball, and then IU has an 80% of scoring next. Write out the transition matrix P and answer the following questions. Transition matrix: IU IU Purdue 0.3 0.8 Purdue 0.7

Example: Suppose IU is playing basketball against Purdue. State 1 is IU has just scored; state 2 is Purdue has just scored. When IU scores, Purdue gets the ball, so in that case Purdue has a 70% chance of scoring next. When Purdue scores, IU gets the ball, and then IU has an 80% of scoring next. Write out the transition matrix P and answer the following questions. Transition matrix: IU IU Purdue 0.3 0.8 Purdue 0.7 0.2

P=

0.3 0.7 0.8 0.2

States: IU, PU P= 0.3 0.7 0.8 0.2

1: If Purdue has just scored, what is the probability that IU scores the next 2 baskets?

States: IU, PU P= 0.3 0.7 0.8 0.2

1: If Purdue has just scored, what is the probability that IU scores the next 2 baskets? Solution: PU IU IU p21 p11 = (.8)(.3) = .24
.8 .3

States: IU, PU P= 0.3 0.7 0.8 0.2

2: If Purdue has just scored, what is the probability that IU scores the basket after the next?

States: IU, PU P= 0.3 0.7 0.8 0.2

2: If Purdue has just scored, what is the probability that IU scores the basket after the next? Solution: We want p21 (2), which is the 2, 1 entry in P 2 .

States: IU, PU P= 0.3 0.7 0.8 0.2

2: If Purdue has just scored, what is the probability that IU scores the basket after the next? Solution: We want p21 (2), which is the 2, 1 entry in P 2 . P2 = .65 .35 .40 .60

p21 (2) = .40

States: IU, PU P= 0.3 0.7 0.8 0.2

2: If Purdue has just scored, what is the probability that IU scores the basket after the next? Solution: We want p21 (2), which is the 2, 1 entry in P 2 . P2 = .65 .35 .40 .60

p21 (2) = .40 Note that the rows in P 2 also sum to 1.

States: IU, PU P = 0.3 0.7 0.8 0.2 .65 .35 .40 .60

P2 =

3: If Purdue has just scored, what is the probability that Purdue scores the basket after the next?

States: IU, PU P = 0.3 0.7 0.8 0.2 .65 .35 .40 .60

P2 =

3: If Purdue has just scored, what is the probability that Purdue scores the basket after the next? Solution: We want p22 (2), which is the 2, 2 entry in P 2 . p22 (2) = .60

States: IU, PU P = 0.3 0.7 0.8 0.2 .65 .35 .40 .60

P2 =

3: If Purdue has just scored, what is the probability that Purdue scores the basket after the next? Solution: We want p22 (2), which is the 2, 2 entry in P 2 . p22 (2) = .60

Alternatively, since this M.C. has only two states p22 (2) = 1 p21 (2) = 1 .40 = .60, using our previous answer.

States: IU, PU P = 0.3 0.7 0.8 0.2 .65 .35 .40 .60

P2 =

4: If Purdue has just scored, what is the probability that IU scores the 4th basket from now?

States: IU, PU P = 0.3 0.7 0.8 0.2 .65 .35 .40 .60

P2 =

4: If Purdue has just scored, what is the probability that IU scores the 4th basket from now? Solution: We want p21 (4), which is the 2, 1 entry in P 4 . P 4 = (P 2 )2 = p21 (4) = .5 .5625 .4375 .5000 .5000

State Vectors
Even when were not sure which state a M.C. is currently in, we can still describe the situation with a state vector.

State Vectors
Even when were not sure which state a M.C. is currently in, we can still describe the situation with a state vector. If a M.C. has two states, and we have no idea which state it is in now, we can say that its state vector X is X = 0.5 0.5 .

The rst number is probability of being in state 1, the second is the probability of being in state 2.

State Vectors
Even when were not sure which state a M.C. is currently in, we can still describe the situation with a state vector. If a M.C. has two states, and we have no idea which state it is in now, we can say that its state vector X is X = 0.5 0.5 .

The rst number is probability of being in state 1, the second is the probability of being in state 2. Note that the numbers in the state vector are probabilities, which must sum to 1.

Example: Suppose a two state M.C. is twice as likely to now be in state 1 as to be in state 2. What is the state vector?

Example: Suppose a two state M.C. is twice as likely to now be in state 1 as to be in state 2. What is the state vector? Solution: X =

Example: Suppose a two state M.C. is twice as likely to now be in state 1 as to be in state 2. What is the state vector? Solution: X = 2x x

Example: Suppose a two state M.C. is twice as likely to now be in state 1 as to be in state 2. What is the state vector? Solution: X = 2x x Since the entries must sum to 1, 2x + x x = 1 =
1/ 3

Example: Suppose a two state M.C. is twice as likely to now be in state 1 as to be in state 2. What is the state vector? Solution: X = 2x x Since the entries must sum to 1, 2x + x x X = 1 = =
1/ 3 2/ 3 1/ 3

Example: Suppose a 4 state M.C. now has a 20% probability of being in state 1, equal probabilities of being in states 2 or 3, and three times the probability of being in state 4 as in state 3. What is its current state vector?

Example: Suppose a 4 state M.C. now has a 20% probability of being in state 1, equal probabilities of being in states 2 or 3, and three times the probability of being in state 4 as in state 3. What is its current state vector? Solution: X =

Example: Suppose a 4 state M.C. now has a 20% probability of being in state 1, equal probabilities of being in states 2 or 3, and three times the probability of being in state 4 as in state 3. What is its current state vector? Solution: X = 0.2 x x 3x

Example: Suppose a 4 state M.C. now has a 20% probability of being in state 1, equal probabilities of being in states 2 or 3, and three times the probability of being in state 4 as in state 3. What is its current state vector? Solution: X = 0.2 x x 3x Since the entries must sum to 1, 0.2 + x + x + 3x 5x x = 1 = 0 .8 = 0.16

Example: Suppose a 4 state M.C. now has a 20% probability of being in state 1, equal probabilities of being in states 2 or 3, and three times the probability of being in state 4 as in state 3. What is its current state vector? Solution: X = 0.2 x x 3x Since the entries must sum to 1, 0.2 + x + x + 3x 5x x X = 1 = 0 .8 = 0.16 = 0.2 0.16 0.16 0.48

Any uncertainty about the current state is likely to be magnied for future states, so well need state vectors to describe future situations as well. X0 = initial state vector X1 = state vector after 1 step X2 = state vector after 2 steps . . . Xk = state vector after k steps

Recall our previous result. The probability of a transition from state i to state j in k steps is pij (k ) = the i , j entry in P k . In English: every factor of P takes you one step further into the future.

The same is true for the state vectors: X1 = X0 P X2 = X1 P = X0 P 2 Xk +1 = Xk P = X0 P k .

The same is true for the state vectors: X1 = X0 P X2 = X1 P = X0 P 2 Xk +1 = Xk P = X0 P k . Example: Suppose a two state M.C. with transition matrix P= 0.4 0.6 0.2 0.8

is equally likely initially to be in either state. What is the probability it is in state 2 after two steps?

The same is true for the state vectors: X1 = X0 P X2 = X1 P = X0 P 2 Xk +1 = Xk P = X0 P k . Example: Suppose a two state M.C. with transition matrix P= 0.4 0.6 0.2 0.8

is equally likely initially to be in either state. What is the probability it is in state 2 after two steps? Solution: X2 = X0 P 2 = 0.5 0.5 0.4 0.6 0.2 0.8 0.4 0.6 0.2 0.8

The same is true for the state vectors: X1 = X0 P X2 = X1 P = X0 P 2 Xk +1 = Xk P = X0 P k . Example: Suppose a two state M.C. with transition matrix P= 0.4 0.6 0.2 0.8

is equally likely initially to be in either state. What is the probability it is in state 2 after two steps? Solution: X2 = X0 P 2 = 0.5 0.5 0.4 0.6 0.2 0.8 0.3 0.7 0.4 0.6 0.2 0.8 0.4 0.6 0.2 0.8 0.26 0.74

= =

Lets continue this example. P = X0 = X1 = X2 = X3 = X32 = 0.4 0.6 0.2 0.8 0.5 0.5 0.3 0.7 0.26 0.74 0.252 0.748

Lets continue this example. P = X0 = X1 = X2 = X3 = X32 = X = 0.4 0.6 0.2 0.8 0.5 0.5 0.3 0.7 0.26 0.74 0.252 0.748 .2500000000000006 .7500000000000018

Lets continue this example. P = X0 = X1 = X2 = X3 = X32 = X = 0.4 0.6 0.2 0.8 0.5 0.5 0.3 0.7 0.26 0.74 0.252 0.748 .2500000000000006 .7500000000000018 .25 .75

X = What does X mean?

.25 .75

X = What does X mean?

.25 .75

You could say it contains the probabilities of being in each state over the last 32 steps of the process.

X = What does X mean?

.25 .75

You could say it contains the probabilities of being in each state over the last 32 steps of the process. We call X the vector of stable probabilities and we denote it by W , because this is Finite Math after all.

X = What does X mean?

.25 .75

You could say it contains the probabilities of being in each state over the last 32 steps of the process. We call X the vector of stable probabilities and we denote it by W , because this is Finite Math after all. You can think of W as containing the long-term probabilities of each state, the fraction of time the M.C. spends in each state in the long run.

How can we calculate W without using a computer?

How can we calculate W without using a computer? Notice that W has two important characteristics: 1. it is a state vector, so its entries sum to 1;

How can we calculate W without using a computer? Notice that W has two important characteristics: 1. it is a state vector, so its entries sum to 1; 2. it is stable, meaning WP = W . The last property is because WP = X P represents the state vector after + 1 steps, which should be the same as X .

How can we calculate W without using a computer? Notice that W has two important characteristics: 1. it is a state vector, so its entries sum to 1; 2. it is stable, meaning WP = W . The last property is because WP = X P represents the state vector after + 1 steps, which should be the same as X . Each of these properties can be expressed in matrix equation form

1. W is a state vector, so its entries sum to 1.

1. W is a state vector, so its entries sum to 1. Think of W as W = w1 w2 w3 . . . wn .

Then property 1 is equivalent to w1 + w2 + w3 + + wn = 1

1. W is a state vector, so its entries sum to 1. Think of W as W = w1 w2 w3 . . . wn .

Then property 1 is equivalent to w1 + w2 + w3 + + wn = 1 1 1 1 ... 1 w1 w2 w3 . . . wn = 1

2. W is stable, meaning WP = W .

2. W is stable, meaning WP = W . Well just do a little elementary algebra: WP = W WP W (where 0 is the zero matrix ) W( = 0

2. W is stable, meaning WP = W . Well just do a little elementary algebra: WP = W WP W (where 0 is the zero matrix ) W (P I ) = 0 = 0

W (P I ) = 0 Now theres a slight problem: In chapters 5 and 6 we learned about matrix methods for solving systems of equations. In all those methods we were solving for a column vector, on the right . For example, AX = B .

W (P I ) = 0 Now theres a slight problem: In chapters 5 and 6 we learned about matrix methods for solving systems of equations. In all those methods we were solving for a column vector, on the right . For example, AX = B . But here we are solving for W , which is a row vector on the left . Is everything we learned previously useless here?

Theres simple trick to save us: turn everything sideways.

Theres simple trick to save us: turn everything sideways. This is called transposing a matrix. Example: A = 1 2 3 4 5 6

At =

Theres simple trick to save us: turn everything sideways. This is called transposing a matrix. Example: A = At = 1 2 3 4 5 6 1 4 2 5 3 6

When you transpose matrices, you also have to reverse the order in which they are multiplied: Youre switching the positions of the lower left and upper right cannons. 7 8 9 1 2 3 4 5 6 becomes 1 4 2 5 3 6 7 8 9

So well just transpose the equation W (P I) = 0: (P I)t W t = 0

So well just transpose the equation W (P I) = 0: (P I)t W t = 0

Notice that the other equation can written in a similar way. w1 w2 1 1 1 . . . 1 w3 = 1 . . . wn 1 1 1 ... 1 Wt = 1

1 1 1 ...

Wt = 1

(P I)t W t = 0 One great feature of matrix equations is that theyre like modular furniture. 1 1 1 1 ... 1 0 t W = 0 t (P I ) . . . 0 stackable,

1 1 1 ... (P I)t 1 t W =

1 0 0 . . . 0

Now we can apply the method of chapter 5: convert to augmented matrix form 1 1 1 ... 1 1 0 t (P I) 0 0 and then row reduce.

Method to Calculate W

To the nd the vector W of stable probabilities for a M.C. with transtion matrix P , form the augmented matrix 1 1 1 ... 1 1 0 t 0 (P I) 0 and row reduce. At the end you will have W t , i.e. W written as a column vector.

Example: Find W for the M.C. in the previous example, with P= 0.4 0.6 0.2 0.8

Example: Find W for the M.C. in the previous example, with P= 0.4 0.6 0.2 0.8

Solution: P I = 0.4 0.6 0.2 0.8 1 0 0 1

0.6 0.6 0.2 0.2

Example: Find W for the M.C. in the previous example, with P= 0.4 0.6 0.2 0.8

Solution: P I = 0.4 0.6 0.2 0.8 1 0 0 1

0.6 0.6 0.2 0.2 0.6 0.2 0.6 0.2

(P I)t =

(P I)t =

0.6 0.2 0.6 0.2

Therefore the augmented matrix we need is 1 1 1 0.6 0.2 0 . 0.6 0.2 0

(P I)t =

0.6 0.2 0.6 0.2

Therefore the augmented matrix we need is 1 1 1 0.6 0.2 0 . 0.6 0.2 0 Now we row reduce. 0.6R1 R2 :
+

1 1 1 0 0.8 0.6 0 0.6 0.2

(P I)t =

0.6 0.2 0.6 0.2

Therefore the augmented matrix we need is 1 1 1 0.6 0.2 0 . 0.6 0.2 0 Now we row reduce. 0.6R1 R2 :
+

1 1 1 0 0.8 0.6 0 0.6 0.2 1 1 1 0 0.8 0.6 0 0.8 0.6

0.6R1 R3 :

1 1 1 0 0.8 0.6 0 0.8 0.6 Notice that row 3 is an exact multiple of row 2. Therefore we can cancel one of them. 1 1 1 0 0.8 0.6

1 1 1 0 0.8 0.6 0 0.8 0.6 Notice that row 3 is an exact multiple of row 2. Therefore we can cancel one of them. 1 1 1 0 0.8 0.6
5/ 4

R2 :

1 1 1 0 1 0.75

1 1 1 0 0.8 0.6 0 0.8 0.6 Notice that row 3 is an exact multiple of row 2. Therefore we can cancel one of them. 1 1 1 0 0.8 0.6
5/ 4

R2 :

1 1 1 0 1 0.75 1 0 0.25 0 1 0.75

1R2 R1 :

1 1 1 0 0.8 0.6 0 0.8 0.6 Notice that row 3 is an exact multiple of row 2. Therefore we can cancel one of them. 1 1 1 0 0.8 0.6
5/ 4

R2 :

1 1 1 0 1 0.75 1 0 0.25 0 1 0.75

1R2 R1 : So that W =

0.25 0.75

Mysteriously, some students do not enjoy calculating stable vectors. If you are repulsed by my method, have a look at the method shown in the book (section 8.3). If youre good at algebra you can solve for W by converting WP = W into ordinary equations and throwing in w1 + w2 + w3 + + wn = 1 to get a system of equations you can then solve to get the the w s.

Regular Markov Chains


Ive misled you. There are M.C.s for which X does not exist.

Regular Markov Chains


Ive misled you. There are M.C.s for which X does not exist.

Regular Markov Chains


Ive misled you. There are M.C.s for which X does not exist.

If this M.C. starts in state 1, then X0 = X1 = X2 = 1 0 0 1 1 0

Regular Markov Chains


Ive misled you. There are M.C.s for which X does not exist.

If this M.C. starts in state 1, then X0 = X1 = X2 = Xeven = Xodd = 1 0 0 1 1 0 1 0 0 1

Similarly with this M.C.

Similarly with this M.C.

The problem is: not enough connections between the states.

The most connections possible: every state is connected to every other state.

The most connections possible: every state is connected to every other state. You can recognize that immediately when you look at the transition matrix. .2 .3 .5 P = .1 .2 .7 .4 .5 .1

The most connections possible: every state is connected to every other state. You can recognize that immediately when you look at the transition matrix. .2 .3 .5 .2 0 .8 P = .1 .2 .7 P = .1 .9 0 .4 .5 .1 .4 .5 .1

The most connections possible: every state is connected to every other state. You can recognize that immediately when you look at the transition matrix. .2 .3 .5 .2 0 .8 P = .1 .2 .7 P = .1 .9 0 .4 .5 .1 .4 .5 .1 On the left: fully connected. On the right: some connections missing.

Do we need every connection to be present in order for X to exist? No, but they must eventually be present.

Do we need every connection to be present in order for X to exist? No, but they must eventually be present. In other words, if we wait long enough all transitions become possible.

Do we need every connection to be present in order for X to exist? No, but they must eventually be present. In other words, if we wait long enough all transitions become possible. A regular Markov chain is for which there is some number k such that P (k ) has all positive entries.

Example: The M.C. with transition matrix .2 0 .8 P = .1 .9 0 .4 .5 .1 has some connections missing, but what if we are patient?

Example: The M.C. with transition matrix .2 0 .8 P = .1 .9 0 .4 .5 .1 has some connections missing, but what if .36 .40 P (2) = P 2 = .11 .81 .17 .50 we are patient? .24 .08 .33

Example: The M.C. with transition matrix .2 0 .8 P = .1 .9 0 .4 .5 .1 has some connections missing, but what if .36 .40 P (2) = P 2 = .11 .81 .17 .50 All two step transitions are possible. This M.C. is regular. we are patient? .24 .08 .33

Determining whether a M.C. is regular

Suppose a Markov chain has transition matrix P . 1. Raise P to higher and higher powers. 2. If the zeros eventually disappear then the M.C. is regular. 3. If notice a repeating cycle containing zeros then it is not regular.

Determining whether a M.C. is regular

Suppose a Markov chain has transition matrix P . 1. Raise P to higher and higher powers. 2. If the zeros eventually disappear then the M.C. is regular. 3. If notice a repeating cycle containing zeros then it is not regular. Two things make this easy : 1. dont do any arithmetic: just note 0s and +s; 2. use repeated squarings to look farther faster.

Example: Is the M.C. with transition matrix .2 0 .8 P = 0 .9 0 .4 0 .1 regular or not?

Example: Is the M.C. with transition matrix .2 0 .8 P = 0 .9 0 .4 0 .1 regular or not? Solution: First: well ignore the numbers and just keep track of 0s and +s. + 0 + P= 0 + 0 + 0 +

0 + 0 P= + 0 + 0 + 0 Now well compute P 2 = P (2). Keep in mind the following simple rules. + + = + 0 + 0 = 0 0 () = 0 + + () = +

0 + 0 P= + 0 + 0 + 0 Now well compute P 2 = P (2). Keep rules. + + = + 0 0 () = 0 + + 0 P2 = 0 + + 0 in mind the following simple + 0 = 0 + () = + + 0 +

Thats an improvement: 4 zeros instead of 5. Whats next?

0 + 0 P= + 0 + 0 + 0 Now well compute P 2 = P (2). Keep rules. + + = + 0 0 () = 0 + + 0 P2 = 0 + + 0 in mind the following simple + 0 = 0 + () = + + 0 +

Thats an improvement: 4 zeros instead of 5. Whats next? P

0 + 0 P= + 0 + 0 + 0 Now well compute P 2 = P (2). Keep rules. + + = + 0 0 () = 0 + + 0 P2 = 0 + + 0 in mind the following simple + 0 = 0 + () = + + 0 +

Thats an improvement: 4 zeros instead of 5. Whats next? + 0 + P 4 = (P 2 )2 = 0 + 0 + 0 +

0 + 0 P + 0 + 0 + 0

+ 0 + P2 = 0 + 0 + 0 +

+ 0 + P4 = 0 + 0 + 0 +

Now we have stopped making progress: weve entered a cycle.

0 + 0 P + 0 + 0 + 0

+ 0 + P2 = 0 + 0 + 0 +

+ 0 + P4 = 0 + 0 + 0 +

Now we have stopped making progress: weve entered a cycle. Since there are still zeros, well never get rid of them. The M.C. is not regular.

Example: Is the M.C. with transition matrix 1 0 0 P= 1 0 0 0 .2 .8 regular or not?

Example: Is the M.C. with transition matrix 1 0 0 P= 1 0 0 0 .2 .8 regular or not? Solution: First: well ignore the numbers and just keep track of 0s and +s. + 0 0 P= + 0 0 0 + +

+ 0 0 P= + 0 0 0 + + Now start repeatedly squaring. P2

+ 0 0 = + 0 0 + + +

+ 0 0 P= + 0 0 0 + + Now start repeatedly squaring. P2

+ 0 0 = + 0 0 + + + + 0 0 = + 0 0 + + +

P4

+ 0 0 P= + 0 0 0 + + Now start repeatedly squaring. P2

+ 0 0 = + 0 0 + + + + 0 0 = + 0 0 + + +

P4

Not regular.

Example: Is the M.C. with transition matrix 0 0 1 P= 0 0 1 .8 .2 0 regular or not?

Example: Is the M.C. with transition matrix 0 0 1 P= 0 0 1 .8 .2 0 regular or not? Solution: First: well ignore the numbers and just keep track of 0s and +s. 0 0 + P= 0 0 + + + 0

0 0 + P= 0 0 + + + 0 Now start repeatedly squaring. P2

+ + 0 = + + 0 0 0 +

0 0 + P= 0 0 + + + 0 Now start repeatedly squaring. P2

+ + 0 = + + 0 0 0 + + + 0 = + + 0 0 0 +

P4

0 0 + P= 0 0 + + + 0 Now start repeatedly squaring. P2

+ + 0 = + + 0 0 0 + + + 0 = + + 0 0 0 +

P4

Not regular.

Example: Is the M.C. with transition matrix 0 1 0 P= 0 0 1 .8 .2 0 regular or not?

Example: Is the M.C. with transition matrix 0 1 0 P= 0 0 1 .8 .2 0 regular or not? Solution: First: well ignore the numbers and just keep track of 0s and +s. 0 + 0 P= 0 0 + + + 0

0 + 0 P= 0 0 + + + 0 Now start repeatedly squaring. P2

0 0 + = + + 0 0 + +

0 + 0 P= 0 0 + + + 0 Now start repeatedly squaring. P2

0 0 + = + + 0 0 + + 0 + + = + + + + + +

P4

0 + 0 P= 0 0 + + + 0 Now start repeatedly squaring. P2

0 0 + = + + 0 0 + + 0 + + = + + + + + + + + + = + + + + + +

P4

P8

0 + 0 P= 0 0 + + + 0 Now start repeatedly squaring. P2

0 0 + = + + 0 0 + + 0 + + = + + + + + + + + + = + + + + + +

P4

P8

Regular.

Das könnte Ihnen auch gefallen